You are on page 1of 160

CHNG 1: HM GII TCH

1. S PHC V CC PHP TNH 1. Dng i s ca s phc: Ta gi s phc l mt biu thc dng (x + jy) trong x v y l cc s thc v j l n v o. Cc s x v y l phn thc v phn o ca s phc. Ta thng k hiu: z = x + jy x = Rez = Re(x + jy) y = Imz = Im(x + jy) Tp hp cc s phc c k hiu l C. Vy: C = { z = x + jy | x R , y R} trong R l tp hp cc s thc. Nu y = 0 ta c z = x, ngha l s thc l trng hp ring ca s phc vi phn o bng 0. Nu x = 0 ta z = jy v l mt s thun o. S phc z = x jy c gi l s phc lin hp ca z = x + jy. Vy Re(z) = Re(z) ,

Im(z ) = Im(z) , z = z . S phc -z = -x - jy l s phc i ca z = x + jy. Hai s phc z1 = x1 + jy1 v z2 = x2 + jy2 gi l bng nhau nu x1 = x2 v y1 = y2.
2. Cc php tnh v s phc: a. Php cng: Cho hai s phc z1 = x1 + jy1 v z2 = x2 + jy2. Ta gi s phc z = (x1 + x2 ) + j(y1 + jy2 ) l tng ca hai s phc z1 v z2. Php cng c cc tnh cht sau: (giao hon) z1 + z2 = z2 + z1 (kt hp) z1 + (z2 + z3) = (z1 + z2) + z3 b. Php tr: Cho 2 s phc z1 = x1 + jy1 v z2 = x2 + jy2. Ta gi s phc z = (x1 - x2 ) + j(y1 - jy2 ) l hiu ca hai s phc z1 v z2. c. Php nhn: Cho 2 s phc z1 = x1 + jy1 v z2 = x2 + jy2. Ta gi s phc z = z1.z2 = (x1x2-y1y2) + j(x1y2 + x2y1) l tch ca hai s phc z1 v z2. Php nhn c cc tnh cht sau: (tnh giao hon) z1,z2 = z2.z1 (z1.z2).z3 = z1.(z2.z3) (tnh kt hp) z1(z2 + z3) = z1.z2 + z2.z3 (tnh phn b) (-1.z) = -z z.0 = 0. z = 0 j.j = -1 d. Php chia: Cho 2 s phc z1 = x1 + jy1 v z2 = x2 + jy2. Nu z2 0 th tn ti mt s phc z = x + jy sao cho z.z2 = z1. S phc:
1

z=

y x 2 y 2 x1 z1 x1x 2 + y 1 y 2 = +j 1 2 2 2 z2 x2 + y2 x2 + y2 2

c gi l thng ca hai s phc z1 v z2. e. Php nng ln lu tha: Ta gi tch ca n s phc z l lu tha bc n ca z v k hiu: z n = z.z L z t w = zn =(x + jy)n th theo nh ngha php nhn ta tnh c Rew v Imw theo x v y. Nu zn = w th ngc li ta ni z l cn bc n ca w v ta vit: z=n w f. Cc v d: V d 1: j2 = -1 j3 = j2.j = -1.j = -j V d 2: (2+j3) + (3-5j) = 5-2j
1 = j j

2 + 5 j (2 + 5 j)(1 + j) 3 + 7 j 3 7 = = = + j 2 1 j 1 j 2 2 2 z + z = ( x + jy) + ( x jy) = 2x = 2 Re z V d 3: V d 4: Tm cc s thc tho mn phng trnh: (3x - j)(2 + j)+ (x - jy)(1 + 2j) = 5 + 6j Cn bng phn thc v phn o ta c: 20 36 x= y= 17 17 V d 5: Gii h phng trnh: z + j = 1 2 z + = 1 + j Ta gii bng cch dng phng php Cramer v c kt qu: 1 j

z=

1+ j 1 2 j (2 j)(1 + 2 j) 4 + 3 j = = = 1 j 1 2j 5 5 2 1

1 j 2 1+ j j 1 ( j 1)(1 + 2 j) 3 j = = = = 1 j 1 2j 5 5 2 1 V d 6: Chng minh rng nu a thc P(z) l mt a thc ca bin s phc z vi cc h s thc:
2

P(z) = a0zn + a1zn-1 + + an th P (z ) = P ( z ) Tht vy ta thy l s phc lin hp ca tng bng tng cc s phc lin hp ca tng s hng, s phc lin hp ca mt tch bng tch cc s phc lin hp ca tng tha s. Do vy: a k z n k = a k .z n k Do :

P( z ) = a k z n k = a k z n k = a k z n k = P( z )
k =0 k =0 k =0

T kt qu ny suy ra nu a thc P(z) c cc h s thc v nu l mt nghim phc ca n tc P() = 0 th cng l nghim ca n, tc P( ) = 0.


3. Biu din hnh hc: Cho s phc z = x + jy. Trong mt phng xOy ta xc nh im M(x,y) gi l to v ca s phc z. Ngc li cho im M trong mt phng, ta bit to (x,y) v lp c s phc z = x + jy. Do ta gi xOy l mt phng phc. Ta cng c th biu din s phc bng mt vec t t do c to l (x,y). 4. Moun v argumen ca s phc z: S phc z c to v l M. Ta gi di r ca vec t OM l moun ca z v k hiu l z . Gc xc nh sai khc 2k c gi l argumen ca z v k hiu l Argz: M r = z = OM y r Argz = Ox, OM = + 2 k c bit, tr s ca Argz nm gia - v gi l gi x O tr chnh ca Argz v k hiu l argz. Trng hp z = 0 th Argz khng xc nh. Gia phn thc, phn o, moun v argumen c lin h: x = rcos y = rsin r = x 2 + y2 y tg = x

y acrtg x y arg z = + acrtg x y + acrtg x

khi x > 0 khi x < 0, y 0 khi x < 0, y < 0

Vi x = 0 t nh ngha ta c:
3

arg z = 2 2

khi y > 0 khi y < 0

Hai s phc bng nhau c moun v argumen bng nhau. z = z

z.z = z T cch biu din s phc bng vec t ta thy s phc (z1 - z2) biu din khong cch t im M1 l to v ca z1 n im M2 l to v ca z2. T suy ra | z | = r biu th ng trn tm O, bn knh r. Tng t | z - z1 | = r biu th ng trn tm z1, bn knh r; | z - z1 | > r l phn mt phc ngoi ng trn v | z - z1 | < r l phn trong ng trn . Hn na ta c cc bt ng thc tam gic: | z1 + z2 | | z1 | + | z2 | ; | z1 - z2 | || z1 | - | z2 || T nh ngha php nhn ta c: z1.z2 = r1.r2 [(cos1cos2 - sin1sin2) - j(sin1cos2 + sin2cos2)] = r1.r2 [cos(1 + 2) + jsin(1 + 2)] Vy: | z1.z2 | = | z1 |.| z2 | Arg(z1.z2 ) = Argz1 + Argz2 + 2k Tng t, nu z2 0 th: z1 r1 = [cos(1 - 2) + jsin(1 - 2)] z 2 r2
z z1 = 1 z2 z2 z1 Arg z = Argz1 + Argz2 + 2k 2
5. Cc v d: V d 1: 3 + 2 j = 32 + 2 2 = 13 V d 2: Vit phng trnh ng trn A(x2 + y2) + 2Bx + 2Cy + D = 0 vi cc h s A, B, C, D l cc s thc trong mt phng phc. Ta t z = x + jy nn z = x jy . x 2 + y 2 =| z |2 = z.z Mt khc 2x = z + z zz 2y = = j(z z ) j Thay vo phng trnh ta c: Azz + B(z + z ) Cj(z z ) = 0
4

hay Az z + Ez + Ez + D = 0 6. Dng lng gic ca s phc: Nu biu din s phc z theo r v ta c: z = x + jy = r(cos + jsin) y l dng lng gic s phc z. V d: z = -2 = 2(cos + jsin ) Cc php nhn chia dng s phc di dng lng gic rt tin li. Ta c: z1 = r1 (cos + j sin )
z 2 = r2 (cos + j sin ) z= z = z1.z 2 = r1r2 [cos( + ) + j sin ( + )] z1 r1 = [cos( ) + j sin ( )] z 2 r2 p dng cng thc trn tnh tch n tha s z, tc l zn. ta c: [r(cos + jsin)]n = rn(cosn + jsinn) c bit khi r = 1 ta c cng thc Moivre: (cos + jsin)n = (cosn + jsinn) Thay bng - ta c: (cos - jsin)n = (cosn - jsinn) V d: Tnh cc tng: s = cos + cos2 + + cosn t = sin + sin2 + + sinn Ta c jt = jsin + jsin2 + + jsinn t z = cos + jsin v theo cng thc Moivre ta c: s + jt = z + z2 + + zn V phi l mt cp s nhn gm n s, s hng u tin l z v cng bi l z. Do ta c:
z n 1 z n+1 z cos( n + 1) + j sin( n + 1) cos j sin = = s + jt = z z 1 z 1 cos + j sin 1 [cos( n + 1) cos ] + j[sin(n + 1) sin ] = (cos 1) + j sin [cos( n + 1) cos ] + j[sin(n + 1) sin ] (cos 1) j sin = . (cos 1) + j sin (cos 1) j sin

Nh vy:
cos(n + 1). cos cos 2 cos(n + 1) + cos + sin(n + 1). sin sin 2 s = Re(s + jt ) = (cos 1) 2 + sin 2 cos(n + 1). cos + sin(n + 1). sin cos(n + 1) + cos 1 = 2 2 cos cos cos(n + 1) + cos n 1 = 2(1 cos )
5

Tng t ta tnh c t = Im(s+jt) Khi biu din s phc di dng lng gic ta cng d tnh c cn bc n ca n. Cho s phc z = r(cos + jsin) ta cn tm cn bc n ca z, ngha l tm s phc sao cho: n = z trong n l s nguyn dng cho trc. Ta t = (cos + jsin) th vn l phi tm v sao cho: n(cosn + jsinn) = r(cos + jsin) Ngha l n = r v n = + 2 k Kt qu l: = n r ; = n C th, cn bc n ca z l s phc: o = n r cos + j sin n n + 2 + 2 1 = n r cos + j sin n n . . . . . . + 2(n 1) + 2(n 1) n 1 = n r cos + j sin n n vi k l s nguyn v ch cn ly n s nguyn lin tip (k = 0, 1, 2,...,n-1) v nu k ly hai s nguyn hn km nhau n th ta c cng mt s phc.
7. To v ca s phc tng, hiu, tch v thng hai s phc: a. To v ca tng v hiu: To v ca tng hai s phc l tng hay hiu 2 vec t biu din s phc . b. To v ca tch hai s phc: Ta c th tm to v ca tch hai s phc bng phng php dng hnh. Cho hai s phc z1 v z2 nh hnh v. Ta dng trn cnh Oz1 tam gic Oz1z ng dng vi tam gic O1z2. Nh vy Oz l tch ca hai s phc z1 v z2. Tht vy, do tam gic Oz1z ng dng vi tam gic O1z2 nn ta c:
z z2 = hay z = z1.z2 z1 1

z2z1=z z2 1

z1

c. To v ca thng hai s phc: Vic tm thng hai s phc a v tm tch 1 1 z1. . V vy ta ch cn tm w = . Trc ht ta gi thit | z | < 1(hnh a) z2 z Ta tm w theo cc bc sau: - v ng trn n v v z
6

- dng ti z ng vung vi Oz v ct ng trn n v ti s - v tip tuyn vi ng trn ti s v ct Oz ti t. 1 - do Ozs & Ost ng dng nn ta c | t |= |z| - ly w i xng vi t. Trng hp | z | > 1 ta v nh hnh b: - v ng trn n v v z - t z v tip tuyn vi ng trn ti s - dng ti s ng vung vi Oz ct Oz ti t 1 - do Ozs v Ost ng dng nn ta c | t | = |z| - ly w i xng vi t.

s z O

z s t w w

8. Dng m ca s phc: Nh cng thc Euler e j = cos + j sin ta c th biu din s phc di dng s m: z = rej = | z |ejArgz

V d z = 1 j = 2e Biu din s phc di dng m rt tin li khi cn nhn hay chia cc s phc:
z1 = r1e j z 2 = r2 e j

3 4

z1z 2 = r1r2 e j( + ) z1 r1 j( ) = e z 2 r2
9. Mt cu Rieman: Ta xt mt mt cu S tm (0, 0, 0.5), bn knh 0.5 (tip xc vi mt phng xOy ti O). Mt phng xOy l mt phng phc z vi Ox l trc thc v Oy l trc o. on thng ni im z = x + jy c to v l N ca mt phng phc vi im P(0, 0, 1) ca mt cu ct mt cu ti im M(a, b, c). Ta gi M l hnh chiu
7

ni ca im z ln mt cu S vi cc P. Php nh x ny lp nn mt tng ng mt mt gia tt c cc im ca mt phng z v ca mt cu S thng ti P. V cc im P, M, v N cng nm trn mt ng thng nn ta c: OT a b PM 1 c = = = = P ON x y PN 1 a b 1 c hay = = c x y 1 M a b a + jb hay: x= ;y = ;z = 1 c 1 c 1 c 2 2 (a + b ) 2 O y b T : z = a 2 (1 c) T 2 2 2 x v do : a +b +c -c=0 N c 2 suy ra: z = 1 c 2 z x y c= ;a= ;b= hay: 2 2 2 1+ z 1+ z 1+ z Hnh chiu ni c tnh cht ng lu sau: mi ng trn ca mt phng z(ng thng cng c coi l ng trn c bn knh ) chuyn thnh mt ng trn trn z+z z+z mt cu v ngc li. Tht vy x = ta thy mi ng trn ca ;y = 2 2j mt phng z tho mn mt phng trnh dng: 1 j Azz + B( z + z ) C( z z ) + D = 0 2 2 Trong A, B, C, D l cc s thc tha mn A 0, B2 + C2 > 4AD, c bit i vsi ng thng A = 0. p dng cc gi tr ca z, x, y ta c: (A - D)c +Ba +Cb + D = 0 y l mt ng trn trn mt cu S.
2. HM MT BIN PHC 1. Khi nim v min v bin ca min: a. im trong ca mt tp: Gi s E l tp hp im trong mt phng phc z v zo l mt im thuc E. Nu tn ti mt s ln cn ca zo nm hon ton trong E th zo c gi l im trong ca tp E. b. Bin ca mt tp: im thuc E hay khng thuc E c gi l im bin ca tp E nu mi hnh trn tm u cha c nhng im thuc E v khng thuc E. Tp hp cc im bin ca tp E c gi l bin ca tp E. Nu im khng thuc E v tn ti hnh trn tm khng cha im no ca E th c gi l im ngoi ca tp E.
8

V d: Xt tp E l hnh trn | z | < 1. Mi im ca E u l im trong. Bin ca E l ng trn | z | = 1. Mi im | | > 1 l im ngoi ca E. c. Min: Ta gi min trn mt phng phc l tp hp G c cc tnh cht sau: - G l tp m, ngha l ch c cc im trong. - G l tp lin thng, ngha l qua hai im tu thuc G, bao gi cng c th ni chng bng mt ng cong lin tc nm gn trong G. Tp G, thm nhng im bin gi l tp kn v k hiu l G . Min G gi l b chn nu tn ti mt hnh trong bn knh R cha G bn trong.

Trn hnh a l min n lin, hnh b l min nh lin v hnh c l min tam lin. Hng dng trn bin L ca min l hng m khi i trn L theo hng th phn ca min G k vi ngi lun nm bn tri. V d 1: V min < arg z < 6 3 Ta v tia Ou 1 sao cho ( Ox, Ou 1 ) = . Sau v tia Ou 2 sao cho ( Ox , Ou 2 ) = . 6 3 Mi im z nm trong u1Ou 2 u c argumen tho mn iu kin bi ton. Ngc li cc im c argumen nm gia v u trong gc u 1Ou 2 6 3 Vy min < arg z < l phn mt phng gii hn bi hai cnh Ou1 v Ou2 6 3 y y u2 u1 O x -1 O x

V d 2: V min Rez > -1 Mi im nm bn phi ng thng x = -1 u tho mn Rez > -1. Ngc li mi im z c phn thc ln hn -1 u nm bn phi ng thng x = -1. Vy min Rez > -1 l na mt phng phc gch cho trn hnh v.
9

2. nh ngha hm bin phc: a. nh ngha: Gi s E l mt tp hp im trn mt phng phc. Nu c mt quy lut cho ng vi mi s phc zE mt s phc xc nh w th ta ni rng w l mt hm s n tr ca bin phc z xc nh trn E v k hiu: w = f(z), zE (1) Tp E c gi l min xc nh ca hm s. Nu ng vi mt gi tr zE ta c nhiu gi tr ca w th ta ni w l mt hm a tr. Sau ny khi ni n hm s m khng ni g thm th l mt hm n tr. 1 V d: Hm w = xc nh trong ton b mt phng phc tr im z = 0 z z Hm w = 2 xc nh trong ton b mt phng phc tr im z = j v z2+1 z +1 = 0 khi z = j Hm w = z + z + 1 xc nh trong ton b mt phng phc. y l mt hm a tr. Chng hn, vi z = 0 ta c w = 1 . V 1 = cos0 + j sin0 nn w c hai gi tr: 0 0 w 1 = cos + j sin = 1 2 2 0 + 2 0 + 2 + j sin w 2 = cos = cos + j sin = 1 2 2 nn ng vi z = 0 ta c hai gi tr w1 = 1 v w1 = -1 b. Phn thc v phn o ca hm phc: Cho hm w = f(z) ngha l cho phn thc u v phn o v ca n. Ni khc i u v v cng l hai hm ca z. Nu z= x+jy th c th thy u v v l hai hm thc ca cc bin thc c lp x v y. Tm li. cho hm phc w = f(z) tng ng vi vic cho hai hm bin thc u = u(x, y) v v = v(x, y) v c th vit w = f(z) di dng: w = u(x, y) + jv(x, y) (2) Ta c th chuyn v dng (2) hm phc cho di dng (1). 1 V d 1: Tch phn thc v phn o ca hm phc w = z Ta c: 1 1 x jy x jy x jy = w= = = = 2 z x + jy ( x + jy)( x jy) x + y 2 x 2 + y 2 x 2 + y 2 Vy: x y u= 2 v= 2 2 x +y x + y2 V d 2: Tch phn thc v phn o ca hm w = z3 Ta c: w = z 3 = ( x + jy) 3 = x 3 + 3 jx 2 y + 3 j2 xy 2 + j3 y 3 = ( x 3 3xy 2 ) + j(3x 2 y y 3 ) Vy: u = x 3 3xy 2 v = 3x 2 y y 3

10

V d 3: Cho hm w = x 2 y + j( x + y 2 ) . Hy biu din w theo z = x + jy v z = x jy zz z+z nn: v y = V x= 2 2j 2 z + z z z 2 j z+ z w = + (z z ) + j 2 2 2 2 Rt gn ta c: 1 1 w = (1 j)(z 2 + z 2 ) + (1 + j) zz + jz 4 2


V d 4: Cho w = x2 - y2 + 2jxy. Hy biu din w theo z 2 2 z+z z + z z z 2 z z Ta c: w = +j + 2 j 2j 2 2 2
2 z z2 z + z z z z + z z + z z z Hay: w = 2 = + = z2 + 2 2 2 2 2 2 2 2

3. Php bin hnh thc hin bi hm bin phc: biu din hnh hc mt hm bin s thc ta v th ca hm s . m t hnh hc mt hm bin s phc ta khng th dng phng php th na m phi lm nh sau: Cho hm bin phc w = f(z), zE. Ly hai mt phng phc xOy (mt phng z) v uOv (mt phng w). V mi im z0E ta c mt im w0 = f(z0) trong mt phng w. Cho nn v mt hnh hc, hm w = f(z0 xc nh mt php bin hnh t mt phng z sang mt phng w. im w0 c gi l nh ca z0 v z0 l nghch nh ca w0. Cho ng cong L c phng trnh tham s x = x(t), y = y(t). nh ca L qua php bin hnh w = f(z) = u(x, y) + jv(x, y) l tp hp cc im trong mt phng w c to : u = u[x(t), y(t)] (3) v = v[x(t), y(t)] Thng thng th nh ca ng cong L l ng cong c phng trnh tham s (3) Mun c phng trnh quan h trc tip gia u v v ta kh t trong (3). Mun tm nh ca mt min G ta coi n c qut bi h ng cong L.Ta tm nh ca L. Khi L qut nn min G th qut nn min l nh ca G. 4. Cc hm bin phc thng gp: a. V d 1: Hm w = kz (k > 0) t z = rej , w = ej = krej . Ta c = kr, = + 2k . Vy y l mt php co dn hay php ng dng vi h s k

11

y z x

v w k u

b. V d 2: w = zej ( R) t z = rej , w = ej = rejej = rej(+). Ta c = r, = + + 2k. Nh vy y l php quay mt phng z mt gc .


y z x v w

c. V d 3: w = z + b vi b = b1 + jb2 t z = x + jy w = u + jv, ta c: u = x + b1 ; v = y + b 2 Vy y l mt php tnh tin


y z b x

d. V d 4: w = az + b vi a = kej l php bin hnh tuyn tnh nguyn. N l hp ca ba php bin hnh: - php co dn s = kz - php quay t = sj - php tnh tin w = t + b e. V d 5: w = z2 t z = rej , w = ej ta c: = r2 ; = 2 + 2k. Mi tia z = o bin thnh tia argw = 2o, mi ng trn | z | = ro bin thnh ng trn | w | = ro2 . Nu D = {z: 0 < < 2 } th f(D) = {-w: 0 < < 2 } ngha l na mt phng phc c Imz > 0 bin thnh ton b mt phng phc w.
12

f. V d 6: w = | z |. z t z = rej , w = ej ta c: = r2 ; = + 2k. Min D = {z: 0 < < } c bin n dip ln chnh n, ngha l na mt phng phc Imz > 0 c bin thnh na mt phng phc Imw > 0. g. V d 7: w = 3 z Vi z 0 th w c 3 gi tr khc nhau. t z = rej , w = ej ta c: = 3 r ; 2k k = + . Min D = {z: 0 < < } c nh l ba min: B1 = w : 0 < < ; 3 3 3 2 2 << B2 = w : < < ; B3 = w : 3 3 3 3. O HM CA HM PHC 1. Gii hn ca hm bin phc: nh ngha gii hn v lin tc ca hm bin phc cng tng t nh hm bin thc. a. nh ngha 1: Gi s f(z) l hm xc nh trong ln cn ca zo(c th tr zo). Ta ni s phc A l gii hn ca f(z) khi z dn ti zo nu khi | z - zo | 0 th | f(z)-A0. Ni khc i, vi mi > 0 cho trc, lun lun tn ti > 0 khi | z - zo | < th |f(z)-A| < . Ta k hiu: lim f( z) = A
z z o

D dng thy rng nu f(z) = u(x,y) +jv( x,y) ; zo = xo + jyo; A = + j th:


zzo

lim f( z) = A lim u( x , y) =
x xo y yo

xxo y yo

lim v( x , y) =

Trong mt phng phc, khi z dn ti zo n c th tin theo nhiu ng khc nhau. iu khc vi trong hm bin thc, khi x dn ti xo, n tin theo trc Ox. b. nh ngha 2: Ta ni s phc A l gii hn ca hm w = f(z) khi z dn ra v cng, nu khi | z | + th | f(z) - A | 0. Ni khc i, vi mi > 0 cho trc, lun lun tn ti R > 0 khi | z | > R th | f(z) - A | < . Ta k hiu: lim f( z) = A
z

c. nh ngha 3: Ta ni hm w = f(z) dn ra v cng khi z dn ti zo, nu khi | z - zo | 0 th | f(z) | +. Ni khc i, vi mi M > 0 cho trc ln tu , lun lun tn ti > 0 khi | z - zo | < th | f(z) | > M. Ta k hiu: lim f( z) =
zzo

d. nh ngha 4: Ta ni hm w = f(z) dn ra v cng khi z dn ra v cng, nu khi | z | + th | f(z) | +. Ni khc i, vi mi M > 0 cho trc ln tu , lun lun tn ti R > 0 khi | z | > R th | f(z) | > M. Ta k hiu: lim f( z) =
z

13

2. Hm lin tc: Ta nh ngha hm lin tc nh sau: nh ngha: Gi s w = f(z) l mt hm s xc nh trong mt min cha im zo. Hm c gi l lin tc ti zo nu lim f( z) = f( z o )
z zo

D thy rng nu f(z ) = u(x, y) + jv(x, y) lin tc ti zo = xo + jyo th u(x, y) v v(x, y) l nhng hm thc hai bin, lin tc ti (xo, yo) v ngc li. Hm w = f(z) lin tc ti mi im trong min G th c gi l lin tc trong min G. V d: Hm w = z2 lin tc trong ton b mt phng phc v phn thc u = x2 - y2 v phn o v = 2xy lun lun lin tc.
3. nh ngha o hm: Cho hm w = f(z) xc nh trong mt min cha im z = x + jy. Cho z mt s gia z = x + jy. Gi w l s gia tng ng ca hm: w = f(z + z) - f(z) w dn ti mt gii hn xc nh th gii hn c gi l Nu khi z 0 t s z dw . Ta c: o hm ca hm w ti z v k hiu l f(z) hay w( z ) hay dz w f ( z + z ) f ( z ) (4) f ' ( z) = lim = lim z 0 z z 0 z V mt hnh thc, nh ngha ny ging nh ngha o hm ca hm bin s thc. w phi c cng gii hn khi z 0 theo mi cch. Tuy nhin y i hi z V d 1: Tnh o hm ca w = z2 ti z. Ta c : w = (z + z)2 - z2 = 2z.z + z2 w = 2z + z z w Khi z 0 th 2z. Do vy o hm ca hm l 2z. z V d 2: Hm w = z = x jy c o hm ti z khng Cho z mt s gia z = x + jy. S gia tng ng ca w l: w = z + z z = z + z z = z = x j y w w w =1 Nu y = 0 th z = x khi w = x ; = = 1 nn lim y0 x z x x 0

x = 0 th z = -jy khi w = -jy ;

w w w = 1 = = 1 nn lim y0 x z jy x 0
w c nhng gii hn khc z

Nh vy khi cho z 0 theo hai ng khc nhau t s

nhau. Vy hm cho khng c o hm ti mi z. 3. iu kin kh vi: Nh th ta phi tm iu kin hm c o hm ti z. Ta c nh l sau:


14

nh l: Nu hm w = f(z) = u(x, y) + jv(x, y) c o hm ti z, th phn thc u(x, y) v phn o v(x, y) ca n c o hm ring ti (x, y) v cc o hm ring tho mn h thc:
u v u v = ; = x y y x

(5)

(5) l iu kin Cauchy - Riemann. y l iu kin cn. Ngc li nu cc hm s u(x, y) v v(x, y) c cc o hm ring lin tc, tho mn iu kin C - R th hm w = f(z) c o hm ti z = x + jy v c tnh theo cng thc: f (z ) = ux + jv x y l iu kin . Ta chng minh iu kin cn: Gi s f(z) tn ti, ngha l gii hn ca t s: w u ( x + x , y + y) + jv( x + x , y + y) u ( x , y) v( x , y) = z x + jy [u ( x + x, y + y) u (x, y)] + j[v(x + x, y + y) v(x, y)] = u + jv = x + jy x + jy bng f(z) khi z 0 theo mi cch. c bit khi z = x th: w x u v = +j x z x x Trong u = xu l s gia ring ca u i vi x. Cho x 0, theo gi thit th v tri dn ti f(z). Do v phi cng c gii hn l f(z). T suy ra: u xu c gii hn l x x v xv c gii hn l x x u v (6) +j v: f (z) = x x Tng t, khi z = y th: u w y u + j y v y v = = j y z y jy y v u (7) Cho z 0 ta c: f (z) = j y y So snh (6) v (7) ta c: u v v u +j j x x y y T y ta rt ra iu kin C - R: u v v u = ; = x y x y
15

Tip theo ta chng minh iu kin : Gi s cc hm u(x, y) v v(x, y) c cc o hm ring lin tc ti (x, y) v cc o hm tho mn iu kin C - R. Ta cn w chng minh c gii hn duy nht khi z 0 theo mi cch. z w u + jv = Ta vit: (8) z x + jy T gi thit ta suy ra u(x, y) v v(x, y) kh vi, ngha l: u u u = x + y + 1x + 2 y x y v v v = x + y + 1x + 2 y x y Trong 1, 2, 1, 2 0 khi x 0, y 0(tc l z 0). Thay vo (8) cc kt qu ny ta c: v u u v x + y + 1x + 2 y + j x + y + 1x + 2 y y x y w x = z x + jy v v u u x + y + j x + j y ( + j1 )x + ( 2 + j 2 )y x y x y + 1 = x + jy x + jy Do iu kin C - R, ta c th ly x + jy lm tha s chung trong t s ca s hng th nht bn v phi: u u u v v u u u x + y + j x + j y = x + y j x + j y y x x y x y x y

= (x + jy ) Vy:

u (1 + j1 )x + ( 2 + j 2 )y w u = j + (9) y x + jy z x Ch l khi x 0, y 0 th s hng th 2 bn v phi dn ti 0. Tht vy: x x x = = 1 x + jy x + jy x 2 + y 2

u u u u + (x + jy ) j = (x + jy ) j x y y x

(1 + j1 )

x 1 + j1 x + jy
x 0 x + jy

Khi x 0, y 0 th 1 0 v 1 0, Vy (1 + j1 ) Tng t ta chng minh c rng ( 2 + j 2 )

y 0 x + jy
16

Cho nn nu cho z 0 theo mi cch th v phi ca (9) s c gii hn l u u j . x y Vy v tri cng dn ti gii hn , ngha l ta chng minh rng tn ti u u f (z) = j . x y Do iu kin C - R nn ta c th tnh o hm bng nhiu biu thc khc nhau: u u u v v u v v f (z) = +j = j = j = +j x x x y y x y y x x V d 1: Tm o hm ca hm s w = e cosy + je siny. Hm c o hm ti mi im v iu kin C - R lun lun tho mn. Tht vy: u = excosy, v = exsiny x u x = e cos y = v y

uy = e x sin y = v x
dw = e x cos y + je x sin y = w dz
V d 2: Tm o hm ca hm w = x + 2y + j(2x + y) u = x + 2y v = 2x + y ux = 1 = v y , uy = 2 v x = 2 V d 3: Xt s kh vi ca hm w = z2 = (x2 - y2) + 2jxy. v u v u V ti mi im hu hn. w = z2 kh vi ti mi im = 2x = ; = 2 y = x x y y z v z = 2z. V d 4: Xt s kh vi ca hm w = z.Rez = x2 + jxy. Do h phng trnh: u v = 2x = x = x y u v = 0 = y = y x ch tho mn ti im (0, 0) nn w ch kh vi ti z = 0 4. Cc quy tc tnh o hm: V nh ngha o hm ca hm bin phc ging nh o hm ca hm bin thc, nn cc php tnh o hm ca tng, tch, thng hm hp hon ton tng t nh i vi hm thc. Gi s cc hm f(z) v g(z) c o hm ti z. Khi : [ f(z) + g(z) ] = f(z) + g(z)
17

[ f(z).g(z) ] = f(z).g(z) + g(z).f(z) f (z) f ' (z).g(z) f (z).g' (z) g (z) = g 2 (z) Nu w = f(z) , z = () u l cc hm c o hm, th o hm ca hm hp w = f[()] l: dw dw dz = . d dz d Nu f(z) l hm n dip c hm ngc l h(w), th: 1 f ' (z) = , h' (w ) 0 h' (w )
5. ngha hnh hc ca o hm: Gi thit hm w = f(z) c o hm ti mi im trong ln cn im zo v f(zo) 0. a. ngha hnh hc ca Arg f(zo): Php bin hnh w = f(z) bin im zo thnh im wo = f(zo). Gi Mo l to v ca zo v Po l to v ca wo. Cho mt ng cong bt k i qua Mo v c phng trnh l z(t) = x(t) + jy(t). Gi s: z(to) = x(to) + jy(to) 0 ngha l ha s x(to) v y(to) khng ng thi trit tiu khi t = to. Vy ng cong L c tip tuyn ti Mo m ta gi l MoT. L y v

M T Mo O x O Po

Gi l nh ca ng cong L qua php bin hnh. Hin nhin ng cong i qua im Po v c phng trnh w = w(t) = f[z(t)]. Theo cng thc o hm hm hp ta c w(to) = f(zo).z(to). Theo gi thit th f(zo) 0, z(to) 0 nn w(to) 0. Nh vy ti P0, ng cong c tip tuyn Po. By gi ta ly z l im khc thuc L. N c nh l w . Theo nh ngha o hm: w w0 = f (z o ) lim (12) zzo z z 0

w wo [Arg(w w o ) Arg(z z o )] Argf (z o ) = lim Arg = zlim z z o z o z z o Gi M, P ln lt l to v ca z v w th ng thc trn c vit l:


Vy
18

Argf (z o ) = lim Ou, Po P lim Ox, M o M


PPo P MMo PL

V khi P Po, ct tuyn PoP dn ti tip tuyn Po vi ; khi M Mo, ct tuyn MoM dn ti tip tuyn MoT vi L nn: Argf (zo ) = Ou, Po Ox, MoT (13) hay: o o o T suy ra Argf(zo) l gc m ta cn quay tip tuyn MoT vi ng cong L ti Mo c hng ca tip tuyn Po vi ng cong ti Po. By gi ta xt hai ng cong bt k L v L i qua Mo, ln lt c tip tuyn ti Mo l MoT v MoT. Gi v l nh ca L v Lqua php bin hnh w = f(z). v ln lt c tip tuyn ti Po l Po v Po. Theo kt qu trn: Argf ( z o ) = Ou , Po Ox , M o T Do (13) c thit lp vi L v bt k nn: Argf (z o ) = Ou, Po ' Ox, M o T' T suy ra: Ou, Po Ou, Po = Ox, MoT + Ox, Mo T Vy gc gia hai ng cong L v L bng gc gia hai nh v c v ln v hng. Ta ni php bin hnh w = f(z) bo ton gc gia hai ng cong hay php bin hnh w = f(z) l bo gic. b. ngha ca | f(zo) |: Do (12) ta c: lim Po P w wo w wo PPo = lim = f (z 0 ) = lim z z o z z z z o z z lim M o M o o

) ( ) (Ou, P ) = Argf (z ) + (Ox, M T)

) (

) (

) (

) (

) (

M M o

Vi z = z - zo kh nh th w cng kh nh v ta c: PP f (z 0 ) o MoM hay: Po P f (z 0 ) .M o M (15) Nu f (z o ) > 1 th PoP > MoM v ta c mt php bin hnh dn. Nu f (z o ) < 1 th PoP < MoM v ta c mt php bin hnh co. Cng thc (15) ng vi mi cp M v P nn ta ni f (z o ) l h s co dn ca php bin hnh ti zo. Trn y ta gi thit f(zo) 0. Nu f(zo) = 0 th kt qu trn khng ng na. V d: Xt hm w = z2. Qua php bin hnh ny, na trc dng Ox (argz = 0), c nh l na trc dng Ou(argw = 0). Na trc Oy dng arg z = c nh l na trc Ou m (argw = ). 2

19

Nh vy gc gia hai tia Ox v Oy khng c bo ton qua php bin hnh. S d nh vy v w(0) = 0.
6. Hm gii tch: a. nh ngha 1: Gi s G l mt min m. Nu hm w = f(z) c o hm f(z) ti mi im thuc G th n c gi l gii tch trong min G. Hm s w = f(z) c gi l gii tch ti im z nu n gii tch trong mt min ln cn no ca z. Trn kia ta ch nh ngha hm s gii tch trong mt min m. Gi s min G gii hn bi ng cong kn L. Nu hm w = f(z) gii tch trong mt min m cha G , th cho gn ta ni n gii tch trong min kn G . b. nh ngha 2: Nhng im ti w = f(z) khng gii tch, c gi l cc im bt thng ca hm s . V d:- Hm w = z2 gii tch trong ton C - Hm w = excosy + j exsiny gii tch trong ton C - Hm w = z khng gii tch z C 1 - w = gii tch trong ton C tr z = 0. im z = 0 l im bt thng duy z nht ca hm - Hm w = zRez ch tho mn iu kin C - R ti z = 0. Vy n khng gii tch trong ton C. c. Tnh cht ca hm gii tch: - Tng, tch ca hai hm gii tch l mt hm gii tch - Thng ca hai hm gii tch l mt hm gii tch tr im lm cho mu s trit tiu. - Hp ca hai hm gii tch l mt hm gii tch. - Hm ngc ca mt hm gii tch n dip c o hm khc khng l mt hm gii tch n dip. V d: - w = z2 + z l mt hm gii tch trong ton C v n l tng ca hai hm gii tch trong C

- w=

z gii tch ti mi im tr z = j z +1
2

7. Quan h gia hm gii tch v hm iu ho: Cho hm w = f (z) = u ( x, y) + jv( x, y) gii tch trong min n lin G. Phn thc u(x, y) v phn o v(x, y) l nhng hm iu ho trong G, ngha l chng tho mn phng trnh Laplace: 2u 2u 2v 2v u = 2 + 2 = 0 v = 2 + 2 = 0 ( x , y) G x y x y Tht vy, theo gi thit, iu kin C - R tho mn, tc l: u x = vy u y = vx

Ly o hm hai v ca ng thc th nht theo x v o hm hai v ng thc th hai theo y ta c:


20

= vyx u xx

= vxy u y

2u 2u Cng hai ng thc ta c: u = 2 + 2 = 0 x y 2v 2v Tng t ta chng minh c: v = 2 + 2 = 0 x y Ngc li, cho trc hai hm iu ho bt k u(x, y) v v(x, y) th ni chung, hm w = u(x, y )+ jv(x, y) khng gii tch. Mun w = u + jv gii tch th u v v phi l hai hm iu ho lin hp, ngha l tho mn iu kin C - R. V cho trc mt hm iu ho, ta c th tm c hm iu ho lin hp vi n nn cho trc phn thc hay phn o ca mt hm gii tch ta tm c hm gii tch . Phng php tm hm v(x, y) iu ho lin hp vi u(x, y) cho trc trong mt min n lin G nh sau: Do iu kin C - R ta bit c cc o hm ring ca v(x, y) l: vx = u y vy = u x Vy bi ton c a v tm hm v(x, y) bit rng trong min n lin G n c vi phn : dv = vx dx + vy dy = u y dx + u x dy y Bi ton ny c ngha v v phi l vi phn ton phn. Tht vy, nu t P = uy v Q = u x th M(x,y) A Q P + u yy = 0 c tho iu kin = u xx x y yo Mo mn. Theo kt qu gii tch th:

v( x , y) =

( x ,y )

( x o , yo )

uy dx + ux dy + C

(16)

x0

Trong tch phn (khng ph thuc ng i) c ly dc theo ng bt k nm trong G, i t im (xo, yo) n im (x, y), cn C l mt hng s tu . Nu tch phn c tnh dc theo ng gp khc MoAM th: v( x , y) = u y ( x , y)dx + u x ( x , y)dy + C
xo x y yo

V d 1: Cho hm u = x - y +2x. Tm v(x,y) v f(z) y l mt hm iu ho trong ton mt phng v u = 0 (x,y). Theo (16) ta chn xo = yo = 0

v( x , y) = 2 ydx + 2dy + C = 2 xy + 2 y + C Vy: f(z) = u + jv = x - y2 +2x + j(2xy + 2y + C) = (x2 + 2jxy - y2) + (2x + 2jy) + jC = (x + jy)2 + 2(x + jy) = jC = z2 + 2z + jC f(z) l mt hm gii tch trong ton C. 1 V d 2: Cho hm u ( x , y) = ln( x 2 + y 2 ) . Tm f(z) 2
21
0

y l mt hm iu ho trong ton b min G tr im gc to . Dng (16) ta xc nh c hm iu ho lin hp: v(x,y) = Arg(x + jy) + C V Argz xc nh sai khc 2k, nn v(x, y) l mt hm a tr.

22

CHNG 2: PHP BIN HNH BO GIC V CC HM S CP C BN


1. KHI NIM V BIN HNH BO GIC 1. Php bin hnh bo gic: a. nh ngha: Mt php bin hnh c gi l bo gic ti z nu n c cc tnh cht: - Bo ton gc gia hai ng cong bt k i qua im z (k c ln v hng) - C h s co dn khng i ti im , ngha l mi ng cong i qua z u c h s co dn nh nhau qua php bin hnh. Nu php bin hnh l bo gic ti mi im ca min G th n c gi l bo gic trong min G. b. Php bin hnh thc hin bi hm gii tch: Cho hm w = f(z) n dip, gii tch trong min G. Do ngha hnh hc ca f(z) ta thy rng php bin hnh c thc hin bi hm w = f(z) l bo gic ti mi im m f(z) 0. Nu ch xt trong mt ln cn nh ca im z, th php bin hnh bo gic l mt php ng dng do tnh cht bo ton gc. Cc gc tng ng trong hai hnh l bng nhau. Mt khc nu xem h s co dn l khng i th t s gia hai cnh tng ng l khng i. Ngc li ngi ta chng minh c rng php bin hnh w = f(z) n dip l bo gic trong min G th hm w = f(z) gii tch trong G v c o hm f(z) 0. 2. B Schwarz: Gi s hm f(z) gii tch trong hnh trn | z | < R v f(0) = 0. Nu | z) | M vi mi z m | z | < R th ta c: M f (z) z , |z |< R R Me j z , thc. Trong ng thc xy ra ti z1 vi 0 < | z | < R ch khi f (z) = R 3. Nguyn l i xng: Trc ht ta tha nhn mt tnh cht c bit ca hm bin phc m hm bin s thc khng c, l tnh duy nht, c pht biu nh sau: Gi s hai hm f(z) v g(z) cng gii tch trong min D v tho mn f(z) = g(z) trn mt cung L no nm trong D, khi f(z) = g(z) trn ton min D. Gi s D1 v D2 nm k nhau v c bin chung l L
y v D1 L D2 x O

z O

w B1 T B2

23

Gi s f1(z) gii tch trong D1 v f2(z) gii tch trong D2. Nu f1(z) = f2(z) trn L th ta gi f2(z) l thc trin gii tch ca f1(z) qua L sang min D2. Theo tnh duy nht ca hm gii tch nu f3(z) cng l thc trin gii tch ca f1(z) qua L sang min D2 th ta phi c f3(z) = f2(z) trong D2. Cch nhanh nht tm thc trin gii tch ca mt hm cho trc l p dng nguyn l i xng sau y: Gi s bin ca min D1 cha mt on thng L v f1(z) bin bo gic D1 ln B1 trong L chuyn thnh on thng T thuc bin ca B1. Khi tn ti thc trin gii tch f2(z) ca f1(z) qua L sang min D2 nm i xng vi D1 i vi L. Hm f2(z) bin bo gic D2 ln B2nm i xng vi B1 i vi T v hm: f1 (z) trong D1 f (z) = f1 (z) = f 2 (z) L f (z) trong D 2 2 bin bo gic D thnh B. Nguyn l i xng thng dng tm php bin hnh bo gic hai min i xng cho trc. 2. CC PHP BIN HNH QUA CC HM S CP 1. Php bin hnh tuyn tnh: Xt hm tuyn tnh w = az + b trong a, b l cc hng s phc. Gi thit a 0. Nu a = | a |ej th w = | a |ejz + b. Php bin hnh tuyn tnh l bo gic trong ton mt phng phc v f(z) = a 0 z C. Hm tuyn tnh c th coi l hp ca 3 hm sau: - = kz (k = | a | > 0) - = ej. ( = Arga) y -w=+b w Nu biu din cc im , , w trong cng mt mt phng th da vo ngha hnh hc ca php nhn v php cng cc s phc ta suy ra rng: - im nhn c t im z bng php co dn z vi h s k x O - im nhn c t im bng php quay tm O, gc quay . - im w nhn c t im bng php tnh tin xc nh bi vec t biu din s phc b. Nh vy mun c nh w ca z ta phi thc hin lin tip mt php co dn, mt php quay v mt php tnh tin. Tch ca 3 php bin hnh trn l mt php ng dng. Vy php bin hnh tuyn tnh l mt php ng dng. N bin mt hnh bt k thnh mt hnh ng dng vi hnh y. c bit, nh ca mt ng trn l mt ng trn, nh ca mt ng thng l mt ng thng. V d: Tm hm w = f(z) bin hnh tam gic vung cn A(3+ 2j), B(7 + 2j), C(5 + 4j) thnh tam gic vung cn c nh ti O1, B1(-2j) v C1(1 - j)

24

y C O1 2 A O 3 B 7 x

y x C1 B1

V cc tam gic ABC v O1B1C1 ng dng nn php bin hnh c thc hin bng mt hm bc nht w = az + b. Php bin hnh ny c th phn tch thnh cc php bin hnh lin tip sau y: * php tnh tin t A v gc, xc nh bng vec t (-3 - 2j). Php tnh tin ny c xc nh bi hm = z - (3 + 2j) j * php quay quanh gc mt gc , ng vi hm = e 2 2 OB 2 1 * php co dn tm O, h s k = 1 1 = = , c thc hin bng hm AB 4 2 1 w= 2 3 j 1 j2 Vy: w = e (z 3 2 j) = (z 3 2 j) = jz + j 1 2 2 2 2. Php nghch o: a. nh ngha: Hai im A v B c gi l i xng i vi ng trn C tm O, bn knh R nu chng cng nm trn mt na ng thng xut pht t O v tho mn ng thc: OA.OB = R2 R2 R R > 1 th OB > R. Ngc li = .R nn nu OA < R D nhin, v OB = OA OA OA nu OA > R th OB < R. Ngha l trong hai im A v B th mt im nm trong v mt im nm ngoi ng trn. Nu A nm trong ng trn th mun c B k ng AH OA v sau v tip tuyn HB.
H A O B O B A H

25

Nu A nm ngoi ng trn th mun c im B ta v tip tuyn AH, sau k HB OA. b. nh l 1: Nu A v B i xng vi ng trn C v C l ng trn bt k i qua A v B th C v C trc giao vi nhau. Chng minh: Gi I l tm v r l bn knh ca C. K hiu PCO l phng tch ca im O i vi ng trn C. Theo gi thit v A v B i xng qua C nn D OA.OB = R2. Mt khc theo cch tnh phng tch ta c: B O A PCO = OA.OB = OI2 - r2 C I T suy ra: 2 2 2 R = OI - r hay: OI2 = R2 + r2 = OD2 + ID2. C Vy OD DI c. nh l 2: Gi s hai ng trn C v C cng trc giao vi ng trn C. Nu C v C ct nhau ti A v B th hai im A v B i xng qua C Chng minh: Gi I1 v I2 ln lt l tm ca ng trn C v C; r1 v r2 l bn knh ca C chng. Gi R l bn knh ca ng trn C. Ta c: 2 PCO = OI1 r12 B O A 2 2 PCO = OI 2 r2 C Nhng do gi thit trc giao ta c: 2 OI1 r12 = R2 2 2 C OI 2 2 r2 = R Vy: PCO = PCO V im O c cng phng tch vi c hai ng trn C v C nn O nm trn trc ng phng AB ca cp vng trn . Mt khc do PCO = OA.OB = R2 nn A v B i xng qua C. 1 d. Php bin hnh w = : Php bin hnh ny n 1 z w= z dip, bin mt phng phc m rng z (tc mt phng z phc c b sung thm im z = ) ln mt phng phc m rng w. nh ca im z = 0 l im w = . Ngc li O 1 nh ca im z = l im w = 0. V w = 2 nn z z php bin hnh bo gic ti z 0 v z .

26

1 = w i z 1 1 xng nhau qua ng trn n v v Arg = Argz = Argz . Mt khc z . = 1. z z Vy mun c w, ta dng w i xng vi z qua ng trn n v ri ly i xng 1 qua trc thc. Ni khc i, php bin hnh w = l tch ca hai php i xng: z * php i xng qua ng trn n v * php i xng qua trc thc 1 e. Tnh cht ca php bin hnh: )Php bin hnh w = bin: z * mt ng trn i qua gc to thnh mt ng thng * mt ng trn khng i qua gc to thnh mt ng trn * mt ng thng i qua gc to thnh mt ng thng * mt ng thng khng i qua gc to thnh mt ng trn i qua gc to . Nu coi ng thng l mt ng trn c bn knh v hn th tnh cht trn 1 c pht biu gn li l: Php bin hnh w = bin mt ng trn thnh mt z ng trn. Chng minh: Xt ng cong C c phng trnh: A(x2 + y2) + 2Bx + 2Cy + D = 0 Trong A, B, C, D l nhng hng s thc. Vit phng trnh y di dng phc ta c: Azz + Ez + Ez + D = 0 (1) Trong E = B - jC Nu A 0, D = 0 th C l ng trn i qua gc to . Nu A = 0 th C l ng thng. Nu A = D = 0 th C l ng thng i qua gc to . nh ca C qua php 1 bin hnh w = l ng cong L c phng trnh: z 1 1 E E A . + + +D=0 w w w w (2) hay: Dww + Ew + Ew + A = 0 Nu D = 0 th L l ng thng. Nu D = A = 0 th L l ng thng i qua gc to . Nu A = 0 th L l ng trn i qua gc to . ) Gi s z1 v z2 l hai im i xng vi nhau qua ng trn C. Khi nu 1 gi w1 v w2 v L l nh ca z1, z2 v C qua php bin hnh w = th w1 v w2 i z 1 xng nhau qua C. Ni khc i, php bin hnh w = bo ton tnh i xng qua mt z ng trn.

Ta s nu ra cch tm nh ca mt im z bt k. Ch l hai im z v

27

Chng minh: Ly 2 ng trn bt k P v Q qua z1 v z2.Theo nh l 1 th P v Q cng trc giao vi C. Qua php bin hnh, P v Q s bin thnh hai ng trn L1 v L2 ct nhau ti w1 v w2. V php bin hnh bo gic nn L1 v L2 trc giao vi C. Theo nh l 2 th w1 v w2 s i xng vi nhau qua L. 1 V d 1: Tm nh ca hnh trn | z | < 1 qua php bin hnh w = z D dng thy rng nh ca ng trn | z | = a (0 < a < 1) l ng trn 1 1 w = . Khi a bin thin t 0 n 1, th gim t + n 1. Trong khi ng trn | a a z | = a qut nn hnh trn | z | < 1 th nh ca n qut nn min | w | > 1. Tm li nh ca min | z | < 1 l mim | w | > 1. nh ca ng trn | z | = 1 l ng trn | w | + 1. V d 2: Tm nh ca bn kinh OB: argz = /6; | z | < 1 qua php bin hnh w = 1/z

y M O B x O

x B N

Ly M bt k trn OB. Thc hin lin tip php i xng qua ng trn n v v php i xng qua trc thc ta c nh N ca n nm trn na ng thng sao cho: OM.ON = 1 Khi M chy t O n B, N chy t n B. az + b : Php bin hnh ch c ngha khi c cz + d v d khng ng thi trit tiu. Ta khng xt trng hp ad = bc v y l trng hp tm thng . Tht vy nu ad = bc th ta c th vit: az + b adz + bd b b w= . = = cz + d cbz + db d d d b Tc l mi z u c cng mt nh w = . c d Vy ta ch xt cc trng hp ad - bc 0. Nu c = 0 ta c hm tuyn tnh xt: a b w = z+ d d az + b cho nn ta gi thit c 0. Php bin hnh w = l n dip v bin ton b mt cz + d
3. Php bin hnh phn tuyn tnh w =
28

d c nh l im c az + b dw + b w= . Ngc li, gii z theo w, ta c hm ngc z = ; tc l mi cz + d cw a a d dw + b im w c nghch nh l z = . nh ca im z = l im w = . c cw a c a nh ca im z = l w = c ad bc nn php bin hnh phn tuyn tnh bo gic ti mi im V w = (cz + d ) 2 d z v z . Phn tch biu thc ca w ta c: c az + b acz + bc acz + ad + bc ad a (cz + d) + bc ad = = = w= cz + d c(cz + d) c(cz + d) c(cz + d) a bc ad 1 . = + c c cz + d T suy ra php bin hnh phn tuyn tnh l tch ca 3 php bin hnh: = cz + d php bin hnh tuyn tnh 1 php nghch o = bc ad a w= . + php bin hnh tuyn tnh c c V mi php bin hnh thnh phn u bin mt ng trn thnh mt ng trn v bo ton tnh i xng ca 2 im i vi ng trn nn php bin hnh phn tuyn tnh cng c cc tnh cht y. Php bin hnh phn tuyn tnh tng qut cha 4 tham s a, b, c, d nhng thc cht ch c 3 tham s l c lp. Tht vy, vi gi thit c 0, ta c: a b z+ c w= c d z+ c a b d Nu ta t a 1 = , b1 = , d1 = th ta c: c c c a z + b1 w= 1 z + d1 Vy mun php bin hnh phn tuyn tnh hon ton xc nh, ta phi cho 3 iu kin. Chng hn ta c th buc n bin 3 im cho trc z1, z2 v z3 ln lt thnh 3 im w1, w2 v w3. Khi cc tham s a1, b1 v d1 l nghim ca h: phng m rng z ln mt phng m rng w. Mi im z
29

a 1z1 + b1 z + d = w1 1 a 1z 2 + b1 = w2 + z d 1 a 1z 3 + b1 = w3 + z d 1 Gii h ny ta tnh c a1, b1 v d1 ri thay vo w = di dng i xng:


w w 2 w1 w 3 z z 2 z1 z 3 = . . w w 3 w1 w 2 z z 3 z1 z 2

a1z + b1 ta c hm phi tm z + d1
(4)

V d 1: Tm php bin hnh bo gic bin na mt phng trn ln hnh trn n v sao cho z = a vi Ima > 0 thnh w = 0 Theo tnh bo ton v tr im i xng th im z = a phi chuyn thnh im w=. Vy php bin hnh phi tm c dng: za w=k za V z = 0 chuyn thnh mt im no trn ng trn | w | = 1 nn suy ra | k | = 1 hay k = ej. Vy: za w = e j za V d 2: Bin hnh trn n v thnh chnh n sao cho z = a vi | a | < 1 thnh w = 0. 1 Theo tnh bo ton v tr i xng th im b = nm i xng vi a qua ng trn a | z | = 1phi chuyn thnh im w = . Php bin hnh cn tm c dng: za za w=k =K 1 az zb Trong k v K l cc hng s no . V z = 1 th | w | = 1 nn ta c: 1 a K =| K |= 1 nn K = ei 1 a za v: w = e j 1 az V d 3: Bin na mt phng trn thnh chnh n Php bin hnh ny c thc hin bng hm phn tuyn tnh bin 3 im z1, z2 v z3 trn trc thc theo chiu dng ca mt phng z thnh 3 im w1, w2, w3 trn trc thc theo chiu dng ca mt phng w.

30

1 1 4. Php bin hnh Giucovski: Ta gi hm phc w = z + l hm Giucovski. 2 z hm ny c rt nhiu ng dng trong k thut. N c mt im bt thng hu hn l 1 1 z = 0. o hm ca n l w = 1 2 , w = 0 ti cc im z = 1. Vy php bin 2 z hnh Giucovski bo gic ti mi im z hu hn khc vi im O v 1. Ta hy tm min n dip ca hm. Gi s z1 z2 nhng: 1 1 1 1 1 = z1 + z2 + hay (z1 z 2 ) 1 (5) =0 2 z1 2 z2 z z 1 2 Ta thy rng ng thc (5) xy ra khi z1.z2 = 1. Vy php bin hnh s n dip trong mi min khng cha hai im nghch o ca nhau. Chng hn min | z | < 1 l min n dip ca hm s; min | z | > 1 cng l mt min n dip khc. V d 1: Tm nh ca php bin hnh Giucovski ca: * ng trn | z | = h 0<h<1 * on thng Argz = , | z | < 1 * hnh trn n v | z | < 1 * na mt phng trn, nm ngoi hnh trn n v tm O. Ta t z = rej. Hm Giucovski c vit thnh: 1 1 1 1 w = u + jv = re j + j = r (cos + j sin ) + (cos j sin ) r re 2 2 Tch phn thc v phn o ta c: 1 1 u = r + cos 2 2 1 1 v = r sin 2 2 T suy ra nh ca ng trn | z | = r = h c phng trnh tham s l: 1 1 u = 2 h + h cos 11 1 v = 1 h sin = h sin 2h h 2 1 1 Trong l tham s. l mt elip (), c tm O v cc bn trc a = h + v 2 h

11 1 1 1 1 b = h , tiu c 2c = a 2 b 2 = 2 h + h = 2 . Cc tiu im h h 4 4 2h ca elip l F1(-1, 0) v F2(1, 0). Khi bin thin t 0 n 2, im z chy dc ng trn | z | = h theo hng dng trong khi nh w tng ng ca n chy trn ellip theo hng m ca mt phng.
31

V khi 0 < < th v < 0 v khi < < 2 th v > 0 nn nh ca na ng trn trn l na elip di, nh ca na ng trn di l elip trn. Ch l khi h 0 th cc bn trc a, b ca elip dn ra , ngha l nu ng trn | z | = h cng nh th nh ca n c cc bn trc cng ln. Khi h 1th a 1 v b 0, ngha l nu ng trn | z | = h cng dn vo ng trn n v th elip nh dt dn v tin ti on kp F1F2 (s d gi l on kp v F1F2 ng thi l nh ca na cung trn n v trn v na cung trn n v di). Ta quy c b trn ca on l nh ca na cung trn n v nm trong na mt phng di; b di ca on thng l nh ca na cung trn n v nm trong na mt phng trn. Nu gi L l nh ca on thng: Argz = | z |< 1 th phng trnh tham s ca L l: 1 1 = u r + cos 2 r 1 v = 1 r sin 2r Kh r trong cc phng trnh ny ta c: u2 v2 =1 (6) cos 2 sin 2 y l mt hyperbol c cc tiu im trng vi F1 v F2. y v

F1 x O1

F2 u

th nh (L) l nhnh hyperbol (6) nm trong gc phn t th t. Khi 2 im z chy trn on bn knh t gc to ti ng trn n v th nh w ca n chy trn nhnh hyperbol nm trong gc phn t th t t ti trc thc O1u. Khi cho h bin thin t 0 n 1 th ng trn | z | = h s qut nn hnh trn | z | < 1. nh () ca L trong mt phng w s qut nn mt phng w, b i lt ct dc on F1F2. B di ca lt ct l nh ca cung trn n v trn. B trn ca lt ct l nh ca cung trn n v di. Na hnh trn n v trn c nh l na mt phng di. Ngc li na hnh trn n v di c nh l na mt phng trn.

Nu 0 < <

32

Tng t nh cu u tin nh ca na ng trn trn: r = h (h > 1) 0 < < c phng trnh tham s l: 1 1 u = 2 h + h cos 0<< 1 1 v = h sin 2 h

1 1 y l mt cung ellip nm trong na mt phng trn , c cc bn trc l a = h + 2 h 1 1 v b = h h 2 Khi na ng trn trn tm O, bn knh h qut nn phn na mt phng trn nm ngoi ng trn n v th nh ca n qut nn na mt phng trn Imz > 0 xem hnh v). v y

-1

-1

O1

V d 2: Tm php bin hnh bin na hnh n v | z | = 1, Imz > 0 thnh na mt phng trn. D thy rng php bin hnh phi tm l hp ca hai php: t = z = e j z
1 1 w = t + 2 t

5. Hm lu tha w = zn: Ta xt hm w = zn vi n nguyn dng, ln hn hay bng 2. Nu z = r(cos + jsin) th w = rn(cosn + jsinn). Vy nh ca tia Argz = l tia Argw = n nhn c bng cch quay tia Argz = quanh gc to gc (n - 1). nh ca ng trn | z | = R l ng trn | w | = Rn. nh ca mt phng z l mt phng w. Tuy nhin php bin hnh t mt phng z ln mt phng w khng n dip v nu hai 2 s phc z1 v z2 c cng mun v c argumen sai khc nhau mt s nguyn ln n n n th z1 = z 2 .
33

Mun hm w = zn n dip trong mt min G no th min G ny phi khng cha 2 . Chng hn bt k cp im no c cng mun v c argumen sai khc nhau gc n 2 l mt min n dip ca hm w = zn. nh ca min qut min qut 0 < arg z < n ny, qua php bin hnh, l mt phng w, b i mt lt ct dc theo na trc thc u > 0 . B trn ca lt ct l nh ca tia argz = 0 v b di ca lt ct l nh ca tia 2 . arg z = n 3 cng l mt min n dip khc ca hm. nh ca Min qut < arg z < n n min qut ny qua php bin hnh l mt phng w, b i mt lt ct dc theo na trc thc m. Hm w = zn gii tch trong ton mt phng, v ta c: dw = nz n 1 z C dz Php bin hnh w = zn bo gic ti mi im z 0.
6. Hm w = n z : y l hm ngc ca hm w = zn. N l mt hm a tr v vi mi s phc z = r(cos + jsin) 0 c n cn bc n cho bi: + 2k + 2k k = 0,1,K, n 1 w = n r cos + j sin n n To v ca n s phc ny l cc nh ca mt a gic u n cnh tm O. Gi z im z vch thnh mt ng cong kn L khng bao quanh gc to O, xut pht t zo. y y

L C zo O x 2 1

w1 o O w2 wo x

Khi im w = n z trong n z l mt gi tr no ca cn thc m ta chn trc s vch nn ng cong kn o, xut pht t w o = n z o v khi z xut pht t zo chy mt vng trn C th Argz bin thin t gi tr ban u Argzo ri quay v ng gi tr y. Cc gi tr cn thc khc vi gi tr chn s vch nn ng cong kn k, c suy ra t o bng cch quay cc gc 2/n quanh gc to .
34

By gi ta gi thit im z vch nn ng cong kn C bao quanh gc to mt vng theo hng dng, xut pht t im zo. Trong trng hp ny, khi z chy mt vng th arumen ca z tng thm 2. Do vy argumen ca w tng thm 2/n. im w s vch nn mt ng cong lin tc t im wo ti 2 2 w 1 = w o cos + j sin . Ngha l w i t gi tr wo ca cn thc ti mt gi tr n n khc ca cn thc. Do im w ch tr v v tr xut pht sau khi z chy n vng trn C. iu chng t rng mun tch c mt hm n tr lin tc t hm a tr w = n z th min xc nh E ca hm n tr ny khng c cha bt k mt ng cong kn no bao quanh gc O. Mun vy ta c th ly E l mt phng phc z ct di mt lt ct t gc to ra . Chng hn, c th chn l na trc Ox dng. Khi cc hm n tr tch ra t hm a tr w = n z , m ta thng gi l cc nhnh n tr cu hm w = n z l nhng hm bin phc bin E(mt phng phc vi lt ct dc theo na trc Ox dng) ln mi hnh qut: 2 0 < arg z < n 4 2 < arg z < n n LL Mun chn ra mt nhnh xc nh trong n nhnh trn ta c th buc nhnh ny phi ly mt gi tr wo khi z = zo vi wo l cn bc n no ca zo. Mi nhnh n tr ca hm w = n z trong min xc nh E c o hm: 1 1 1 1 n 1 n ( z ) = n = = z ( w ) nw n 1 n nn n l hm gii tch trong E. Nu ta khng dng lt ct th khng th tch c cc nhnh n tr v khi im z vch nn ng cong kn th im w s chuyn t nhnh n sang nhnh kia. V vy O cn c gi l im r nhnh ca hm a tr w = n z . V d: Xt hm a tr w = 3 z 2 4 . Nhng nhnh n tr ca ca hm ; Ot2 l tia Argw = Gi Ot1 l tia Argw = 3 3 w = 3 z l cc php bin hnh n dip, bin mt phng phc z, b i lt ct dc theo na trc Ox dng ln mi gc uOt1, t1Ot2, t2Ou. Nhnh w = 3 z = 3 r (cos + j sin ) = 3 r cos + j sin vi 0 < < 2 bin 3 3 hai im A v B nm ln lt b trn v b di ca lt ct thnh hai im A thuc 2 . iu chng t na trc Ox l ng gin tia argw = 0 v B thuc tia arg w = 3 on ca nhnh ny.
35

t1

A O B x

B O

t2
7. Hm m: a. nh ngha: Ta gi hm phc c phn thc u(x,y) = excosy v phn o v(x,y)=exsiny l hm m bin phc v k hiu l ez. w = ez = ex + jy = ex(cosy + jsiny) (1) x x Cho y = 0 ta c w = e , ngha l khi z = x thc ta c hm bin thc e bit. Ta ni rng hm m w = ez l thc trin ca hm m thc ex t trc thc ra ton b mt phng phc. Theo nh ngha trn ta c: (2) | w | = ex v Argw = y + 2k, k nguyn b. Cc php tnh v hm m: e z1 .e z2 = e z1+z2 e z1 = e z1z2 (3) z1 e (e z ) n = e nz , n nguyn Ta chng minh cng thc u tin. Cc cng thc sau cng tng t. Ta c: z1 = x1 + jy1 ; z2 = x2 + jy2 Theo nh ngha ta c: e z1 = e x1 (cos y1 + jsin y1 ) v e z 2 = ex2 (cos y 2 + j sin y 2 )

Vy: e z1 .e z2 = e x1 (cos y1 + j sin y1 ) e x 2 (cos y 2 + j sin y 2 ) Hay: e z1 .e z 2 = e x1 + x 2 [cos( y1 + y 2 ) + jsin( y1 + y 2 )] Theo nh ngha hm m phc ta c: e z1 .e z2 = e( x1+ x2 )+ j( y1+ y2 ) = e z1+z2 c. Chu k ca hm m: Theo inh ngha, ta c: e2jk = cos2k + jsin2k = 1 ( k nguyn) Theo (3) th: (4) e2jk+z = ez. e2jk = ez z Cng thc ny cho thy rng hm w = e l hm tun hon vi chu k 2j. Vy hai im nm trn mt ng song song vi trc o v cc nhau mt khong bng bi s ca 2j th c cng nh. Cn ch l nu e z1 = e z2 th:
36

e z1 = e z2 = z 2 = z1 + 2 jk

(5)

e z1 = e z1z2 = 1 = e 2 jk v z1 - z2 = 2jk v: z1 e d. Cng thc Euler: Trong (1), cho x = 0 ta c cng thc Euler: e jy = cos y + jsin y (6) Thay y bng -y ta c: e jy = cos y jsin y (7) Nh c cng thc Euler m s phc z = r(cos + jsin) vit c di dng m z = z = r(cos + jsin) = rej rej. Ta c: V d: 1 = cos0 + jsin0 = ej0 j j = cos + j sin = e 2 2 2 j 1 + j = 2 cos + j sin = 2e 4 4 4 4 jarctg 4 4 3 + 4 j = 5cos arctg + j sin arctg = 5e 3 3 3 e2+3j = e2(cos3 + jsin3) e-2j = cos2 - jsin2 f. Tnh gii tch ca hm w = ez: Hm w = ez gii tch trong ton b mt phng v z, iu kin C - R c tho mn: x (e cos y ) = (e x sin y ) y x x (e cos y) = (e x sin y ) x y x (e cos y ) + j (e x sin y ) w (z) = x x g. Php bin hnh w = ez: V | w | = ex nn nh ca ng thng x = C1 l ng trn w = e C1 . V y l mt gi tr ca Argw, nn ng thng y = C2 c nh l tia Argw= C2. Khi C2 bin thin t 0 n 2 (0 < C2 < 2) th ng y = C2 s qut nn min G l bng 0 < y < 2. nh ca ng thng y = C2 l tia Argw = C2 s qut nn min l nh ca G. R rng l mt phng w, b i lt ct dc theo na trc thc u dng; b trn ca lt ct ny ng vi ng y = 0, b di ca lt ct l nh ca ng y = 2. Php bin hnh t bng G ln min l mt php bin hnh n dip. Tng t, php bin hnh w = ez cng bin mi bng 2k < y < 2(k+1)( k nguyn), c chiu rng k, ln min ni trn. Php bin hnh w = ez bin c mt phng z ln mt phng w, nhng khng n dip.
37

Tht vy, nghch nh ca mi im w 0 gm v s im, v nu z thuc nghch nh ca w , tc l ez = w th cc im z = 2jk cng thuc nghch nh ca w v ez+2jk = ez. y v 2 C2 O x O C2 u

8. Hm loga: a. nh ngha: hm ngc ca hm z = ew c gi l hm loga v k hiu l: w = Lnz b. Phn thc v phn o ca hm w = Lnz: t w = Lnz = u+ jv, th theo nh ta c: eu+jv = z Vy eu = | z | hay u = ln| z | v v = Argz. Tm li: w = Lnz = ln| z | + jArgz (9) hay: w = ln| z | + j(argz + 2k) (10) Hm w = Lnz l mt hm a tr. Vi mi gi tr ca z c v s gi tr ca w. Cc gi tr ny c phn thc bng nhau cn phn o hn km nhau mt bi s nguyn ca 2. nh ca im z l nhng im w nm trn ng thng song song vi trc o v cch nhau mt on c di bng bi s nguyn ca 2. b. Tch nhnh n tr: tch mt nhnh n tr ca hm w = Lnz, ta lm nh sau. Trong cng thc (10) ta gi s k = k1 l mt s nguyn c nh. Khi ta c mt nhnh n tr ca hm loga v k hiu l (w)1. Nhnh ny bin min - < argz < ca mt phng z (tc l mt phng z vi lt ct dc theo na trc x < 0) ln bng (2k11) < Imz < (2k1+1) ca mt phng w. Nu khng v mt lt ct i t im z = 0 ra , th khi im z vch nn mt ng cong kn quanh gc O theo hng dng, argumen ca z s tng thm 2, v nh vy ta s i t nhnh n tr ny sang nhnh n tr khc. Vy im O cng l mt im r nhnh ca hm a tr w = Lnz. c bit, nu trong (10) ta chn k = 0 th s c mt nhnh n tr c gi l nhnh chnh ca hm a tr w = Lnz. Nhnh ny c k hiu l lnz: lnz = ln| z | + jargz (11) Nu z l s thc dng z = x > 0 th argz = 0, | z | = x nn lnz = lnx, ngha l gi tr chnh ca hm loga trng vi hm bin thc lnx. Ni khc i, lnz l thc trin ca hm thc lnx , t trc thc x >0 ra mt phng phc z. V d: Tnh Ln(-1); ln(-1) ; ln(1 + j) ; Lnj * Ln(-1) = ln| -1 | + j[arg(-1) + 2k] = j( + 2k)= j(2k + 1) * ln(-1) = ln| -1 | + jarg(-1) = j
38

1 nn ln(1 + j) = ln 2 + j = ln2 + j 4 4 2 4 * V | j | = 1 ; argj = nn Ln = j + 2k 2 2 d. Tnh cht gii tch: Nhnh n tr w = lnz l mt hm gii tch trong mt phng phc, b i lt ct dc theo na trc x < 0. Theo cng thc tnh o hm ca hm ngc ta c: 1 1 1 (ln z) = w = w = (e ) e z e. Cc php tnh: Hm Lnz c cc tnh cht: Ln (z1.z 2 ) = Lnz1 + Lnz 2

* V | 1 + j | =

2 ; arg(1 + j) =

Ln

z1 Lnz1 Lnz 2 z2

(12)

Ln (z n ) = nLnz + 2 jk Ta chng minh, chng hn, cng thc u: Ln(z1.z 2 ) = ln z1.z 2 + jArg(z1.z 2 ) = ln z1 + ln z 2 + j Argz1 + Argz 2

= (ln z1 + jArgz1 ) + (ln z 2 + jArgz 2 ) = Lnz1 + Lnz 2

9. Hm lng gic: a. nh ngha: T cng thc Euler ta c: e jy + e jy 2 cos y = e jy + e jy cos y = 2 jy e e jy jy jy 2 sin y = e e sin y = 2j Cc hm lng gic bin s phc c nh ngha nh sau: e jz + e jz e jz e jz cos z = sin z = 2 2j

sin z e e cos z e + e = cot gz = = jz jz cos z j(e + e ) sin z e jz e jz V ejz v e-jz l nhng hm n tr nn cc hm lng gic bin phc cng l cc hm n tr. b. o hm ca cc hm lng gic: V ejz v e-jz l nhng hm gii tch trong ton C nn cc hm lng gic bin phc w = cosz v w = sinz cng l cc hm gii tch trong ton C. Ta c: 1 jz 1 1 (sin z) = (e ) (e jz ) = je jz + je jz = e jz + e jz = cos z 2j 2j 2 Tng t ta c: (cosz) = -sinz tgz =
jz jz

jz

jz

(13)

] [

39

Hm w = tgz gii tch ti mi im c cosz 0. Xt phng trnh cosz = 0. Ta c: cos z = 0 = e jz = e jz hay: e2jz = ej. Do : 2jz = j + 2jk Phng trnh ny c nghim l: z = + k 2 Nh vy tgz gii tch ti mi im z + k . Ta d dng tnh c: 2 1 ( tgz) = cos 2 z Tng t : 1 (cot gz) = 2 sin z c. Tnh cht: Hm lng gic bin s phc c cc tnh cht sau: cos(-z) = cosz sin(-z) = -sinz tg(-z) = -tgz cos(z + 2) = cosz sin(z + 2) = sin z trong(z + ) = tgz 1 1 Tht vy: cos( z ) = [e j( z ) + e j( z ) ] = [e jz + e jz ] = cos z 2 2 1 1 cos( z + 2) = [e j( z+2 ) + e j( z+ 2 ) ] = [e jz + e jz ] = cos z 2 2 2j -2j v e =e =1 Tng t ta chng minh c cc tnh cht cn li. d. Cc php tnh: Ta c cc cng thc quen bit: sin2z + cos2z = 1 sin(z1 + z2) = sinz1cosz2 + sinz2cosz1 cos2z = cos2z - sin2z (15) z + z2 z + z2 sin z1 + sin z 2 = 2 sin 1 cos 1 2 2 Ta chng minh, chng hn, cng thc u tin: sin2z + cos2z = cos2z - j2sin2z = (cosz + jsinz)(cosz - jsinz) = ejz.e-jz = 1 V d 1: Tnh cosj Theo nh ngha: e 1 + e1 1 1 cos j = = + e 1,543 2 2e Qua v d ny ta thy c nhng s phc c | cosz | > 1. iu ny khng th xy ra i vi s thc. V d 2: Gii phng trnh sinz = sinzo vi zo l s phc cho trc. Phng trnh trn c vit thnh: sinz - sin zo = 0, hay: z zo z + zo sin z1 sin z o = 2 sin cos =0 2 2
40

Cho sin

z zo 2 z + zo 2

= 0 ta c = 0 ta c

z zo 2 z + zo 2

= k . Vy nghim ca phng trnh z = zo + 2k = + k , vy nghim ca phng trnh z = - zo + 2

Cho cos

2k Tm li nghim ca phng trnh l: z = zo + 2k v z = - zo + 2k.

10. Hm hyperbol: a. nh ngha: Cc hm hyperbol bin phc c nh ngha theo cc cng thc sau: ez + ez ez ez shz chz chz = shz = thz = coth z = (16) chz shz 2 2 Nhng hm ny l thc trin ca hm hyperbol bin thc t trc thc ra mt phng phc. D dng thy rng hm chz l hm chn cn cc hm shz, thz, cothz l cc hm l. V ez tun hon vi chu k 2j nn cc hm shz v chz cng tun hon vi chu k 2j. Hm thz tun hon vi chu k j. Tht vy: shz e z e z e 2 z 1 thz = = = (17) chz e z + e z e 2 z + 1 D dng kim tra thy th(z + j) = thz b. Cc php tnh: Ta c cc cng thc ging nh trong gii tch thc: ez = chz + shz e-z = chz - shz ch2z - sh2z = 1 (18) sh(z1 + z2) = shz1chz2 + shz2chz1 ch2z = ch2z + sh2z .... c. Quan h vi cc hm lng gic: T nh ngha ta suy ra: sinjz = jshz cosjz = chz d. Tch phn thc v phn o ca hm lng gic v hm hyperbol: Ta c: sinz = sin(x + jy) = sinxcosjy + sinjycosx = sinxchy + jshycosx Tng t: cosz = cosxchy - jsinxshy shz = shxcosy + jsinychx (20) chz = chxcosy + jsinxshy e. o hm ca hm hyperbol: Cc hm w = shz v w = chz gii tch trong ton b mt phng v c o hm: (shz) = chz (chz) = shz Hm w = thz gii tch trong ton mt phng tr ti im z m e2z + 1 = 0 hay e2z = -1 = e2, tc l:
41

z = j + k 2 1 Ta c: ( thz ) = 2 ch z V d 1: Tnh sin(1 - 2j) Ta c: sin(1 - 2j) = sin1.cos2j - sin2jcos1 = sin1.ch2 - jsh2.cos1 Theo (19) th cos2j = ch2, sin2j = sh2. Tra bng s ta c sin1 sin57o19 0,8415 cos1 0,5463 ch2 3,7622 sh2 3,6269. Kt qu l: sin(1 - 2j) = 0,84153,7622 - j0,54633,6269 = 3,1659 - 1,9595j V d 2: Cho php bin hnh w = sinz. Tm nh ca bng < x < 2 2 Trc ht ta tm nh ca ng thng x = C. Theo (20): u(x, y) = Re(sinz) = sinxchy v(x, y) = Im(sinz) = cosxshy nn phng trnh tham s ca ng thng x = C l: u ( x, y) = sin Cchy y l tham s - < y < (21) v ( x , y ) = cos Cshy Nu C = 0 th cc phng trnh (21) biu din trc o u. Nu C 0 th n biu din mt cung hyperbol. Tht vy, kh C trong (21) ta c: u2 v2 =1 (22) sin 2 C cos 2 C v cung hyperbol bn tri nu Ta c cung hyperbol bn phi nu 0 < C < 2 < C < 0 . Hyperbol (22) c tiu trc l trc thc, cc tiu im F1(w = -1) v F2(w 2 = 1), cc bn trc l | sinC | v | cosC |. Tim cn ca n l cp ng thng v = cotgCu. Cho C bin thin t n , ng thng x = C s qut bng < x < . nh 2 2 2 2 ca C trong mt phng w s qut nn min G l nh ca bng < x < . Ch l 2 2 theo (21) th nh ca ng thng x = c phng trnh tham s u = chy, v = 0 v 2 l tia F2u.Tng t ta c nh ca ng thng x = l tia F1u. Vy min G l 2 mt phng w b i hai tia F2u v F1u.
y v

42

11. Hm lng gic ngc:Hm ngc ca z = sinw c k hiu l w = Arcsinz. Ta c: e jw e jw e 2 jw 1 z = sin w = = 2j 2 je jw hay: e2jw - 2jzejw - 1 = 0 Ta xem y l phng trnh bc hai i vi ejw. Gii ra ta c: e jw = jz + 1 z 2

Vy hay:

jw = Ln jz + 1 z 2 1 w = Ln jz + 1 z 2 j

( (

) )

Nh vy: w = Arc sin z = jLn ( jz + 1 z 2 ) (23) Tnh a tr ca hm w = Arcsinz c suy ra t tnh lng tr ca cn thc v tnh a tr ca hm loga. Tng t ta nh ngha: w = Arccosz l hm ngc ca z = cosw w = Arctgz l hm ngc ca z = tgw w = Arccotgz l hm ngc ca z = cotgw Lp lun tng t trn ta c: w = Arc cos z = jLn (z + z 2 1) j 1 + jz (24) w = Arctgz = Ln 2 1 jz j z j w = Arc cot gz = Ln 2 z+ j V d 1: Tnh Arcsinj Theo (23) ta c: Arc sin j = jLn(1 2 ) Nu trc cn ly du + ta c: Arc sin j = jLn(1 + 2n ) = j ln( 2 1) + j(0 + 2k) = 2k j ln( 2 1) Nu trc cn ly du - ta c: Arc sin j = jLn(1 2 ) = j ln( 2 + 1) + j( + 2k) = 2(k + 1) j ln( 2 + 1)

43

Vit gp li ta c: Arc sin j = n j ln 2 (1) n n nguyn

V d: Tnh Arctg2j Theo (24) ta c: j 1 j 1 j 1 2 (2k + 1) ln 3 = Ln = ln + j( + 2k) = Arctg2 j = Ln +j 2 2 2 3 2 3 2 1+ 2


V d 2: Gii phng trnh 4cosz + 5 = 0 5 5 25 5 5 3 1 = jLn Ta c: cos z = , z = Arc cos = jLn 4 4 4 16 4 4 Nu trc cn ly du + ta c: 5 3 1 1 z = jLn + = jLn = jln + j( + 2k) = (2k + 1) + j ln 2 4 4 2 2 Nu trc cn ly du - ta c: 5 3 z = jLn = jLn( 2) = j[ln 2 + j( + 2k)] = (2k + 1) j ln 2 4 4 Tm li: z = (2k + 1) j ln 2 12. Hm hyperbol ngc: Ta gi w = Arshz l hm ngc ca z = shw w = Arshz l hm ngc ca z = shw w = Arshz l hm ngc ca z = shw Biu din cc hm ny qua logarit ta c: Arshz = Ln (z + z 2 + 1)
Archz = Ln (z + z 2 1) 1 1+ z Archz = Ln 2 1 z V d: Arshj = Lnj = j + 2k 2 13. Hm lu tha phc tng qut w = zn: Gi s a l mt s phc bt k, a = + j. Ta nh ngha: za = eaLnz (25) j t z = re ta c: Lnz = lnr + j( + 2k). Do : za = elnr-(+2k)ej[(+2k)+lnr] Trong k l mt s nguyn tu . T biu thc trn ta thy, nu 0 th hm za c v s tr.To v ca chng nm trn ng trn | w | = elnr-(+2k) , k = 0, 1, 2, 3,...
44

cn argumen ca chng l: ( + 2k) + lnr , , k = 0, 1, 2, 3,... Nu = 0, ngha l a l mt s thc th cc to v ca za nm trn vng trn | w | = elnr = r v argumen ca z l + 2k p C th chng minh c rng nu l mt s hu t, chng hn = , th ch c q q to v khc nhau ca z . Trong trng hp ny hm w = z l hu hn tr. Nu l mt s v t th hm w = z l v s tr. Ta cng c th tch c nhnh n tr ca hm w = za. im z = 0 l im r nhnh ca n. V d: Tm jj v 32+j Theo nh ngha ta c:

j =e =e =e 2+ j ( 2+ j) Ln 3 ( 2+ j)(ln 3+ 2 jk ) 3 =e =e = e ( 2 ln 32 k )+ j(ln 3+ 4 k ) = e ( 2 ln 32 k ) [cos(ln 3) + j sin(ln 3)]


j jLnj

j + 2 k j 2

+ 2 k 2

3. MT S V D V PHP BIN HNH BO GIC Mun lm mt bi ton v php bin hnh bo gic ta phi bit vn dng cc php bin hnh c bn. Nu l php ng dng, ta dng hm tuyn tnh. Mun bin mt cung trn thnh cung trn (hay ng thng) ta dng hm phn tuyn tnh. Mun bin mt gc thnh na mt phng ta dng hm lu tha. Mun bin mt bng song song vi trc thc ln na mt phng ta ngh ti hm m. Cng thc Schwartz - Christophell cho php bin a gic thnh na mt phng. Hm Giucovski bin min ngoi ng trn n v ln mt phng b i lt ct dc theo on [ -1, 1 ] V d 1: Tm php bin hnh n dip v bo gic bin min hnh qut 0 < arg z < 6

ln hnh trn n v | w | < 1 sao cho nh ca cc im z1 = e , z 2 = 0 ln lt l cc im w1 = 0 v w2 = j v y w B z O1 u /6 O x O A A B

j 12

45

D dng thy hm = z6 bin min qut 0 < arg z <

ln na mt phng trn 6 ( 0 < arg z < ). Mt khc ta li bit php bin hnh, bin na mt phng trn ln hnh trn n v | w | < 1 l: a w = e j a Vy php bin hnh min qut 0 < arg z < ln hnh trn n v c dng 6 6 z a . w = e j 6 z a Ta s xc nh v a sao cho cc iu kin ph c tha mn. T w(ej/12) = 0 hay

w =e

e a e a
j 2

j 2

= 0 suy ra a = e = j, a = j . Vy:

j 2

z6 j z6 + j Cui cng, php bin hnh phi tm l: z6 j w = j 6 z +j V d 2: Tm php bin hnh, bin na mt phng trn ca hnh trn n v G={| z | < 1. Imz > 0} ln mt phng trn y A1 B z B1 A A A1 O A1 x O 1 -1 w = e j
v A2

A2 O

A2
B2

O1

46

z 1 bin im z = 1 thnh im = 0, im z = -1 z +1 thnh im = . Nh vy on AA c bin thnh na trc thc m. Do tnh cht z 1 bo gic, cung trn ABA c bin thanh na trc o trn. Vy hm = bin z +1 min G thnh gc phn t th hai < arg < . Thc hin php quay mt gc 2 2 quanh gc to bng php bin i = -j ta c gc phn t th nht 0 < arg < . Sau ta t w = 2 ta s tng gc nh A ln gp i bin thnh 2 na mt phng trn Imw > 0. Tm li php bin hnh phi tm l: 2 z 1 2 2 w = ( j ) = = z + 1 V d 3: Tm php bin hnh bo gic bin min G z <1 j 1 z 2 > 2 tc min gii hn bi ng trn n v tm O v ng trn tm ti w = 0.5j, bn v knh 0.5, thnh min D l bng -1 < Rew < 1

Ta dng hm phn tuyn tnh =

B(j) I A

y z A

x CR

A1
O

B1 A2

-1 O1

1 u

B Nu ta dng mt hm phn tuyn tnh bin im z = j thnh im w = th hai j 1 ng trn | z | =1 v z = s bin thnh hai ng thng song song. Hm phn 2 2 1 tuyn tnh c th chn l = z j 1 1 j 1 1 Ta c (1) = = (1 + j), (1) = = ( 1 + j), (0) = j, ( j) = 1 j 2 1+ j 2 2 1 T ta suy ra nh ca ng trn | z | = 1 l ng thng Im = , nh ca ng 2

47

trn

j 1 = l ng thng Im = 1. Min G c bin thnh bng 2 2

1 < Im < 1 . By gi ta ch cn thc hin php ng dng tc l php bin hnh tuyn 2 tnh bin min D thnh mt phng w: 3j w = 4 j = 4 j + 3 4 1 3z + j Tm li w = 4 j l php bin hnh phi tm. +3= z j z j V d 4: Trong mt phng z cho cung trn AB: A l to v ca z = a, B l to v ca z = -a, trung im H ca cung trn AB l to v ca z = jh. Trong mt phng w cho ng trn i qua hai im w = a v tm ti w = jh. Hy tm mt php bin hnh bo gic bin min ngoi G ca cung AB(tc l mt phng z c mt lt ct dc theo cung AB) thnh min D l min bn ngoi hnh trn . Ch l vi cc gi thit cho , tip tuyn ti mt B vi cung AB to vi trc h Ox mt gc ( - ) vi = arctg . Cn trong mt phng w tip tuyn vi ng trn a ti w = a to vi trc Ou mt gc . 2 za Ta dng hm = bin cung AB thnh tia B1A1 trong mt phng . Qua z+a php bin hnh ny nh ca B l B1 trng vi gc to . nh ca A l A1 = . V d 1 d = > 0 vy arg = 0 nn tia A1B1 cng nghing vi trc thc mt gc dz z=a 2a dz z=a (-). Qua php bin hnh ny, min ngoi ca cung trn AB c bin thnh min G1 l min ngoi ca tia B1A1 (tc l mt phng c mt lt ct dc theo A1B1) v y

H(jh) A a z O
/2

w
/2-/2

B -a

N(-a)

O1 /2

C(a)

48

A1

N1 E1 O C1 N1
/2-/2

O B1

V pha mt phng w, ta cng thc hin mt php bin hnh phn tuyn tnh bin cung trn thnh ng thng. Php bin hnh c chn l: w a = w+a Qua php bin hnh ny, ng trn bin thnh ng thng C1E1N1 i qua gc. d 1 = > 0 nn nh ca C l C1 trng vi gc to . nh ca N l N1 = . V dw w =a 2a ng thng C1E1N1 cng to vi trc thc gc . Min ngoi ca ng trn 2 c bin thnh min D1 l na mt phng nm bn phi ng thng N1C1E1. Nh php bin hnh = 2 min D1 c bin thnh min G1. Qua php bnh phng ny ng thng C1N1 gp li thnh tia B1A1. Tm li, min G bn ngoi cung trn AB trong mt phng z c bin thnh min D l min ngoi ng trn nh php bin hnh: 2 za w a = z+a w+a T rt ra: 1 a2 z= w+ hay w = z + z 2 a 2 2 w V d 5: Tm php bin hnh bin min D = { -V < Imw < V } ca mt phng w ln min G l mt phng z b i hai lt ct Imz = jh v Rez < 0. Ta s tm php bin hnh bin bng 0 < Imw < V ln na mt phng Imw > 0 b i lt ct I = jh sao cho nh ca trc thc Imw = 0 l Imz = 0. Sau dng nguyn l i xng. v E w V O1 B Cu C E1 O B1 C1
49

A1 A1

C1

y C3 A3 A3 E3 O jh B3 C3 z x

C2

E2 O A2 B2 C2

w bin bng 0 < Imw < V thnh V bng 0 < Im< trong mt phng . Qua php bin hnh ny trc thc ca mt phng w c bin thnh trc thc ca mt phng ; ng thng Imw = V bin thnh ng thng Im = . z Tip theo ta dng php bin hnh = e bin bng 0 < Im < thnh na mt phng trn. nh ca trc thc Im = 0 l na trc thc dng trong mt phng . z bin na mt phng trn Im > 0 ln na mt phng trn Imw > 0, b i lt ct Imz = jh, ta dng hm : h z = ( + 1 + ln ) Qua php bin hnh ny na trc thc dng trong mt phng c bin thnh c trc thc trong mt phng z. z Tm li bin bng 0 < Imw < V ln na mt phng Imz > 0, b i lt ct Imz = jh, Rez < 0, ta dng php bin hnh: w w h h V z = (e + 1 + ln e ) = e + 1 + (17) V Dng nguyn l i xng ta thy php bin hnh (17) cng bin bng -V < Imw < V ln min G.

zTrc ht ta dng php bin hnh tuyn tnh =

50

CHNG 3: TCH PHN HM PHC


1. TCH PHN NG CA HM BIN PHC 1. nh ngha: Cho ng cong C nh hng, trn tng khc v trn C cho mt hm phc f(z). Tch phn ca f(z) dc theo C c nh ngha v k hiu l:
lim f ( t k )(z k z k 1 ) = f (z)dz
n k =1 C n

(1)

Trong a = zo , z1,..,zn = b l nhng im k tip nhau trn C; a v b l hai mt, tk l mt im tu ca C nm trn cung [ zk, zk-1]. Gii hn (1) thc hin sao cho max lk 0 vi lk l di cung [ zk, zk-1]. 2. Cch tnh: t f(z) = u(x,y) + jv(x,y), zk = xk + jyk xk = xk - xk-1, yk = yk - yk-1 tk = k +jk; u(k , k) = uk; v(k , k) = vk ta c:

f ( t k )(z k z k 1 ) = (u k x k v k y k ) + j (u k x k + v k y k )
k =1 k =1 k =1

(2)

Nu ng cong C trn tng khc v f(z) lin tc tng khc, gii ni th khi n v phi ca (2) tin ti cc tch phn ng ca hm bin thc. Do tn ti: (3) f (z) = (udx vdy) + j (udy + vdx )
C C C

Nu ng cong L c phng trnh tham s l x = x(t), y = y(t) v t th ta c th vit di dng hm bin thc: z = x(t) + jy(t) = z(t) t vi z(a) = ; z(b) = . Khi ta c cng thc tin dng:
C

f (z)dz = f [ z( t ) ].z( t ) dt

(4)

V d 1: Tnh I = Rezdz , L l on thng ni 2 im 0 v 1 + j theo chiu t 0 n L y y 1+j.

j C

1 x -a O O Phng trnh tham s ca L c th ly l: x ( t ) = t Vy z(t) = (1 + j)t, t thc t [0, 1] y ( t ) = t im O ng vi t = 0 v im B ng vi t = 1. Theo (4):

51

I = Re(1 + j) t.z( t )dt = (1 + j) tdt = (1 + j) tdt =


0 0 0

1+ j 2

dz , L l na cung trn nm trong na mt phng trn, ni im -a L z v a, chiu ly tch phn t -a n a. Phng trnh tham s ca ng cong L l: x = acos t y = asin t Vy z(t) = a(cost + jsint) = aejt, z(t) = jaejt. im -a ng vi t = , im a ng vi t = 0. Theo (4): 0 dz 0 jae jt dt I= = = j dt = j jt L z ae V d 3: Tnh I = (1 + j 2 z )dz , C l cung parabol y = x2, ni gc O v im B c

V d 2:Tnh I =

to (1,1). Hm f(z ) = 1 + j 2z = 1 + j 2( x jy) . Tch phn thc v phn o ta c u(x, y)=1-2x v(x, y) = 1 + 2y. Dng (3) ta c: I = (1 2 x )dx (1 + 2 y)dy + j (1 + 2 y)dx + (1 2 x )dy
C C

Chuyn mi tch phn ng loi 2 thnh tch phn xc nh ta c:


C

(1 2x )dx (1 + 2 y)dy = (1 2x )dx (1 + 2x (1 + 2 y)dx + (1 2x )dy = (1 + 2x


0 0 1 2

)2 xdx = (4x 3 4x + 1)dx = 2


0 1

)dx + (1 2x )2xdx = (2x 2 + 2x + 1)dx =


0

4 3

Thay vo trn ta c: 4j I = 2 + 3 V d 4: Tnh I = z 2 dz , AB l on thng ni im A l to v ca s phc 2 v


AB

im B l to v ca s phc j. f(z) = z2 = (x + jy)2 = (x2 - y2 + 2jxy) nn u = x2 - y2 v v = 2xy. Theo (3) ta c: I = ( x 2 y 2 )dx 2 xydy + j ( x 2 y 2 )dy + 2 xydx
AB AB

V AB c phng trnh x = 2 - 2y, dx = -2dy (chn y lm tham s) nn:


AB

(x

y )dx 2xydy = (4 + 4 y 2 8y y 2 )(2dy) 2(2 2 y)ydy =


2 0 1

8 3 1 3

AB

2 2 2 2 ( x y )dy + 2xydx = (4 + 4 y 8y y )dy + 2 y(2 2 y)(2 ydy) = 0

Thay vo ta c:
52

8+ j 3 2 V d 5: Tnh I k = (z ) dz k = 1, 2 I=
C

vi C1 l on thng ni 0 v 1 + j v C2 l ng gp khc ni 0, 1, 1 + j p dng (4) vi C1 ta c z = (1 + j)t, t i t 0 n 1 nn: 1 2 2 I1 = (z ) dz = (1 j) 2 t 2 (1 + j)dt = (1 j) 3 C1 o Tng t: 1 1 2 2 2 I1 = (z ) dz = t dt + (1 jt ) 2dt = (2 + j) 3 o o C2


3. Cc tnh cht ca tch phn: T cng thc (3) ta suy ra rng tch phn ca hm bin phc dc theo mt ng cong c tt c cc tnh cht thng thng ca mt tch phn ng loi 2. Ta nu li cc tnh cht : - Tch phn khng ph thuc tn gi bin s tch phn f (z)dz = f ( )d
AB

AB

[f (z) + g(z)]dz = f (z)dz + g(z)dz


AB AB

AB

- Nu a l hng s phc th: af (z)dz =a f (z)dz


AB AB

AB

f (z)dz = f (z)dz
BA

- Nu A, B v C l 3 im cng nm trn mt ng cong th: f (z)dz = f (z)dz + f (z)dz


AC z AB BC

zo

dz = z z o

4. Cc cng thc c lng tch phn: Nu M l gi tr ln nht ca | f(z) | trn ng cong L (ngha l | f(z) | M z L) th ta c:

f (z)dz
L n

f (z)dz dz Ml
L

(5)

Chng minh: V mun ca mt tng nh hoc bng tng cc mun nn:

f ( k )z k
k =1

f ( k ) z k
k =1

Nhng theo gi thit | f(k) | M nn:

53

f ( k )z k M z k = M z k
k =1 n k =1 k =1

Vy:

f ( k )z k M z k
k =1 k =1

Ch l

k =1

z k bng chiu di ng gp khc c cc nh ti zo, z1, z2 ,..,zn. Khi

max | zk | 0 th trong (6) ta c:

k =1

z k dn ti di l ca ng cong L. Chuyn qua gii hn

f (z)dz
L

Ml

(5)

2. NH L CAUCHY CHO MIN N LIN 1. nh l: Nu f(z) gii tch trong min n lin D v C l mt ng cong kn nm trong D th: (6) f (z)dz = 0
L

Chng minh: Gi thit ch i hi f(z) gii tch trong D , nhng vi gi thit ny, cch chng minh s kh hn. n gin cch chng minh, ta gi thit thm f(z) lin tc trong D . Vy u(x, y) v v(x, y) lin tc v c o hm ring lin tc trong D . Theo (3) th: f (z)dz = udx vdy + j vdx + udy
L L L

Trong gii tch, nu bit P(x, y), Q(x, y) lin tc v c o hm ring lin tc trong Q P D th iu kin cn v Pdx + Qdy = 0 C D l = x y C p dng kt qu cho, ta thy udx vdy = 0 . Tht vy, y P = u v Q = -v. Do
L

gi thit f(z) gii tch nn cc iu kin C - R c tho mn, vy u P ( v) Q = = = y y x x Tng t ta chng minh c vdx + udy = 0 . Do f ( z)dz = 0 L L y
V d 1: Nu L l ng cong kn bt k gii hn mt min n lin G, th e z dz = 0 v f(z) = ez gii tch
L

j 0 -j
54

trong c mt phng.

V d 2: Tnh I =

sin z dz , L l ng trn | z - 1| = 1. 2 + z 1 L

sin z c hai im bt thng l nghim ca phng trnh z2 + 1 = 0 l j. 2 z +1 Vy f(z) gii tch trong min | z - 1 | 1 . p dng nh l Cauchy ta c I = 0. dz V d 3: Tnh I = , L l ng trn tm zo, bn knh R, tch phn ly theo L z z0 chiu dng. Phng trnh tham s ca L l: x = x o + a cos t y = y o + a sin t Vy z(t) = x(t) + jy(t) = zo + aejt; z(t) = jaejt. Theo (4) ta c: 2 jae tj I = tj dt = 2j 0 ae 1 c im bt thng ti z = zo v gi thit ca nh l S d I 0 v hm f (z) = z z0 Cauchy khng c tho mn. Qua v d ny ta thy nu f(z) c im bt thng trong G th nh l Cauchy khng cn ng na.

Hm f ( z ) =

V d 4: Tnh I = ze z dz
o

Ta c th vit: I = ze dz = ze
z o j+1 1

z j o

e z dz = je j (e j 1) = 1 + ( j 1)(cos1 + j sin 1)
o

= (1 cos1 sin 1) + j(cos1 sin 1)


V d 5: Tnh I = (z 1)100 zdz

t t = z - 1 ta c:

t102 t101 j102 j101 1 j I = t100 ( t + 1)dt = ( t101 + t100 )dt = + = + = + 102 101 102 102 o o 0 102 101
j j

3. NH L CAUCHY CHO MIN A LIN 1. nh l: Gi s min G l a lin m bin L gm ng cong bn ngoi Lo, v cc ng cong bn trong L1, L2,.., Ln.(hnh a) Nu f(z) l mt hm gii tch trong G th: (7) f (z)dz = f (z)dz + f (z)dz + L + f (z)dz
Lo L1 L2 Ln

55

L1 L2 Lo L1

A B Lo

Cc tch phn u ly theo hng dng, ngha l ngc chiu kim ng h. Chng minh: Ta s chng minh f (z)dz = f (z)dz
Lo L1

nu bin bn trong ch c mt ng cong kn L1(hnh b). Cch chng minh tng t nu bin bn trong c nhiu ng. Gi s AB l lt ct ni im A trn ng Lo v im B trn ng L1. Do lt ct AB, min G tr thnh n lin, do c th p dng nh l Cauchy nu phn trn. Ta c: f (z)dz + f (z)dz + f (z)dz + f (z)dz = 0
Lo AB L1 BA

K hiu

L1

f (z)dz ch tch phn theo hng thun chiu kim ng h.

Theo tnh cht ca tch phn ta c: f (z)dz = f (z)dz


L1 L1 AB

f (z)dz = f (z)dz
BA

Thay vo trn ta c: f (z)dz f (z)dz = 0


Lo L1

y l iu cn chng minh. Ghi ch: Cng thc (7) c th vit thnh: f (z)dz f (z)dz f (z)dz L f (z)dz = 0
Lo L1 L2 Ln

hay:

Lo

f (z)dz + f (z)dz + f (z)dz + L + f (z)dz = 0


L1 L2 Ln

hay gn hn: f (z)dz = 0


Lo + L1+L+ Ln

Gi L l bin c hng dng ca min G th ng thc trn c vit l: f (z)dz = 0


L

y l cng thc (1) suy rng cho min a lin.


56

H qu: Gi s f(z) gii tch trong min D c bin C v lin tc trong D th vi mi ro D th: f (z) = 2jf (z o ) C z zo V d :Tnh I =

dz vi n nguyn dng, zo cho trc. L l ng cong kn n ( ) z z L o

khng qua zo
zo

zo

Gi G l min gii hn bi ng cong L. 1 l hm gii tch trong G nn theo nh l Gi s zo G. Khi f (z) = (z z o )n Cauchy th I = 0 Gi s zo G. Loi khi G mt min l hnh trn tm zo , bn knh a. Nh vy f(z) s gii tch trong min nh lin cn li. Theo (8) th: dz dz I= = n n L (z z o ) (z z o ) l ng trn | z - zo | = a. Nu n = 1 th I = 2j Nu n 1, ch l khi z th: z = zo + aejt, dz = jaejtdt 0 t 2 Vy: 2 2 dz jae jt dt j 2 j(1n ) t 1 I= = = e dt = e j(1n ) t = 0 v ej(1-n)2 = eo = 1 n n jnt n 1 n 1 0 a 0 (1 n )a 0 a e L (z z o ) Ta tm tt kt qu dng sau ny: 2 j khi n = 1, L bao z o dz (z z )n = 0 z L o
2. Tch phn khng ph thuc ng i: nh l: Gi s f(z) l mt hm gii tch trn min n lin G v zo l mt im c nh thuc G. Khi tch phn ca hm f(z) dc theo mt ng cong kn nm

trn trong G, i t im zo n im z phn

zo

f (z)dz

khng ph thuc vo ng ly tch

57

Nu cn trn z thay i th tch phn l mt hm gii tch ca z trong G v c o hm c xc nh bi cng thc: z d f (z)dz = f (z) (10) dz zo Chng minh: Ly hai ng cong bt l L1 v L2 nm trong G v i t zo n z. Do f(z) gii tch nn p dng nh l Cauchy cho ng cong kn MomMnMo: f (z)dz + f (z)dz = 0
M o mM MnM o

L2 Mo(zo) n M(z) m L1

hay:

M o mM

f (z)dz f (z)dz = 0
M onM L2

tc l: f (z )dz = f (z )dz
L1

V L1 v L2 l bt k nn ta c th kt lun rng tch phn i t zo n z khng ph thuc ng ly tch phn m ch ph thuc cn trn z. By gi ta cn phi chng minh rng nu t F(z) = f (z)dz th F(z) = f(z). V
zo z

tch phn khng ph thuc ng i nn ng vi mi z tch phn c mt gi tr hon ton xc nh. Vy F(z) l mt hm n tr. Ta c: F = F(z + z) F(z) =
z + z zo

f ( ) d f ( ) d = + f ( ) d = f ( ) d
zo z z

zo

z + z

V f() gii tch, nn n lin tc ti z. Do c th vit f() = f(z) + () vi () l hm gii tch, dn ti 0 khi 0. Vy:

F =

z + z z

[f (z) + ()]d = f (z)d + ()d = f (z) d + ()d


z z z z

z + z

z + z

z + z

z + z

= f (z)z +

z + z z

()d

()d F hay (11) = f (z) + z z z Cho z0 th z + z z. S hng th hai bn v phi dn ti 0. Tht vy, do tch
z + z

z + z

phn khng ph thuc ng i nn trong tch phn

()d
z

ta chn ng i t z

ti z + z l on thng ni hai im . Chiu di on thng ny l | z |. Sau p dng cng thc c lng tch phn ta c:
z + z z

()d

max ( ) . z
58

z + z

Vy:

( ) d
z

max ( )
z + z

Cho z0 th max | () | 0. Do

()d
z

0 . T (11) ta suy ra F(z) = f(z).

4. TCH PHN BT NH Ta gi F(z) l nguyn hm f(z) nu F(z) = f(z). Hin nhin, nu F(z) l nguyn hm ca f(z) th F(z) + C , trong C l mt hng s phc cng l nguyn hm ca f(z). Ngc li nu (z) v F(z) u l nguyn hm ca f(z) th chng phi khc nhau mt hng s phc C: (z) - F(z) = C Tht vy, t g(z) = (z) - F(z) = u(x, y) + jv(x, y). Ta phi chng minh rng u v v l nhng hng s. Ta c: g(z) = (z) - F(z) = 0 (12) Nhng theo cng thc tnh o hm: u u v v g(z) = +j = j =0 x x y y Nh vy: u v u v = = 0; = =0 x y y x Ngha l u(x, y) v v(x, y) l cc hng s. Ta suy ra rng nu F(z) l nguyn hm ca f(z) th h hm s F(z) + C vi C l mt hng s phc tu , cha tt c cc nguyn hm ca f(z). Ta gi h hm s ny l tch phn bt nh ca hm f(z) v k hiu l f (z)dz .

Tm li: f (z)dz = F(z) + C : F(z) = f (z) Theo bng o hm ta c th suy ra bng nguyn hm, ging nh trong tch phn thc: z z e dz = e + C
z n +1 z dz = n + 1 + C sin zdz = cos z + C
n

zdz = ln z + C
59

5. CNG THC NEWTON - LEIBNITZ nh l: Gi s f(z) l mt hm gii tch trong min n lin G v c nguyn hm F(z). Khi :
z2 z1

f ()d = F(z 2 ) F(z1 )


z zo

(13) cng l mt nguyn hm f(z). Vy:

Chng minh: Ta bit l

f ( )d

zo z

f ()d = F(z) + C f ( )d = F(z) F(z o )


1 f ()d = F(z1 ) F(z o ) = F(z) z z

Thay z = zo vo 2 v ta c: 0 = F(zo) + C. Do C = - F(zo). Nh vy:


zo
z1 zo

Khi z = z1:
o

Cng thc ny c gi l cng thc Newton - Leibnitz. Khi tnh tch phn ca mt hm gii tch ta dng trc tip cng thc ny m khng a v tnh tch phn ng loi 2.
V d 1: Tnh I = z 2 dz
2 j

z3 j+8 = I = z dz = 3 2 3 2 dz Vi d 2: Tnh I = , L l cung trn i t im z = -a n im z = a( a> 0) L z


j 2

dz a dz I = = = ln z = ln a ln(a ) = ln a [ln a + j arg(a )] = j a z L z a


V d 3: Tnh I = ze z dz
0 j

I = ze dz = ze
z 0

z j 0

e z dz = ze z e z = 0.381 0.301 j
0 0 0

6. CNG THC TCH PHN CAUCHY 1. Tch phn Cauchy: nh l: Gi s G l mt min n lin hoc a lin gii hn bi bin L v z l mt im bn trong G. Nu f(z) gii tch trong G th ta c cng thc:
60

1 f ( ) d (14) 2 j L z Tch phn bn v phi c gi l tch phn Cauchy ca hm f(z). Cng thc (14) c gi l cng thc tch phn Cauchy. ngha: Cng thc ny cho php ta tnh c gi tr ca hm gii tch bn trong min G khi bit gi tr ca n trn bin. Ni khc i, gi tr ca hm gii tch trong min, hon ton c xc nh bi gi tr ca n trn bin. Chng minh: Ly zo bt k trong min G, ta s chng minh zo rng: 1 f ( )d f (z o ) = (15) 2 j L z o L f (z) . Loi khi min G mt hnh trn t (z) = z zo bn knh r bt k nh c tm ti zo th (z) s gii tch trong min a lin cn li. p dng nh l Cauchy cho min a lin ta c: f ( ) d f ( ) d z = z L o o f (z) = l ng trn | - zo | = r V cng thc trn ng vi mi r kh b nn ( ng trn nm trong min G) ta c th vit: f ( )d f ( ) d z = lim r 0 zo L o Ta c: f ( ) f ( z o ) f ( ) f ( z o ) + f ( z o ) 1 f ( ) d d d + f ( z o ) d = z = z z z o o o o = f ( ) f ( z o ) d + 2 jf (z o ) zo (16)

Do tnh lin tc ca hm f() nn > 0 ta c th chn r kh b | f() - f(zo) | < . Khi ta c | - zo | = r v: f ( ) f ( z o ) < r zo p dng cng thc c lng tch phn ta c: f ( ) f ( z o ) 2r = 2 z r o V b tu nn:
61

f ( ) f ( z o ) d = 0 r 0 z o T (16) suy ra: f ( ) lim d = 2f (z o ) r 0 z o l iu cn chng minh. Nh cng thc tch phn Cauchy ta c th tnh mt s tch phn ly dc theo mt ng cong kn. e z dz V d 1: Tnh I = trong cc trng hp sau: L z ( z 3) - L l ng trn tm ti 2, bn knh 1.5(ng L1) L2 - L l ng trn tm O, bn knh 0.25(ng L2) - L l ng trn tm 0.5, bn knh 5(ng L3) L1 e z dz ta dng (15) - tnh tch phn I = L1 z ( z 3) lim

ez , zo = 3; hm f(z) gii tch z 3 trong hnh trn z 2 . Vy gi thit 2 ca nh l c tho mn. Ta c: e z dz ez I= = 2 jf (3) =2 j 3 L1 z ( z 3)
Chn f (z) =

L3

ez e z dz - tnh I = ta t f (z) = , zo = 0; hm f(z) gii tch trong hnh trn z3 L2 z ( z 3) 1 z . Vy gi thit ca nh l c tho mn. Ta c: 4 e z dz e0 2 j I= = 2 jf (0) = 2 j = 03 3 L1 z ( z 3) ez gii tch trong min a lin m bin ngoi l z(z 3) Lo v hai bin trong l L1 v L2. p dng nh l Cauchy cho min a lin ta c: 1 ez e z dz e z dz e z dz z(z 3) = z(z 3) + z(z 3) = 2 j 3 + 3 L L1 L2 dz V d 2: Tnh I = 2 trong 2 trng hp: L z +1 - L l ng trn | z - 2 | = 3/2(ng L1) - L l ng trn | z - j | = 1 (ng L1) - Hm di du tch phn f (z) =
62

V hm f (z ) =

L2 3 trn z 2 nn theo nh l Cauchy ta 2 c: dz I= 2 =0 L1 z + 1 dz dz I= 2 . = L1 L2 z + 1 L 2 ( z + j)( z j) 1 Ta t f (z) = , zo = j. Hm f(z) gii tch trong hnh trn | z - j | 1. p dng (15) z+ j ta c: 1 dz 1 = f ( j) = 2 2 j L 2 z + 1 2j Nh vy: dz z2 + 1 = L2 2. Tch phn loi Cauchy: nh ngha: Gi s L l mt ng cong trn v f(t) l mt hm lin tc trn L. Xt hm: 1 f ( t )dt , z bt k L (17) ( z) = 2 j L t z Nu z L th hm s di du tch phn l mt hm lin tc. Vy tch phn tn ti v cho ta mt hm s ca z xc nh khp ni, tr cc im thuc L. nh l: Hm (z) xc nh bi tch phn loi Cauchy mt hm gii tch ti mi im z L. o hm cp n ca n c tnh theo cng thc: n1 f ()d , z bt k L ( n ) (z) = 2j L t z

1 gii tch trong hnh z +1


2

7. O HM CP CAO CA MT HM GII TCH 1. o hm cp cao ca mt hm gii tch: nh l: Nu f(z) gii tch trong min gii ni D v lin tc trong D vi bin C th ti mi z D hm f(z) c o hm mi cp v: n! f ( t )dt (20) f ( n ) (z) = ; n = 1,2,... 2 j C ( t z) n +1 Trong chiu i trn bin C l chiu dng. Chng minh: Theo nh ngha o hm v cng thc tch phn Cauchy ta c:

63

f (z) = lim

f (z + h ) f (z) 1 1 1 1 lim f ( t ) = dt h 0 h 2 j h0 h C t z h t z 1 1 f ( t )dt 1 f ( t )dt lim = = 2 j h0 h C ( t z h )( t z) 2 j C ( t z) 2


1 z c nh tzh

Vic qua gii hn di du tch phn thc hin c v hm g( t ) = thuc D v t chy trn C hi t u trn C n

1 khi h 0. tz Ta chng minh cng thc trn vi n = 1. Vi n > 1 ta chng minh bng cch quy np. Nh vy ta suy ra nu f(z) gii tch trong min n lin gii hn bi ng cong C v lin tc trong D , zo D th : f (z)dz 2 j ( n ) (z z ) n1 = n! f (z o ) n = 0, 1, 2,... C o vi quy c 0! = 1, f (o)(zo) = f(zo). cos zdz V d: Tnh I = , L l ng trn | z - j | = 1 3 L ( z j) Ta vit cng thc (20) di dng khc: n! f (z)dz f ( n ) (z o ) = 2 j L (z z o ) n +1 Trong cng thc ny f(z) = cosz, zo = j, n = 2. Ta c: cos zdz 2 jf ( j) I= = = jf ( j) 3 2! L ( z j) Do f(z) - -sinz, f(z) = -cosz nn f(j) = -cosj = -ch1. Vy: I = -jch1 2. Bt ng thc Cauchy v nh l Liouville: a. Bt ng thc Cauchy: Gi s G l mt min c bin L v f(z) l hm gii tch trong G . Gi M l gi tr ln nht ca | f(z) | trong min G , R l khong ch t im zo G ti bin, l l di ca L th t (20) suy ra: n! f ( t )dt n!Ml f ( n ) (z o ) n +1 2 L ( t z o ) 2R n +1 Nu G l hnh trn | z - zo | < R th l = 2pR v cng thc trn tr thnh: n!M (21) f ( n ) ( z o ) n1 R Bt ng thc trn gi l bt ng thc Cauchy. b. nh l Liouville: Nu hm f(z) gii tch trong ton mt phng v b chn th n l mt hng s.

64

Chng minh: Gi thit | f(z) | < M z C. T (21) suy ra f (z)

V v tri khng ph thuc R nn | f(z) | = 0 z C. Tm li f(z) = 0 trong ton mt phng, p dng cng thc Newton - Leibnitz, chn zo c nh ta c:

M vi R ln. R

f (z) f (z o ) = f (z)dz = 0
zo

Vy f(z) = f(zo) z.
8. CNG THC SCHWARTZ - CHRISTOPHELL a. nh l: Gi P l mt a gic trong mt phng w c n nh l w1, w2, w3,.., wn vi wk k wn v y w1

w2 wk w3 O1 u a1 O a2 an x

Gi k l gc trong ca a gic ti nh wk v 0 < k < 2 :

k = (n 2)
k =1

Hm w = f(z) bin na mt phng trn Imz > 0 ln min trong ca a gic P sao cho nh ca cc im a1, a2,..an - < a1 < a2 < < an < trn trc thc Ox l cc nh w1, w2,.., wn ca a gic P, c xc nh bi cng thc Schwartz - Christophell:

f (z) = C ( a1 ) .( a 2 ) L ( a n ) d + C1
1 1

1 1

(10)

zo

Trong zo, C v C1 l cc hng s phc. b. Dng khc ca cng thc Schwartz - Christophell: Nu mt nh ca a gic tng ng vi im , chng hn nh w1 tng ng vi a1 = , th (10) c thay bi:

f (z) = C ( a 2 ) .( a 3 ) L ( a n ) d + C1
1 1 1

(11)

zo

Nh vy trong (11) vng mt tha s ( a 2 ) Tri li nu mt trong cc nh ca a gic l im , chng hn wk = th trong (10)
1

65

ta phi t k = -k trong k l gc gia hai cnh cng i qua wk ti giao im hu hn ca chng. c. S dng cng thc Schwartz - Christophell: Khi ta phi bin mt a gic P cho trc trong mt phng w ln na mt phng Imz > 0 th ta s dng cng thc (10). Ch l ta cha bit ak l nh ca cc nh a gic v cc hng s zo, C1 v C2. Theo nh l Rieman, ta c th chn tu nh ca 3 nh a gic, ngha l chn tu 3 s a1, a2 v a3. Cc s an cn li v nhng hng s tch phn zo, C1, C2 s c xc nh tu theo iu kin bi ton, V d 1: Bin min G gch cho ln na mt phng Imz > 0 v A1 A1 A3

A2

O1

A2 u

a2

a3 1 O

a1

a2 x

Min G c th coi l mt tam gic c nh A1 = , A2 = v A3 c to v w = jh. Cc gc nh tam gic l 1 = 0, 2 = -, 3 = + . Ta s bin cc im A1, A2 v A3 ln lt thnh cc im a1, a2 v a3. Ta c:

w = C z 1 (z + 1) dz + C1
zo

V w(-1) = jh nn ta c th ly zo = -1 v C1 = jh. Vy: z (z + 1) w = C dz + jh z 1 xc nh hng s tch phn C, ta s lm nh sau: cho im z chy trn na cung trn bn knh r kh b z = rej sao cho bin thin t n 0. Gi w l s gia tng ng ca w khi z chy trn cung trn . Ta c: (z + 1) w = C dz z Khai trin (1 + z) theo lu tha ca z ta c: ( 1) 1 w = C + + z + L dz 2! z t z = rej ri tch phn theo t n 0 ta c: w = - Cj +O(r) trong O(r) 0 khi r 0 Mt khc trong mt phng w im w tng ng chuyn t tia A1A3 sang tia A1A2 nn ta c: w = - jh +O(r)
66

h Tm li php bin hnh phi tm l hm ngc ca hm: h z (z + 1) (13) w= dz + jh 1 z Trng hp = 1 ta c: h z (z + 1) h h z w= dz + jh = (z + ln z) 1 + jh = (z + 1 + ln z) (14) 1 z y l php bin hnh , bin na mt phng Imw > 0 c mt lt ct dc theo A1A3 thnh na mt phng trn Imz > 0. V d 2: Tm php bin hnh bo gic bin hnh ch nht c cc nh A1(w1 = k), A2(w2= h+ jk) , A3(w = -h + jk), A4(w4 = -h) ln na mt phng trn Imz > 0 T suy ra -jh = - Cj hay C =

v A3 B A2 a4 1/k a3

a1 1

a2 1/k x

A4

O1

A1 u

1 O

Gi w = f1(z) l php bin hnh bin gc phn t th nht (Rez > 0, Imz > 0) thnh hnh ch nht O1A1A2B sao cho o1 ng vi O. A1 ng vi im z = 1 B ng vi im z = . Trong php bin hnh ny A2 s ng vi im z = 1/k vi k l mt hng s dng nh hn 1 m ta phi xc nh. Qua php bin hnh, on BO1 ng vi na trc Oy dng. Theo nguyn l i xng, hm w = f(z) l hm phi tm bin na mt phng trn Imz > 0 ln hnh ch nht A1A2A3A4 l thc trin ca hm f1(z) qua trc o. Cng theo nguyn l i xng, cc im i xng qua BO1 ng vi cc im i xng qua Oy. Vy A4 ng im z = -1; A3 ng vi im z = -1/k. p dng cng thc Schwartz - Christophell vi 1 = 2 = 3 = 4 = v zo = 0 ta c: 2

w = f (z) = C ( + 1)
0

1 1 2

( 1)

1 1 2

1 k

1 1 2

1 + k

1 1 2

d + C1

V f(0) = 0 nn C1 = 0, vy: z z d d = C w = C 2 ( 1 )(1 k 2 2 ) 0 0 2 1 2 ( 1) 2 k Ta cn phi xc nh hng s C v k. V A1(w1 = h) ng vi = 1 nn:


67

h = C
0

(1 )(1 k )
2 2 2

V A2(w2 = h +jk) ng vi z = 1/k nn : 1 1 1 k k d d C h + jk = C + j = (1 2 )(1 k 2 2 ) ( 1 2 )(1 k 2 2 ) 0 0 0


= h + j
0 1 k

2 2 2 (1 )(1 k ) (15) d

(1 )(1 k )
2 2 2

Suy ra:
k=
0 1 k

(1 )(1 k )
2 2 2

(16)

Cc ng thc (15) v (16) s cho php ta xc nh C v k.

68

CHNG 4: CHUI HM PHC


1. KHI NIM CHUNG 1. nh ngha: Cho dy cc hm bin phc u1(z), u2(z), u3(z),... xc nh trong min E. Ta gi biu thc:

u n (z) = u1 (z) + u 2 (z) + L + u n (z) + L


n =1

(1)

l chui hm bin phc. Tng ca n s hng u tin l: Sn(z) = u1(z) + u2(z) + + un(z) c gi tng ring th n ca chui hm (1). N l mt hm phc xc nh trong min E. Nu ti z = zo, chui

u n(z o )
n =1 n =1

hi t th zo c gi l im hi t ca chui khng hi t th zo c gi l im phn k

hm (1). Nu ti z = zo, chui

u n(z o )

ca chui hm (1). Tp hp cc im hi t ca chui hm c gi l min hi t ca n. Nu gi f(z) l tng ca chui (1) ti im hi t z th f(z) hin nhin l mt hm bin phc xc nh trong min hi t G. 2. Khi nim v hi t u: Theo nh ngha 1 ta c z G: lim Sn (z) = f (z) (2)
n

Nu t Rn(z) = f(z) - Sn(z) th ng thc (2) c vit l: lim R n (z) = 0


n

iu c ngha l > 0 cho trc, tn ti mt s N(, z) dng ph thuc vo v z sao cho khi n > N th | Rn(z) | < . a. nh ngha: Chui hm (1) c gi l hi t u trn tp Go G, nu >0 cho trc, tn ti mt s N ch ph thuc : N = N() sao cho khi n > N() th | Rn(z) | < z Go.
b. Tiu chun Weierstrass: Nu | un(z) | an z G v nu chui

a n hi t
n =1

th chui hm (1) hi t u trong min G. Ni vn tt hn, chui (1) s hi t u trong G nu chui cc mun ca n, tha nhn mt chui s dng tri hi t. Chng minh: Cho trc > 0, ta s chng minh rng tn ti N() sao cho khi n > N() th | Rn(z) | < z G. Tht vy v chui

a n hi t nn > lun lun tn ti N()


n =1

sao cho khi n > N() th: rn = an+1 + an+2 + < Nhng v | un+1(z) | < an+1, | un+2(z) | < an+2, | un+3(z) | < an+3... nn: | Rn(z) | = | un+1(z) + un+2(z) + | < | un+1(z) | + | un+2(z) | + < an+1 + an+2 + <
69

z G. l iu cn chng minh. c. Tnh cht ca chui hi t u: nh l 1: Nu tt c cc s hng un(z) ca chui hm (10) u lin tc trong min G v nu chui hm (1) hi t u trong G th tng f(z) ca n cng lin tc trong G. Chng minh: Gi s z v z + h l hai im bt k trong G. Ta c: f(z) = Sn(z) + Rn(z) f(z + h) = Sn(z + h) + Rn(z + h) Cho trc > t phi chng minh vi | h | nh, ta c: | f(z + h) - f(z) | < Tht vy: | f(z + h) - f(z) | = | Sn(z + h) + Rn(z + h) - Sn(z) - Rn(z) | = | Sn(z + h) - Sn(z) + Rn(z + h) - Rn(z) | | Sn(z + h) - Sn(z) | + | Rn(z + h) - Rn(z) | (4) Do tnh hi t u ca chui ta c th tm c s n ch ph thuc vo sao cho: R n (z + h ) < ; R n (z) < 3 3 Vi n chn trn, xt hm Sn(z). l tng ca mt s hu hn cc hm lin tc trong min G. Vy Sn(z) cng lin tc trong G. Do ta c th chn h kh nh : Sn ( z + h ) Sn ( z ) < 3 Thay vo (4) ta c: | f(z + h) - f(z) | l iu cn chng minh. nh l 2: Nu tt c cc s hng ca chui hm (1) u lin tc trn cung L v chui hm (1) hi t u trn cung th ta c th tnh tch phn tng s hng ca chui hm (1) dc theo Lo, ngha l nu f(z) l tng ca chui hm (1) th: f (z)dz = u1 (z)dz + u 2 (z)dz + L + u n (z)dz + L
L L L L

Chng minh: Trc ht ta nhn xt rng v f(z) lin tc trn L nn tn ti tch phn f (z)dz . t n = u1 (z)dz + u 2 (z)dz + L + u n (z)dz . Ta cn chng minh rng:
L L L L

lim n = f ( z)dz
n L

lim f (z)dz n = 0 n L hay lim f (z)dz [u1 (z) + u 2 (z) + L + u n (z)]dz = 0 (6) n L L V chui (1) hi t u trn L nn vi mi > 0 cho trc ta tm c N() sao cho khi n > N() th | Rn(z) | < z L. p dng cng thc c lng tch phn ta c:
hay
70

R n (z)dz
L
n L

l , l l chiu di ca cung L

V b nn lim R n (z)dz = 0 . y l iu cn phi chng minh.


d. nh l Weierstrass: Nu cc s hng ca chui hm (1) l gii tch trong min G v chui (1) hi t u trong min th tng f(z) ca chui cng l mt hm gii tch trong G. i vi chui hm (1) ta c th o hm tng s hng ti cp tu , ngha l: m) (m) (m) f ( m ) ( z) = u ( 1 ( z) + u 2 ( z) + L + u n ( z) + L z G, m nguyn bt k Chng minh: Ta nhn thy trong nh l ny khng gi thit g v tnh hi t ca chui o hm . Ly z bt k thuc G. C l ng trn tn z bn knh r kh nh sao cho hnh trn Go bao bi C nm trn trong G. chng minh f(z) gii tch trong Go, ta s chng minh f(z) c biu din bng mt tch phn loi Cauchy, c th ta s chng minh rng: 1 f ( ) d (7) f (z) = 2 j C z Tht vy, do gi thit, chui hm (1) hi t u trong G. Vy n hi t u trn C. Ta c: u1() + u2() + + un() + = f() u z C (8) 1 1 V vi C th - z 0 nn nhn 2 v vi ta c: 2 j z 1 u 1 ( ) u 2 ( ) u n ( ) f ( ) L L = + + + + 2 j( z) u z C 2 j z z z V chui hm v tri hi t u trn C nn theo nh l 2, ta c th tch phn tng s hng dc theo C: 1 u1 ( )d u 2 ( )d u n ( )d 1 f ( ) + + + + L L = d (9) z z 2 j C z C C 2 j C ( z ) Mt khc v mi s hng un(z) gii tch nn theo (9), tch phn Cauchy: 1 u n ( ) d = u n (z) 2 j C ( z ) Vy (9) vit c: 1 f ( ) u1 (z) + u 2 (z) + L + u n (z) + L = d 2 j C ( z ) 1 f ( ) tc: f (z) = d 2 j C ( z ) Vy f(z) gii tch trong min G. V trn kia ly z bt k trong G nn c th kt lun f(z) gii tch trong G. Lp lun tng t nh trn ta chng minh c rng c th o hm tng s hng ca chui (1) ti cp tu .

71

m! ta c: 2j( z) m+1 m! u1 ( ) u 2 ( ) u n ( ) m!f ( ) + +L+ + L = m +1 m +1 m +1 m +1 2j ( z) ( z ) ( z ) 2j( z) u C. Do tnh hi t u ta c th tch phn tng s hng dc theo C v c: m! f ( )d m! u1 ( )d u ( )d u ( )d + L = +L+ 1 + 1 (9) m +1 m +1 m+1 m +1 2j C ( z) C ( z ) C ( z ) 2j C ( z) V un(z) gii tch theo gi thit v f(z) gii tch do kt qu chng minh trn nn theo (2) mc 12, chng 4 ta c : m! u1 ( )d m! f ( )d ( m +1) ; = u ( z ) = f ( m ) (z) n m +1 m +1 2j C ( z) 2j C ( z) Vy (9) tr thnh: ( m +1) u1 (z) + u (2m+1) (z) + L + u (nm+1) (z) + L = f ( m ) (z) l iu cn chng minh. Nhn 2 v ca (8) vi
2. CHUI LU THA 1. nh ngha: Ta gi chui lu tha, chui hm m cc s hng l cc hm lu tha. N c dng:
n =0

c n (z a ) n = c1 (z a ) n + c 2 (z a ) n + L + c n (z a ) n + L

(10)

Trong cn (n = 0, 1, 2,...) v a l nhng hng s phc, a c gi l tm ca chui. Bng cch i bin = z - a, chui (1) c dng:
n =0

c n n = c1 n + c 2 n + L + c n n + L

(11)

c tm ti = 0. 2. nh l Abel: Nu chui lu tha (11) hi t ti o 0 th n hi t tuyt i trong trong hnh trn | | < o. Trong mi hnh trn | | < , (11) hi t u. Chng minh: Ly l mt s dng bt k < | o | ta s chng minh trong hnh trn || th chui (11) th nhn mt chui tri hi t. Tht vy, theo gi thit, chui
n =0 n hi t. Do cn o
n n lim c n o = 0 . Dy s {c n o } c gii hn. Vy n b chn, ngha n

l tn ti s M > 0 sao cho: n cn o M n nguyn dng T (12) suy ra rng vi bt k no trong hnh trn kn | | ta c:
n cn n = cn o

(12)
M o
n

n
n o

n = cn o

72

iu chng t rng chui


M

n =0

c n n tha nhn mt chui dng tri l chui


< 1. o

n =0

. Chui dng ny l mt cp s nhn hi t v cng bi l

Vy theo tiu chun Weierstrass, chui (11) hi t tuyt i v u trong mt trn | | . V s c th chn gn | o | bao nhiu cng c nn (11) hi t tuyt i ti mi im ca hnh trn m | | < o. 3. H qu: Nu chui (11) phn k ti 1 th n phn k ti mi im ca min || < | 1 |. Chng minh: ta chng minh bng phn chng. Gi s chui (11) hi t ti o thuc min | | > | 1 |. p dng nh l Abel suy ra chui hi t trong hnh trn | | < | o | , c bit chui hi t ti 1 v | 1 | < | o |. iu ny mu thun vi gi thit. 4. Bn knh hi t: Trc ht ch l im = 0 bao gi cng l im hi t ca chui (11). Ti chui hm tng l co. y By gi ta xt tia Ot bt k, xut pht t gc = 0. C th xy ra 3 trng hp: t * Trn tia Ot c c nhng im hi t v nhng im phn k. V theo nh l Abel, mi im hi t u x O nm gn gc hn mt im phn k bt k. Do G C trn tia Ot tm c mt im * ngn cch nhng dim hi t trn tia vi nhng im phn k. Bn thn * , tu trng hp, c th l im hi t hay phn k. Cng theo nh l Abel, chui hi t trong hnh trn G: | | < | * | v phn k bn ngoi tc trong min | | < | * |. Hnh trn G c gi l hnh trn hi t ca chui hm (11), bn knh ca n R = | * | c gi l bn knh hi t. Trn bin C ca hnh trn c th c c im hi t ln phn k. * Trn tia Ot, tt c cc im u l im hi t. Khi , theo nh l Abel, chui hm hi t trong mt hnh trn bn knh ln tu . Ngha l n hi t trong ton mt phng v ta ni rng bn knh hi t l . * Trn tia Ot khng c im no l im hi t tr = 0. Khi theo h qu ca nh l Abel, chui hm phn k bn ngoi mt hnh trn m bn knh c n nh tu . Ni cch khc, mi im c khc 0 u l im phn k v ta ni bn knh hi t R = 0. Lp lun tng t gii tch thc, da vo tiu chun DAlembert hay Cauchy, ta thy bn knh hi t c th tm theo cng thc: c R = lim n (13) n c n +1
73

hay: R = lim

1 cn
n =0

n n

(14)

Ghi ch: i vi chui

c n (z a ) n bng php i bin = z - a ta a c v dng


n =0

n =0

cn n nn ta suy ra hoc chui

c n (z a ) n

ch hi t ti tm z = a, hoc hi t

trong c mt phng hoc hi t trong hnh trn | z - a | < R v phn k bn ngoi hnh trn .
V d 1: Xt chui
n =0

zn = 1 + z + z2 + L + zn + L .

Ta tnh bn knh hi t R ca n bng cng thc (13): c R = lim n = 1 v cn = cn+1 = 1 n c n +1 Vy chui hi t tuyt i trong hnh trn | z | < 1. Trong hnh trn | z | 1, chui hi t u. Ta xt tng ring: 1 zn 2 n 1 Sn ( z ) = 1 + z + z + L + z = 1 z 1 Cho n , nu | z | < 1 th lim z n = 0 . Vy lim Sn (z) = . Nh vy: n 1 z n 1 zn = 1 + z + z2 + L + zn + L = 1 z ; | z | < 1 n =0 V d 2: Xt s hi t ca chui hm: (z 1) n z 1 (z 1) 2 1 = + + +L n! 1 2 n =0 Bn knh hi t ca chui cho bng: 1 c (n + 1)! R = lim n = lim n! = lim = lim(n + 1) = n n n n c 1 n! n +1 (n + 1)! Vy chui hi t trong ton mt phng phc. (z j) 2 n +1 V d 3: Tm hnh trn hi t ca chui n 0 ( n + 1) 4 p dng tiu chun DAlembert cho chui moun cc s hng ta c: 2 n +3 2 2 z j z j z j (n + 1) (n + 1)4 n u n +1 lim = = = lim d = lim 2 n +1 n u n ( n + 2) 4 n +1 n ( n + 2) 4 4 z j n
Nh vy min hi t ca chui l | z - j |2 < 4 hay | z - j | < 2 .

74

3. CHUI TAYLOR

Gi s chui lu tha

n =0

cn (z a )n c bn knh hi t l R = 0. Theo kt qu

trn, trong hnh trn bn knh | z - a | < R th chui hi t u. V mi s hng ca chui hng ca chui u l hm gii tch v v chui hi t u nn theo nh l Weierstrass tng f(z) ca chui l mt hm gii tch trong min | z - a | . By gi ta t vn ngc li: cho trc mt hm f(z) gii tch trong mt ln cn im a. Hi c th khai trin n thnh chui lu tha ca (z - a) hay khng. Ni khc i, c th tm thy chui dng
n =0

cn (z a )n

c tng l f(z) trong ln cn a hay khng?

nh l 1: Mi hm f(z) gii tch trong hnh trn | z - a | < R u c th khai trin mt cch duy nht thnh chui lu tha ca (z - a). Chng minh: ly z bt k thuc hnh trn | z - a | < C R. Ta v hnh trn C = {| z - a | = } ( < R) bao im z bn trong. Gi C l ng trn | z - a | = R, C l ng trn | z - a | = . Theo cng thc tch a L phn Cauchy ta c: C 1 f ( ) d (16) f (z ) = z 2 j C ' Ta s tm cch khai trin hm s di du tch phn thnh chui lu tha ca (z - a) hi t u i vi bin trn ng trn C. Mun vy ta vit: 1 1 1 = = z a (z a ) za ( a ) 1 a

Nhng v C nn | z - a | < | - a | v mt chui nhn (xem cng thc 15) ta c:


2

za < 1 . Vy theo cng thc tnh tng ca a


n

1 za za za = 1+ + +L+ +L za a a a 1 a
1 1 1 za 1 za 1 za Vy: = + + +L+ +L z a aa aa aa f () f () f () z a f () z a f () z a = + + +L+ + L (17) z a aa aa aa
75
2 n 2 n

Vi z c nh, khi bin thin trn ng C th

f () l mt hm lin tc i vi . z za za f () za M . Mt khc Vy n b chn, tc = = = q <1 z a a

nn:

f () z a Mq n aa Theo tiu chun Weierstrass, chui bn v phi ca (17) hi t u vi tng C. Vy ta c th tch phn tng s hng dc theo C v c:
f () z

f ( )d f ( )d f () z a f () z a z = a + a a d + a a d C' C' C' C' n f () z a +L+ d + L a a C' f ( )d f () f () d + ( z a ) 2 d = + (z a ) 2 3 C' a C' ( a ) C' ( a ) f () + L + (z a )n d + L n +1 ( a ) C' Thay vo (16) ta c: 1 f ( )d ( z a ) f ( ) (z a )2 f () f (z ) = + d + d 2 2j C' a 2j C ' ( a ) 2j C ' ( a )3 (z a )n f () d + L +L+ 2j C ' ( a ) n +1 Nh vy ti mi im z thuc hnh trn | z - a | < R ta c th vit: f(z) = co + c1(z - a) + c2(z - a)2 + + cn(z - a)n + 1 f ( )d vi c n = , n = 0,1, 2,... 2 j C ' ( a ) n + 1 Theo cng thc tnh o hm cp n ca hm gii tch ta c: n! f ( )d f ( n ) (a ) = , n = 0,1, 2,... 2 j C ' ( a ) n + 1 nn ta c: f ( n ) (a ) cn = n! Tm li ta khai trin c f(z) thnh chui lu tha ca (z - a): f ( n ) (a ) f ( )d 1 n f (z ) = (z a ) = ( z a ) n n +1 n! n =0 n = 0 2 j C ' ( a )

(18)

76

Chui lu tha trn c gi l chui Taylor ca hm f(z) ti z = a. Nu f(z) gii tch trong hnh trn | z - a | < R th n khai trin c thnh chui Taylor ti im a. By gi ta cn phi chng minh tnh duy nht ca khai trin. Gi s f(z) c khai trin thnh chui lu tha:

f (z) = cn (z a )n , z a < R
n =0

(19)

f ( n ) (a ) ta s chng minh rng c n = n! Tht vy,trong (18) cho z = a, ta c co = f(a). o hm tng s hng ta s c:

f (z ) = nc n (z a ) n 1
n =0

Nh vy f(a) = c1. Tip tc o hm ri thay z = a vo 2 v, ln lt ta c: f(a) = 2c2, f(a) = 3!c3,..., f(n)(a) = n!cn. f ( n ) (a ) Vy : c n = n! Ngha l chui (19) ng l chui Taylor ca hm f(z). l iu phi chng minh. Ghi ch: Trong (18) ta vit: 1 f ( ) d cn = 2 j C ' ( a ) n + 1 Gi s L l ng cong kn bt k nm hon ton trong min | z - a | < R. V hm f (z ) gii tch trong min nh lin c bin l L v C nn theo nh l Cauchy ta ( z a ) n +1 c: f ( ) d f ( ) d ( a ) n +1 = ( a ) n + 1 C' L Vy ta c th vit: 1 f ( ) d cn = n +1 2j L ( a ) Ta s khai trin Taylor mt s hm s cp c bn. Trc ht ta lp chui Taylor ca hm f(z) = ez ti z = 0: f ( n ) ( 0) 1 Ta c f(z) = ez, f(n)(0) = 1; = n! n! 2 n z z z Vy: e z = 1 + + + L + + L 1! 2! n! Cng thc trn c ngha z C v ez gii tch trong ton b C. Tng t ta c:

77

z3 z5 z7 sin z = z + + L ( R = ) 3! n! 7! z2 z4 z6 cos z = 1 + + L ( R = ) 2! 4! 6! z2 z3 z4 ln(z + 1) = z + + L ( R = 1) 2 3 4 3 z z5 z7 arctgz = z + + L ( R = 1) 3 5 7

4. CHUI LAURENT Trong mc trc ta thy rng nu f(z) gii tch trong mt hnh trn tm a, th n khai trin c thnh chui Taylor ti a. By gi ta gi thit rng f(z) gii tch trong mt ln cn im a, tr ti z = a hay tng qut hn f(z) gii tch trong mt hnh vnh khn tm a. y 1. nh l Laurent: Gi s f(z) l mt hm gii tch n tr trong hnh vnh khn G: z r<|z-a|<R a Khi ta c z G: 2 n f (z) = c o + c1 (z a ) + c 2 (z a ) + L + c n (z a ) + L G c c 2 c n + 1 + +L+ +L 2 z a (z a ) (z a ) n O x (22) Trong cc h s cn c tnh theo cng thc:
cn = 1 f( )d 2 j L ( a ) n+1 n = 0 , 1, 2 ,...

(23)

L l mt ng cong kn bt k bao im a v nm trn trong hnh vnh khn. Chui bn phi hi t u ti f(z) trong mi hnh vnh khn kn : r | z - a | R (r>R, R<R) v c gi l chui Laurent ca hm f(z) vi tm ti a. Chng minh: Ly z bt k thuc G. Bao gi ta cng v c 2 ng trn: L1 : | z - a | = r L2 : | z - a | = R m r < r < R < R sao cho z thuc hnh vnh khn Go: r < | z - a | < R. V f(z( gii tch trong Go nn p dng cng thc tch phn Cauchy cho min nh lin Go m bin ngoi l L2 v bin trong l L1 ta c: 1 f ( )d 1 f ( ) d f (z) = (24) n +1 2j L2 ( a ) 2j L1 ( a ) n +1 Tch phn th nht trong (24) l mt hm gii tch bn trong ng trn ln L2. Ta s tm cch khai trin n theo chui lu tha ca (z - a). Tch phn th hai l mt hm gii tch bn ngoi hnh trn nh v dn ti 0 khi z . Ta s tm cch khai trin

78

n theo chui lu tha ca

Khi L2 th: 1 1 = = z a (z a ) V:

1 . za
1 za ( a )1 a

za < 1 nn: a
za 1 = z a n =0 a 1 a n

1 1 za Vy: = z a n =0 a

1 f ( ) 1 f ( ) z a f ( ) ( z a ) n = = 2j z 2j a n =0 a 2j n =0 ( a ) n +1 Lp lun tng t ta thy chui bn v phi hi t u vi L2. Vy c th tich phn tng s hng dc theo L2: 1 f ( )d 1 f ( )d n = (z a ) . 2j L2 a 2j L2 ( a ) n +1 n =0 Nu t: 1 f ( ) d cn = , n = 0, 1, 2 ,.... (25) 2j L2 ( a ) n +1 th ta c: 1 f ( )d n = c n (z a ) (26) 2j L2 a n =0

Ch l khng c vit: c n =

f ( n ) (z) v trong gi thit ca nh l khng ni g ti n!

tnh gii tch ca f(z) ti a. za > 1 ; khi ta c th lm nh sau: Khi L1 th a a < 1 nn: Hin nhin ta c za
1 1 1 = = = z a ( z a ) ( z a ) ( a )
1 a = a ( z a ) k =0 z a (z a )1 za

79

f ( ) f ( ) ( a ) k == 2j( z) 2j k =0 (z a ) k +1 Chui bn phi hi t u i vi L1. Vy ta c th tch phn tng s hng dc theo L1: 1 f ( )d 1 1 = ( a ) k +1 f ( )d k +1 2j L1 a 2j k =0 (z a ) L2 Trong v phi, i k hiu ca ch s chy bng cch t k + 1 = -n. Khi k = 0, 1, 2,.. th n = -1, -2,...Vy: 1 f ( ) d 1 (z a )n . f ()d = n +1 2j L1 a 2j n =1 L 2 ( a ) Nu t: 1 f ( )d , n = -1, -2 ,... (27) cn = 2j L1 ( a ) n +1 1 f ( )d n (28) = c n (z a ) th: 2j L1 a n = 1 Thay cc kt qu vo (24) ta c:

f (z) = c n (z a ) +
n n =0

n = 1

c n (z a )

n =

c n (z a )

(29)

1 f ( )d nu n = 0, 1, 2 ,... 2j L 2 ( a ) n +1 1 f ( )d cn = nu n = -1, -2 ,... 2j L1 ( a ) n +1 Nu gi L l mt ng cong kn bt k bao im a v nm gn trong vnh khn G th trong biu thc tnh cn c th thay tch phn dc theo ng L1 v L2 bi tch phn dc theo L, ngha l: 1 f ( ) d n = 0, 1, 2, 3,... (30) cn = 2j L ( a ) n +1 Tm li ta chng minh c rng vi z bt k thuc G ta c khai trin (29) vi cn tnh theo (30). cn = Trong khai trin Laurent chui
n =0

Vi:

c n (z a )

gm cc lu tha dng ca (z - a),

c gi l tch phn u ca chui Laurent v chui

n =1

c n (z a )

gm cc lu tha

nguyn m c gi l phn chnh . Nh vy chui Laurent c th xem l tng ca hai chui phn u v phn chnh. Theo nh l Abel, phn u hi t bn trong hnh trn ln | z - a | < R, v hi t u trong hnh trn kn | z -a | R (R bt k nh hn R). Tng t, phn chnh hi t
80

phn chnh hi t bn ngoi vng trn nh t l trong min | z - a | > r v hi t u trong min | z - a | r (r bt k ln hn r) Mun chng minh tnh duy nht ca khai trin Laurent ta lm tng t nh khi chng minh tnh duy nht ca khai trin Taylor. 2. Ghi ch: Nu hnh trn nh | z - a | r khng cha im bt thng ca f(z), ngha f (z) l nu f(z) gii tch trong hn trn ln | z - a | < R th (n = -1, -2, -3,...) cng (z a ) n +1 gii tch trong hnh trn . Vy theo nh l Cauchy: 1 f ( ) d cn = = 0 , n = -1, -2, -3,... 2j L ( a ) n +1 Phn chnh s trit tiu v khai trin Laurent tr thnh khai trin Taylor. Ni khc i, khia trin Taylor l trng hp ring ca khai trin Laurent. 3. Mt s phng php khai trin thnh chui Laurent: Trong mt s trng hp ta c th dng nhng phng php khai trin thnh chui Laurent n gin hn l p dng cng thc (23). Chng hn, nu f(z) gii tch trong min r < | z -a | < R, c th vit c di dng tng ca hai hm : f(z) = f1(z) + f2(z) hay di dng tch ca 2 hm: f(z) = f1(z).f2(z) trong f1(z) gii tch trong hnh trn ln | z - a | < R, cn f2(z) gii tch bn ngoi hnh trn nh, tc trong min | z - a | > r, th ta tm cch khai trin f1(z) thnh chui lu tha i vi (z - a) v khai trin f2(z) thnh chui lu tha i vi (z - a)-1. Cng c th da vo cc khai trin Taylor ca cc hm s cp nh ez, cosz, sinz... khai trin mt s hm siu vit thnh chui Laurent. V d 1: Khai trin hm : y 1 f (z) = (z 1)(z 2) thnh chui Laurent tm ti 1 trong cc min sau: - hnh trn b tm 0 < | z - 1 | < 1 - min ngoi hnh trn trn 2 x 0 Vi hnh trn b tm 0 < | z - 1 | < 1ta vit: 1 1 f (z) = z 2 z 1 1 V hm f 2 ( z) = gii tch khp ni tr ti z 1 z = 1. Bn thn hm f2(z) l mt lu tha ca (z - 1) nn ch cn khai trin f1(z). V trong min | z -1 | < 1, hm f1(z) gii tch nn n khai trin c thnh chui Taylor ti z = 1.

81

f1 ( z ) =

1 1 1 = = z 2 z 1 1 1 (z 1)

= 1 + (z 1) + (z 1) 2 + L + (z 1) n + L Vy trong min 0 < | z -1 | < 1 ta c: 1 f (z) = 1 ( z 1) ( z 1) 2 L ( z 1) n L z 1 By gi ta tm khai trin trong hnh trn | z -1| > 1. Trong min ny ta c: 1 1 1 = = f1 (z) = 1 z 2 z 11 (z 1) 1 z 1 1 V < 1 nn ta c khai trin : z 1 1 1 1 = 1+ + +L 2 1 z 1 ( z 1 ) 1 z 1 Vy: 1 1 1 1 1 1 1 1+ = + + L = + +L+ +L 2 2 z 2 z 1 z 1 (z 1) (z 1) 2 z 1 (z 1)

f (z) =

1 1 1 1 1 = + +L+ +L 2 3 z 2 z 1 (z 1) (z 1) (z 1) n 1 theo cc lu tha ca z khi z (z 1)(z 2)

V d 2: Vit khai trin ca hm f (z) =

thuc cc min sau: - hnh trn | z | < 1 - hnh vnh khn 1 < | z | < 2 - min ngoi hnh trn tm O, bn knh 2 : | z | > 2 Trong hnh trn | z | < 1, hm f(z) gii tch, vy n khai trin c thnh chui Taylor. Ta phn tch F(z) ri vit khai trin cho tng s hng. 1 1 1 1 z z 2 z3 = = = 1 + + 2 + 3 + L z2 2 2 2 2 z z 2 1 21 2 2
1 1 = = 1 + z + z 2 + z3 + L z 1 1 z

Vy:

82

1 z z 2 z3 f (z) = 1 + + 2 + 3 + L + 1 + z + z 2 + z 3 + L 2 2 2 2

1 1 1 1 + 1 2 z + 1 3 z 2 + L + 1 n +1 z n + L 2 2 2 2 1 Xt trong min 1 < | z | < 2. V hm gii tch trong min | z | < 2 nn n khai z2 trin c thnh chui Taylor i vi z: 1 1 z z 2 z3 = 1 + + 2 + 3 + L z2 2 2 2 2 1 Cn hm gii tch bn ngoi hnh trn on v nn ta tm cch khai trin n theo 1 z 1 chui lu tha ca . Ta c: z 1 1 1 = = 1 z 1 1 z 1 z 1 z z 1 < 1 nn: V y z 1 1 1 = 1+ + 2 +L 1 z z 1 z 1 1 1 1 1 1 1 = 1 + + 2 + L = 2 3 L Vy: 1 z z z z z z z
Hay: f (z) =

1 1 1 1 1 z z 2 z3 1 f (z) = = 1 + + 2 + 3 + L + 2 + L + n + L z 1 z 1 2 2 2 2 z z z 1 z z2 zn 1 1 1 = 2 3 L n +1 L 2 L n L 2 2 2 2 z z z Xt trong min | z | > 2. Ta phi khai trin hai hm s tha ca


1 : z 1 1 1 1 = 2 3 L 1 z z z z 1 1 v theo chui lu z2 z 1

83

1 1 = z2 2 z 1 z 2 1 2 22 < 1 nn: = 1+ + 2 +L V y 2 z z z 1 z 2 1 1 2 2 2 n 1 = + + +L+ n +L Vy: z 2 z z2 z3 z

1 1 1 22 1 2 n 1 1 = + 3 +L+ +L f (z) = z 1 z 1 z2 z zn
z V d 3: Khai trin hm s f (z) = sin thnh chui Laurent tm ti 1. z 1 Ta vit: 1 1 1 z f (z) = sin + cos1. sin = sin1 + = sin 1. cos z 1 z 1 z 1 z 1 Da vo khai trin ca sinz v cosz ta c: 1 1 1 1 sin = + L 3 z 1 z 1 3!(z 1) 5!(z 1) 5 1 1 1 cos = 1 + L 2 z 1 2!(z 1) 4!(z 1) 4 Hai khai trin trn ng z 1. Vy: cos1 sin 1 cos1 z +L+ sin = sin 1 + 2 z 1 2!(z 1) 3!(z 1) 3 z 1 sin 1 sin 1 + (1) n +L (1) n 2n (2n + 1)!(z 1) 2 n +1 2n!(z 1) V d 4: Khai trin thnh chui Fourrier hm s: a sin t ( t ) = (| a | < 1) , t l bin s thc 1 2a cos t + a 2 Theo cng thc Euler: e jt + e jt e jt e jt cos t = ; sin t = 2 2 Thay vo biu thc ca (t) ta c: 1 e 2 jt ( t ) = 1 2 j e 2 jt a + e jt + 1 a Xt hm:
84

1 z2 f (z) = 1 2 j z 2 a + z + 1 a jt Hin nhin f(e ) = (t). Vy (t) l gi tr ca hm f(z) trn ng trn n v z = ejt. D thy rng hm f(z) gii tch trong mt hnh vnh khn tm O, cha ng trn n v | z | = 1. Ta s khai trin f(z) thnh chui Laurent trong hnh vnh khn ny. Trc ht ta phn tch f(z) thnh tng cc phn thc n gin: 1 1 1 f (z) = 1+ + z 1 az 2j 1 a Ch rng vi | az | < 1 ta c: 1 = 1 + az + (az) 2 + (az) 3 + L 1 az cn vi | z | > | a | ta c: 1 1 a a a2 a3 1 + + 2 + 3 + L = = z z a z z z z 1 1 a a z 1 Vy trong min a < z < ta c: a 1 n n 1 1 a a2 a3 2 2 3 3 f (z) = az a z a z a z n + + + L L = 2j 2j z z 2 z3 z n =1 jt Khi z = z , | z | = 1, ta c: 1 e njt e njt n ( t ) = a n (e njt e njt ) = a n = a sin(nt ) 2 j n =1 2j n =1 n =1 l khai trin Fourrier cn tm.
5. IM BT THNG CA HM GII TCH 1. Phn loi: Gi s a l im bt thng c lp ca hm f(z), ngha l tn ti mt ln cn kh b ca a trong ch c a l im bt thng. Nh vy f(z) s gii tch trong hnh vnh khn nh tm a. Theo mc 5, ta c th khai trin f(z) thnh chui Laurent trong hnh vnh khn ny. Ta cn c vo khai trin Laurent phn loi tnh bt thng ca im a. Nu khai trin Laurent khng cha phn chnh tc l cn = 0 n < 0. Do : (31) f(z)= co + c1(z - a) + c2(z - a)2 + th im a c gi l im bt thng b c. Nu a l im bt thng b c, th theo (3) ta c: lim f (a ) = c o
za

85

Do nu t f(a) = co th hm f(z) c b sung gi tr ti im a. Nh vy n s l mt hm gii tch trong c ln cn ni trn ca a. iu gii thch ngha ca thut ng b c c dng y. Nu trong phn chnh ch c mt s hu hn cc s hng th a c gi l cc im. Khi khai trin c dng: c n c n +1 c (32) f (z) = + + L + 1 + c o + c1 (z a ) + L n n 1 (z a ) (z a ) (z a ) Trong c-n 0. S m n c gi l cp ca cc im. Nu a l cc im th t (32) suy ra: lim f (z) =
za

Nu phn chnh ca khai trin c v s s hng th ta gi a l im bt thng ct yu ca f(z). i vi im bt thng ct yu ta c nh l Xakhtxki: Nu a l im bt thng ct yu ca f(z) th vi mi s A cho trc, lun lun tn ti mt dy {zk} dn ti im a sao cho dy {f(z)} dn ti A.
sin z . N tha nhn im z = 0 lm im bt thng c lp. z Khai trin f(z) theo lu tha ca z ta c: sin z 1 z3 z5 z2 z4 = z + L = 1 3! + 5! + L z z 3! 5! Vy im z = 0 l im bt thng b c ca hm. Nu ta b sung nh sau: sin z khi z 0 f (z) = z khi z = 0 1 th f(z) gii tch c ti z = 0. ez V d 2: Hm f (z) = 3 tha nhn im z = 0 lm im bt thng c lp. Khai trin z theo lu tha ca z ta c: 1 1 ez 1 z z 2 z3 1 1 z L = + + = 1 + + + + z 3 z 2 2!z + 3! + 4! + L z3 z3 1! 2! 3! T suy ra im z = 0 l cc im cp 3 ca f(z).

V d 1: Xt hm f ( z ) =

V d 3: Xt hm f (z) = e . im z = 0 l in bt thng ct yu ca hm v:

1 z

1 1 1 1 + + +L+ +L 2 3 1! z 2! z 3! z n! z n f (z) 2. nh l: Gi s f (z) = 1 trong f1(z) v f2(z) l cc hm gii tch ti a. Nu f 2 (z) im a khng phi l khng im ca t s, tc f1(a) 0 v l khng im cp m ca mu s, th a l cc im cp m ca f(z). f (z) = e = 1 +
86

1 z

Chng minh: theo gi thit ta c f2(z) = (z - a)m(z) vi (z) gii tch ti a v (a) = 0. f (z) Hm 1 gii tch ti a nn c th khai trin n thnh chui Taylor ln cn im a (z) f1 ( z ) f (a ) = b o + b1 (z a ) + b 2 (z a ) 2 + L vi b o = 1 0 (z) (a ) T suy ra khai trin Laurent ca f(z) l: bo f1 ( z ) b1 f (z) = = + +L (z a ) m (z0 (z a ) m (z a ) m1 iu chng t a l cc im cp m ca f(z) z +1 V d: Xt hm f ( z) = 2 z +4 2 V z + 4 = ( z + 2j )( z - 2j ) nn mu s c hai khng im n l z = 2j. Vy f(z) phi c hai cc im n l z = 2j. sin z V d: Xt hm f (z) = 2 (z + 1) 3 V ( z2 + 1)3 = ( z2 +j )( z2 - j )3 nn z = j l nhng khng im cp 3 ca mu s. V vy z = j l nhng cc im cp 3 ca mu s.

87

CHNG 5: L THUYT THNG D


1. KHI NIM V THNG D 1. nh ngha thng d: Gi s f(z) l mt hm gii tch trong mt ln cn ca im a tr chnh im a (ngha l a l im bt thng c lp ca f(z)). Nu C l ng cong kn bt k bao ly im a v nm trong ln cn ni trn th theo nh l Cauchy, tch phn f (z)dz l mt s khng ph thuc C. Ta gi thng d ca hm f(z) ti a l
C

kt qu php chia f (z)dz cho 2j. Thng d c k hiu l Res[f(z), a]. Tm li:
C

1 f (z)dz 2j C 1 2j 1 1 , a = dz = =1 V d: Res z a 2 j z a 2 j C
Res[f(z), a] =

(1)

2. Cch tnh thng d: Cng thc chung tnh thng d l: (2) Res[f(z), a] = c-1 1 Trong c-1 l h s ca trong khai trin Laurent ca hm f(z) ti ln cn im za a. Chng minh: Theo cng thc tnh h s ca khai trin Laurent: 1 f ( ) d cn = 2j C ( a ) n +1 Khi n = -1 ta c: 1 c 1 = f ()d = Res[f(z), a] 2j C a. Thng d ti cc im n: Nu a l cc im n ca hm f(z) th : (3) Res[f(z), a] = lim[(z a )f (z)]
za

z2 V d 1: V z = 2 l cc im n ca nn z2 z2 = lim z 2 = 4 Res[f(z), a] = lim (z 2) z 2 z 2 z 2 1 . Tnh thng d ti a = 0 V d 2: Cho f (z) = sin z Ta bit : z2 z4 z3 z5 sin z = z + L = z 1 + L 3! 5! 3! 5!

88

Cn c vo khai trin ny ta thy im z = 0 l khng im n ca sinz. vy im z 1 . Theo (3) ta c: = 0 l cc im n ca f (z) = sin z 1 =1 Res[f(z), a] = lim z z0 sin z f (z) nh l: Gi s f (z) = 1 , trong f1(z) v f2(z) l nhng hm gii tch ti a. im f 2 (z) a l khng im n ca f2(z0 v khng phi l khng im ca f1(z). Khi : f (a ) (4) Res[f(z), a] = 1 (a ) f2 Chng minh: Theo gi thit ta thy a l cc im n ca f(z). Theo (3) ta c: f1 ( z ) f1 ( z ) = lim Res[f(z), a] = lim (z a ) z a f 2 (z) z a f 2 ( z ) (z a ) V f2(a) = 0 nn ta c th vit: lim f1 (z) f (a ) z a = 1 Res[f(z), a] = f ( z ) f 2 (a ) f 2 (a ) lim 2 za (z a ) V d 3: Tnh thng d ca f(z) = cotgz V a = 0 l n ca cotgz nn theo (4) ta c: f (a ) cos 0 = =1 Res[f(z), a] = 1 (a ) cos 0 f2 z +1 V d 4: Tnh thng d ca hm f ( z) = 2 ti a = 2j. z +4 V 2j l khng im n ca (z2 + 4) nn n l cc im n ca f(z). Theo (4) ta c: f (a ) 2 j + 1 1 1 = = j Res[f(z), a] = 1 (a ) f2 4j 2 4
ez ti a = j V d 5: Tnh thng d ca hm f (z) = (z j)(z + j) Ta thy f(z) c hai cc im n l j. p dng cng thc (4) ta c: ez ej j Res[f(z), j] = lim = = (cos1 + j sin 1) z j z + j 2j 2 z j e e j Res[f(z), -j] = lim = = (cos1 j sin 1) z j z j 2j 2 b. Thng d ti cc im cp m: Nu a l cc im cp m ca f(z) th:

89

1 d m1 Res[f(z),a] = (5) lim m1 (z a ) m f (z) (m 1)! za dz 1 ti a = j V d 1: Tnh thng d ca hm f (z) = 2 (z + 1) 3 V (z2 + 1)3 = (z + j)3(z - j)3 nn j l khng im cp 3 ca (z2 + 1)3. Vy j l cc im cp 3 ca hm f(z). Theo (5) vi m = 3 ta c: 1 d2 1 1 d2 1 3 = lim 2 2 Res[f(z), a] = lim 2 (z j) 2 3 2! z j dz (z + 1)3 2 z j dz (z + j) 1 12 6 3j = lim 2 = = 5 5 2! z j (z + j) (2 j) 16 z e V d 2: Tm thng d ca hm f (z) = 3 z Ta thy z = 0 l khng im cp 3 ca z3 nn z = 0 l cc im cp 3 ca hm f(z). Dng cng thc (5) ta c: d 2e z 1 1 Res[f(z), a] = lim = 2 2! z0 dz 2

2. NG DNG THNG D 1. nh l 1: Nu f(z) gii tch trong min G , gii hn bi ng cong kn L, ngoi tr ti mt s hu hn cc m a1, a2, ..,as bn trong th:

f (z)dz = 2j Re s[f (z), a k ]


L k =1

(8)

Chng minh: Loi i khi min G cc hnh trn 1, 2,...,s c tm ln lt l a1, a2, ..,as v c bn knh nh ta c mt min a lin . p dng nh l Cauchy cho min a lin ny ta c: 1 1 1 f (z)dz = f (z)dz + L + f (z)dz 2j L 2j 1 2j s Nhng v: 1 f (z)dz = [ Resf(z), ak], k = 1, 2,..., s 2j k nn thay vo ta c: f (z)dz = 2jRes[f(z), ak] + + 2jRes[f(z), ak]
L

2. nh l 2: Nu f(z) gii tch trong ton b mt phng ngoi tr ti mt s hu hn cc m a1, a2, ..,as = th:

Re s[f (z), a k ] + Re s[f (z), a k ] = 0


k =1

Chng minh: Chn R ln ng trn | z | = R bao ly tt c cc im a1, a2,.., an, Ta c:


90

k =1

Re s[f (z ), a k ] = 2j f (z )dz
C

Theo nh ngha thng d ti : 1 1 f ( z )dz = Res[f(z), ] = f (z )dz 2 j C 2 j C Cng cc v ca hai ng thc ny ta c iu cn phi chng minh. z 2dz V d 1: Tnh 2 , L l ng trn tm | z | = 2 ( z + 1 )( z + 3 ) L z2 c 3 cc im l z = j, z = -j v z = -3. Hm f (z ) = 2 (z + 1)( z + 3) Trong hnh trn | z | < 2 c hai cc im l j, u l cc cc im n. Tnh thng d ti cc cc im ta c: z2 j2 j 1 + 3j = = = Res[f(z), j] = lim(z j)f (z ) = lim z j z j ( z + j) + ( z + 3) 2 j(z + 3) 2z + 6 20 z2 z2 j2 1 3j f1 ( j) z + 3 Res[f(z), -j] = = = = = 2z 2z ( z + 3) z = j 2 j(3 j) 20 f 2 ( j)
z= j

Vy

1 + 3 j 1 3 j j + I = Res[f(z), j] + Res[f(z) , -j] = 2j = 20 5 20 cos zdz V d 2: Tnh I = 2 , L l ng trn | z | = 2 L z ( z 2) cos z c z = 0 l cc im cp 2 v im z = 2 l cc im cp 1. Hm f ( z ) = 2 z ( z 2) Trong hnh trn | z | < 1 ch c mt cc im z = 0 nn: I = 2j.Res[f(z), 0] Nhng v: cos z 2 cos z sin z ( z 2) cos z 1 cos z = lim = lim z 2 = = lim Res 2 2 ( z 2) 4 z ( z 2 ) z 0 z ( z 2) z 0 z 2 z 0 j nn I = 2 e z dz V d 3: Tnh I = 2 vi C l ng trn | z | = 3 C z +1 Hm f(z) di du tch phn c hai im bt thng j v -j nm trong hnh trn bin C. Theo v d mc trc ta c:

91

Res[f(z), j] = Nn: I = 2jsin1

ej e j v Res[f(z), -j] = 2j 2j

z+3 dz vi C l ng trn | z - 0.5 | =1 2 + ( z 1 )( z 1 ) C Trong min gii hn bi C, hm f(z) di du tch phn ch c mt im bt thng l z = 1, cc im n. Do : I = 2j.Res[f(z), 1] = lim(z 1)f ( z ) = 2j

V d 4: Tnh I =

z 1

3.Tch phn thc dng

R( x )dx trong R(x) l mt phn thc hu t

a. B 1: Gi s CR l mt na ng trn tm O, bn knh R, nm trong na mt phng trn Imz > 0. Nu f(z) gii tch trong y na mt phng trn, tr ti mt s hu hn im bt thng v tho mn: lim zf (z ) = 0 0 arg z CR z

th:
R

lim f ( z )dz = 0
CR

Chng minh: Phng trnh CR c dng z = Rej vi l tham s bin thin t 0 n . Chn R kh ln sao cho cc im bt thng ca f(z) u nm trong min | z | < R. Vy hm f(z) lin tc trn CR v theo cch tnh tch phn ta c:
CR

f (z )dz = f (Re
0

) Re j d
z

Ta c lng tch phn ny. V lim zf ( z ) = 0 nn > 0 cho trc ta lun tm c mt s N > 0 sao cho khi | z | > N th | z.f(z) | < . Vy nu z CR+ vi R > N th: | f(Rej).Rej | = | z.f(z) | < Do :
CR

f (z )dz

d =
0
R CR

V b tu nn ta suy ra lim f ( z )dz = 0


b. nh l 1: Gi s R(z) l mt phn thc m a thc mu s c bc ln hn a thc t s t nht l hai n v, R(z) c mt s hu hn cc im a1, a2,..., an nm trong na mt phng trn v khng c cc im nm trn trc thc. Khi ta c:
+

Re s[ R (z ), a k ] R ( x )dx = 2jk =1

(9)

Ta tha nhn m khng chng minh nh l ny.


92

dx 4 0 x +1 V hm di du tch phn l chn nn ta c: dx 1 dx = 4 Ta c: I = 4 2 x + 1 0 x +1 1 t R ( z ) = 4 . Phng trnh z4 + 1 = 0 c hai nghim trong na mt phng trn z +1 l: 2 2 2 2 + j , z2 = j z1 = . R rng R(z) iu kin p dng (9). Ta c 2 2 2 2 2 z1 1 j 2 1 z2 = + = I = j Re s[R (z ), a k ] = j j ( z z ) = + + 1 2 4z 4 4 z 4 4z 3 4 z 3 4 4 k =1 1 1 2 2
V d 1:Tnh I =

x 1 dx 2 2 ( x + 1) z 1 z 1 tho mn cc gi thit ca nh l. Trong na Hm R ( z ) = 2 = ( z + 1) 2 ( z j) 2 ( z + j) 2 mt phng trn, n c cc im cp 2 l z = j. Theo (9); I = 2jRes[R(z), j] d d z 1 2 + j z = 2j lim = = 2j lim [(z j) 2 R (z )] = 2j lim 2 3 z j dz z j dz ( z + j) z j ( z + j) 2 x 2dx V d 3: Tnh I = 4 0 x +1 V hm di du tch phn l chn nn ta c: dx 1 dx = 4 Ta c: I = 4 2 x + 1 0 x +1 z2 t R ( z ) = 4 . Phng trnh z4 + 1 = 0 c hai nghim trong na mt phng trn z +1 l: 2 2 2 2 + j , z2 = j z1 = . R rng R(z) iu kin p dng (9). Ta c 2 2 2 2
V d 2: Tnh I =
+

I = j Re s[R (z ), a k ]
k =1

Res[R(z), j] =
Tng t:

z2 z2 1 = = = 4 3 ( z + 1) z = z1 4z z = z1 4z1

1 4 2 (1 + j) 2

1 1 j = 2 2 (1 + j) 4 2

93

Res[R(z), j] =

1 1 j = 2 2 (1 j) 4 2

2 1 j 1 j 2 Vy: I = j + = = 4 4 2 4 2 4 2 c. nh l 2: Gi s R(z) l mt phn thc hu t m bc ca mu s ln hn bc ca t s t nht 2 n v. Hm R(z) c cc cc im trong na mt phng trn l a1, a2,.., as v c m cc im n trn trc thc l b1, b2,..,bm. Khi ta c:
+

R ( x )dx = 2j Re s[R (z ), a k ] + j Re s[R (z ), bi ]


k =1 i =1 +

(11)

4. Tch phn dng

R ( x ) cos xdx v R( x ) sin xdx ( > 0)


j x

Theo cng thc Euler th e sinx=Im(ejx). Vy:


+ +

= cosx + jsinx nn cosx = Re(ejx) v

R ( x ) cos xdx = Re R ( x )e R ( x ) sin xdx = Im R ( x )e

jx

dx

jx

dx
+

Do mun tnh cc tch phn cho, ch cn tnh


+

R ( x )e

jx

dx ri ly phn thc

hay phn o ca n l c. Khi tnh

R ( x )e

jx

dx ta dng b sau:

a. B Jordan: Gi CR l cung trn | z | = R Imz > a (a l s thc c nh cho trc) ngha l CR l cung trn tm O, bn knh R v nm pha trn ng thng y=a. Nu F(z) c dng ejzf(z) trong l mt s dng c nh cn f(z) gii tch trong na mt phng Imz a , tr ti mt s hu hn im bt thng v tho mn lim f ( z ) = 0 th:
z
R

lim F( z )dz = lim e jz f ( z )dz


CR R CR

Ta tha nhn khng chng minh b ny b. nh l 1: Gi s R(z) l mt phn thc hu t tho mn cc iu kin sau: * R(z) gii tch trong na mt phng trn, tr ti mt s hu hn cc cc im a1, a2,.., as * R(z) khng c cc im trn trc thc * trong biu thc ca R(z), bc ca mu s ln hn bc ca t s t nht l 1 n v. Th th:
+ jx jx R ( x )e dx = 2j Re s[ R (z )e , a k ] k =1 s

(14)
94

Trong l mt s cho trc. Ta cng khng chng minh nh l ny. + x cos x dx V d 1: I = 2 x 2 x + 10 + xe jx dx Ta c: I = Re 2 x 2 x + 10 tnh I ta p dng (14). Mun vy ta phi tm cc cc im ca z . Gii phng trnh z2 - 2z + 10 = 0 ta c hai nghim l z = 1 3j. R (z) = 2 z 2z + 10 l hai cc im n ca R(z). Cc im z = 1 + 3j nm trong na mt phng trn. Dng cng thc (14) ta c: + (1 + 3 j)e 3+ j ze jz ze jz xe jx = 2j ,1 + 3 j = 2j dx = 2j. Re s 2 2 6j 2 z 2 z =1 + 3 j x 2 x + 10 z 2z + 10 = e 3 (cos1 3sin1) + j e 3 (3 cos1 + sin1) 3 3 T suy ra: I = e 3 (cos1 3sin 1) 3 c. nh l 2: Gi s R(z) l mt phn thc hu t tho mn cc iu kin sau: * R(z) gii tch trong na mt phng trn, tr ti mt s hu hn cc cc im a1, a2,.., as * R(z) c m cc im trn trc thc b1. b2,...,bn * trong biu thc ca R(z), bc ca mu s ln hn bc ca t s t nht l 1 n v. Th th vi l mt hng s dng cho trc :
+ jx jx jx R ( x )e dx = 2j Re s[ R (z )e , a k ] + j Re s[ R (z )e , bk ] k =1 k =1 s m

(16)

V d: Tnh I =

sin x dx x 0

sin x l hm chn nn ta c th vit c: x 1 + sin x I= dx 2 x Mt khc: + + jz sin x e x dx = Im z dz + jz 1 e Vy: I = Im dz 2 z


95

1 c cc im duy nht ti z = 0 nn theo (6) ta c: z + jz e jz e dz = j Re s e jz = j z z ,0 = j lim z 0 Thay vo trn ta c: I = 2

V hm R ( z ) =

2 0

5. Tch phn dng

f (sin t, cos t )dt

t z = ejt th lnz = jt, dt =

dz v theo nh ngha cc hm lng gic ta c: jz 1 1 z z+ z. z , sin t = cos t = 2 2j Khi t chy t 0 n 2, im z v nn ng trn C: | z | = 1. Vy: 2 1 1 1 jdz j z , z + (17) f (sin t, cos t )dt = f 2 z 2 z z 0 L Trong L l ng trn | z | = 1 2 2 + cos t dt V d 1: Tnh I = 2 sin t 0 Theo (17) ta c: 1 1 2 + z + 4z + z 2 + 1 jdz z 2 + 4z + 1 dz 2 z jdz = 2 I= = 2 1 z j L L 4z + jz j z L z 4 jz 1 z 2 + z z 2 Hm di du tch phn c 3 im cc l z = 0, z = 2 j j 3 . V
( 2 3 ) j = 2 3 < 1; ( 2 + 3 ) j = 2 + 3 > 1 nn bn trong L ch c 2 cc im l

a1 = 0 v a2 = 2 3 . ta tnh thng d: z 2 + 4z + 1 z 2 + 4z + 1 ,0 = lim 2 = 1 Res 2 z 0 z 4 jz 1 z ( z 4 jz 1 )


z 2 + 4z + 1 z 2 + 4z + 1 Res 2 , ( 2 3 ) j = z ( z 4 jz 1) z ( 2z 4 j) ( 2 Theo nh l 1 mc trc ta c: 2 3 4 3 2 3 + I = 2 j 1 + 1 j = 2j j 3 = 3 3 =1+ j
3) j

2 3 3

V d 2: Tnh I =

dt 0 2 + cos t
96

t z = ejt , v hm di du tch phn l chn nn ta c: 1 dt 1 dz 1 dt 1 dz = = 2 = I= 1 j C z + 4z + 1 j C ( z a )( z b) 2 2 + cos t 2 j C + + 2 z z z 2 Trong C l ng trn | z | = 1, a = 2 + 3 v b = 2 3 l cc nghim ca phng trnh z2 + 4z + 1 = 0. V | a | < 1 v | b | > 1 nn ta c: 1 2 = ,a = I = 2.Res 3 ( z a )( z b) a b

97

CHNG 6: PHP BIN I LAPLACE


1. PHNG PHP CA PHP TNH TON T Cho hai tp hp A v B. Mt nh x T cho ng mt phn t ca A vi mt phn t xc nh ca B, k hiu l Tx, c gi l mt ton t. Phn t Tx c gi l nh ca x cn x c gi l gc ca hay nghch nh ca Tx. V d: )Nu A = B = R th ton t T l mt hm s thc ca bin s thc. ) Nu A l tp hp cc s thc dng v B = R. nh x cho mi s a A thnh mt s thc thuc B l Ta = lna c gi l ton t logarit. Nh c ton t loga m php nhn cc gc c chuyn thnh php cng cc nh: (1) T(a1.a2) = Ta1 + Ta2 Do mun tnh tch a1.a2, ta tm nh ca n theo (1) sau dng bng logarit tra ngc li ) Cho A l tp hp cc hm dao ng hnh sin c cng tn s gc , B l tp hp cc hm bin s thc t nhng ly gi tr phc. Cho ng mi hm v(t) = Vsin(t +) A vi mt hm Tv B theo cng thc: Tv = V.ej(t + ) cng l mt ton t. Nh ton t ny m cc php tnh o hm v tch phn gc c chuyn thnh cc php tnh i s i vi nh. Trong chng ny ta s nghin cu ton t Laplace. Bi ton t ra l bit gc, tm nh ton t Laplace ca n v ngc li bit nh ca mt hm, tm li gc ca n. 2. NH NGHA HM GC Ta gi hm f(t) ca bin thc t l hm gc nu n tho mn cc iu kin sau: Hm f(t) lin tc tng khc khi t 0, ngha l nu ly mt khong [a, b] bt k trn na trc t 0, bao gi cng c th chia n thnh mt s hu hn cc khong nh sao cho trong mi khong nh f(t) lin tc v ti mt ca mi khong nh n c gii hn mt pha Khi t +, hm f(t) tng khng nhanh hn mt hm m, ngha l tn ti mt s M>0, so 0 sao cho: f ( t ) Me s o t t > 0 (2) trong so c gi l ch s tng ca f(t) f(t) = 0 khi t < 0. iu kin ny c t ra v trong cc ng dng thc t t thng l thi gian. V d 1: Hm: ( t ) = 0 khi t < 0 1 khi t > 0 l hm gc. Tht vy v | (t) | 1 nn iu kin 2 c tho mn nu chn M = 1, s0 = 0; d dng kim tra c iu kin 1. V d 2: Hm:

98

khi t < 0 f ( t ) = ( t ). sin t = 0 sin t khi t > 0 l hm gc. Tht vy v | (t).sint | 1 nn iu kin 2 c tho mn nu chn M = 1, s0 = 0; d dng kim tra c iu kin 1. V d 3: Hm: 0 khi t < 0 f ( t ) = ( t ).t 2 = t2 khi t > 0 l hm gc. Tht vy v | (t).t2 | 2et nn iu kin 2 c tho mn nu chn M = 2, s0 = 1; d dng kim tra c iu kin 1. Quy c: Ta vit (t) thay cho (t).(t) gii hn phi ca f(t), tc l khi t + 0 c vit l f(0)

3. NH L C BN Nu f(t) l hm gc c ch s tng l s0 th tch phn:

F( p) = e pt f ( t )dt
0

(3)

trong p = s + j l mt tham s phc s hi t trong min Rep = s > so (na mt phng phc bn phi ng thng s = so) Tch phn (3) l mt hm ca bin s phc p. Hm bin phc F(p) gii tch trong min Rep > so v dn ti 0 khi p sao cho Rep = s +. Chng minh: Ly p bt k thuc min Rep > so, ta s chng minh tch phn (3) hi t. Mun vy ta chng minh n tha nhn mt tch phn tri hi t tuyt i. Tht vy v f ( t ) Me s o t nn f ( t )e pt Me s o t e st = Me ( s o s ) t . Do :
+

f ( t ).e

pt

dt M e
0

(so s) t

Me ( s o s ) t dt = so s

V s0 - s < 0 nn lim e
t +
+

(s o s ) t

= 0 . Do :
(4)

f ( t ).e pt dt

M so s

iu chng t (3) hi t. Khi p = s + j + sao cho s + th

M 0 nn so s

F(p) 0. Ta cn phi chng minh F(p) gii tch trong min Rep > so. Mun vy ta chng minh
+ 0

o hm ca F(p) tn ti ti mi im ca min y. Xt tch phn


+

t.e

pt

f ( t )dt thu

c bng cch ly o hm mt cch hnh thc

e
0

pt

f ( t )dt di du tch phn.


99

Trong na mt phng Rep s1 vi s1 bt k ln hn so th tch phn tha nhn mt tch phn tri hi t v khng ph thuc tham s p: + + + M (so s) t pt (5) dt < M t.e ( s o s1 ) t dt = f ( t ).e dt M t.e (s1 s o )2 0 0 0 Vy theo nh l Weierstrass, tch phn hi t u i vi p trong min v l o hm ca F(p). Tm li:

F( p) = te pt f ( t )dt
0

(6)

4. NH NGHA TON T LAPLACE Ton t Laplace, cn gi l php bin i Laplace. Nu f(t) l mt hm gc th hm F(p) c xc nh bng tch phn (3) l mt hm gii tch trong na mt phng Rep>so. Ta gi n l nh ca f(t) qua php bin i Laplace ca f(t) v k hiu: F(p) = L{ f(t) } hay f(t) = F(p). Ta c:

L{f ( t )} = e pt f ( t )dt
0

(7)

Ch : Cc iu kin trong nh ngha hm gc f(t) ch l iu kin nh tn 1 khng phi l hm gc ti ch khng phi l iu kin cn. Chng hn hm f ( t ) = t + 1 1 v lim = . Tuy vy tch phn e pt dt vn tn ti t +0 t t 0 Khng phi mi hm phc F(p) u c nghch nh l mt hm gc. Chng hn F(p) = p2 khng th l nh ca mt hm gc no c v lim F( p) = . iu ny mu thun
p

vi kt lun ca nh l 1. Nu F(p) gii tch ti th F(p) 0 khi p mt cch bt k ch khng phi ch trong trng hp p sao cho Rep +. V d 1: Tm nh qua php bin i Laplace (gi tt l nh) ca hm (t): ( t ) = 0 khi t < 0 1 khi t > 0

e pt L{f ( t )} = F( p) = e dt = p 0
+ pt

1 e ( s + j ) t = p p
-st

1 e st e jt = p p

Nu Rep = s > 0 th khi t , e 0; khi t 0, e 1. Vy: 1 F(p) = p V d 2: Tm nh ca hm f(t) = eat trong a = + j = const e (a p) t at pt (a p) t dt = Ta c F( p) = e e dt = e ap 0 0
+ +

-st

(8)

Khi t 0 th e Vy:

(a-p)t

1. Nu Rep>Rea (s>) th khi t +, e(a-p)t = e(-s)tej()t 0.


100

1 pa V d 3: Tm nh ca f(t) = t. F( p ) = te pt 1 + pt F( p) = te dt = tde = p p 0 0
+ pt

(9) e pt 1 + pt + te dt = 2 p p 0

Khi t 0 th e 1. Khi t +, e 1 F( p ) = 2 p V d 4: Tm nh ca f(t) = tn.


+
n

-pt

-pt

0. Vy:

1 + n pt t n e pt F( p) = t e dt = t de = p 0 p 0 Sau n ln tch phn phn on ta c: n! F( p ) = n + 1 p


pt

1 + n 1 pt + t e dt p 0

5. CC TNH CHT CA PHP BIN I LAPLACE 1. Tnh cht tuyn tnh ca ton t: Gi s f(t) v g(t) l hai hm gc. A v B l hai hng s thc hay phc. Nu f(t) = F(p), g(t) = G(p) th: (10) Af(t) + B g(t) = F(p) + G(p) Tht vy theo nh ngha:

L{Af ( t ) + Bg( t )} = e pt [Af ( t ) + Bg( t )]dt


0

Do tnh cht tuyn tnh ca tch phn ta c:


+ 0

e [Af ( t ) + Bg( t )]dt = A e


pt 0

pt

f ( t )dt + B e pt g( t )dt
0

Nhng theo gi thit :


+ 0 + 0

e e

pt

f ( t )dt = F( p) g( t )dt = G( p)

pt

Thay vo trn ta c: L{Af ( t ) + Bg( t )} = AF( p) + BG( p) V d 1:Tm nh ca f(t) = sinat v cosat Theo cng thc Euler ta c: e jat e jat 1 1 sin at = = e jat e jat 2j 2j 2j Nhng theo (9):
101

1 1 ; e jat p ja p + ja S dng tnh cht tuyn tnh ta c: 1 1 1 a sin at = 2 2 j p ja p + ja p + a 2 ejat =

(11)

1 1 1 a = 2 2 2j p ja p + ja p + a e jat + e jat 1 jat 1 jat Tng t cos at = = e + e 2 2 2 1 1 1 p + cos at = 2 2 p ja p + ja p + a 2 V d 2: Tm nh ca ch(at) v sh(at) e at + e at 1 at 1 at chat = = e + e 2 2 2 at at e e 1 1 shat = = e at e at 2 2 2 1 1 1 p + chat = 2 2 p a p + a p a2 L{sin at } = 1 1 1 a = 2 2 2 p a p + a p a V d 3: Tm nh ca sin(t + ) v cos(t + ) Ta c sin(t + ) = sintcos + sincost. Do tnh cht tuyn tnh: p p sin + cos + cos 2 = sin(t + ) sin 2 2 2 p + p + p 2 + 2 p cos sin Tng t: cos(t + ) s = p 2 + 2 V d 4: Tm nh ca sin3t 1 Ta c: sin 3 t = (3 sin t sin 3t ) 4 1 3 3 3 1 1 2 2 Vy: sin 3 t 2 = 2 4 p +1 p + 9 4 p +1 p + 9 shat
2. Tnh cht ng cp: Nu L{ f(t) } = F(p) th L{ af(t) } = aF(p)

(12)

(13) (14)

3. Tnh cht ng dng: Gi s l mt hng s dng bt k. Nu f(t) F(p) th 1 p f (t ) F (15)


102

Chng minh: Theo nh ngha ta c:

f (t ) e pt f (t )dt
0

Trong tch phn v phi, i bin t = t1, dt =


+ t pt

1 dt 1 ta c:

1 + p 1 1 = e f ( t ) dt e f ( t 1 )dt 1 = F( p) 0 0

4. Tnh cht chuyn dch nh: Cho a l mt s phc bt k. Nu L{ f(t) } = F(p) th eatf(t) F(p - a) (16) Chng minh: Theo nh ngha ta c:

e f ( t ) e e f ( t )dt = e ( p a ) t f ( t )dt = F( p a )
at at pt 0 0

V d 1: Tm nh ca e sint v eatcost p Ta c sin t 2 v cos t p + 2 p 2 + 2 Nn: e at sin t ( p a ) 2 + 2 pa e at cos t ( p a ) 2 + 2 V d 2: Gi s f(t) F(p). Tm nh ca f(t)sint e jt e jt = f ( t ) sin t f ( t ) Ta c: 2j Do cng thc dch chuyn nh: f(t)ejt F(p - j) f(t)e-jt F(p + j) Theo tnh cht tuyn tnh ta c: 1 f(t)sint [ F(p - j) + F(p + j) ] 2j 5. Tnh cht tr: a. Trng hp l mt hng s dng: Nu f(t) F(p) th: (t - )f(t - ) e-pF(p) (17) Trc ht ta thy rng nu (t)f(t) c th l ng cong C th th ca (t-)f(t-) c c bng cch dch chuyn ng cong C sang mt on theo trc honh. Nu t v l cc i lng ch thi gian th qu trnh biu din bi hm (t-)f(t - ) xy ra ging qu trnh biu din bi hm (t)f(t) nhng chm hn mt khong thi gian

at

103

(t)f(t)

(t-)f(t-)

O
Chng minh: Theo nh ngha ta c:
+ 0

( t )f ( t ) e pt ( t )f ( t )dt
V :
( t ) = 0 khi t < 1 khi t >
+ 0

nn: ( t )f ( t ) e pt f ( t )dt Trong tch phn bn v phi, i bin t1 = t - ta c:


+ 0

pt

f ( t )dt = e
0

p ( t1 + )

f ( t 1 )dt 1 = e

+ 0

pt1

f ( t 1 )dt 1 = e p F( p)

V d: ta bit hm f(t) = e2t c nh l F( p) =

1 . Tm nh ca hm f(t - 1) = e2(t - 1) p2

Theo (17) ta c:

e p f ( t 1) = e p2 b. Biu din mt hm xung qua hm (t):Ta gi mt hm xung l hm c


2 ( t 1)

dng:

khi t < a 0 f ( t ) = ( t ) khi a < t < b khi t > b 0 Ta c th vit: f(t) = (t - a)(t) - (t - b)(t) V d 1: Tm nh ca hm (t -) 1 V ( t ) nn theo tnh cht tr th: p 1 ( t ) e p p V d 2: Tm nh ca hm xung n v khi t < a 0 f (t) = 1 khi a < t < b khi t > b 0 Theo (18) th: f(t) = (t - a) - (t - b) Theo (19) th:

(18)

(19)

104

1 pb 1 1 pa e = (e e pb ) p p p V d 3: Tm nh ca hm: khi t < 0 0 f ( t ) = sin t khi 0 < t < khi t > 0 Theo (18) ta c th vit: f(t) = (t)sint - (t - )sint V sint = sin( - t) = -sin(t - ) nn: f(t) = (t)sint + (t - )sin( - t) Theo tnh cht tr ta c: 1 ( t ) sin( t ) e p 2 p +1 Kt qu 1 1 1 (1 + e p ) + e p 2 = 2 f (t) 2 p +1 p +1 p +1 V d 4: Tm nh ca hm bc thang sau: khi t < 0 0 2 khi 0 < t < 1 f (t) = 4 khi 1 < t < 2 1 khi 2 < t < 3 khi t > 3 0 t: khi t < 0 0 h1 ( t ) = 1 khi 0 < t < 1 0 khi t >1 khi t < 1 0 h 2 (t) = 1 khi 1 < t < 2 khi t > 2 0 f ( t ) e pa

(20)

2 O 3 t

khi t < 2 0 h 3 (t) = 1 khi 2 < t < 3 khi t > 3 0 Nh vy: f(t) = 2h1(t) + 4h2(t) + h3(t) V theo (20): 1 1 1 h 1 ( t ) (1 e p ) ; h 2 ( t ) (e p e 2 p ) ; h 3 ( t ) (e 2 p e 3p ) p p p 1 1 nn: f ( t ) (2 2e p + 4e p 4e 2 p + e 2 p e 3p ) = (2 + 2e p 3e 2 p e 3p ) p p V d 5: Tm nh ca hm f(t) nh hnh v
105

Hm f(t) c coi l tng ca hai hm xung h1(t) v h2(t): khi t < 0 0 1 t h1 ( t) = khi 0 < t < h h 0 h O khi t > h khi t < h h 2 (t) = 0 1 khi t > h Theo (18) ta c: t t h 1 ( t ) = ( t ) ( t h ) h h h 2 ( t ) = ( t h ) t t t th f ( t ) = ( t ). ( t h ). + ( t h ) = ( t ) ( t h ). h h h h Vy: 1 = [( t ).t ( t h ).( t h )] h Theo tnh cht tr ta c: 1 1 1 1 f ( t ) = 2 e hp 2 = 2 (1 e hp ) h p p hp

6. NH CA MT HM TUN HON Nu f(t) l mt hm gc, tun hon vi chu k T, ngha l f(t) = f(t + T) t > 0 th nh ca n c tnh theo cng thc: ( p) (21) F( p ) = 1 e pT

Trong : ( p) = e pt f ( t )dt l nh ca hm:


0

khi t < 2 0 ( t ) = f ( t ) khi 0 < t < T khi t > T 0 Chng minh: Theo nh ngha ta c:

F( p) = e f ( t )dt = e f ( t )dt + e pt f ( t )dt


pt pt

T 0

Trong tch phn th v phi, i bin t = u + T ta c:


+ T

e e

pt

f ( t )dt = e
0

p( u + T )

f ( u + T )du = e

pT

+ 0

pu

f ( u + T )du

Do tnh cht tun hon f(u + T) = f(u), nn:


+ T pt

f ( t )dt = e

pT

+ 0

pu

f ( u )du = e pT .F( p)

Thay vo trn ta c:
106

F(p) = (p) + e-pTF(p) T rt ra: ( p) F( p ) = 1 e pT V d 1: C mt h thng xung nh hnh v. Tm nh ca hm : 1 O Ta c: e pt ( p) = e f ( t )dt = e dt = p 0 0


+ pt pt

T 1 (1 e p ) p

=
0

Vy: F( p) =

1 1 e p 1 e pT V d 2: Cho mt h thng cc xung hnh sin nh hnh v. Tm nh

pt

Ta thy rng hm f(t) = | sint | tun hon vi chu k T = . Trong v d 3 5 ta bit: 1 (1 e p ) ( p) = 2 p +1 1 1 + e p 1 p coth = Vy: F( p) = 2 1 e p p 2 + 1 p +1 2
7. O HM GC 1. o hm cp 1: Gi s f(t) l hm gc, c o hm f(t) cng l hm gc. Nu f(t)F(p) th: f(t) pF(p) - f(0) (22) Chng minh: Theo nh ngha:

f ( t ) e pt f ( t )dt
Trong tch phn bn v phi, dng phng php tch phn tng phn, t u = e-pt ta c du = -p.e-pt, dv = f(t)dt nn v= f(t). Thay vo ta c:
0

107

+
0

pt pt e f ( t )dt = f ( t )e

+
o

+ p e pt f ( t )dt = f ( t )e pt
0

+
o

+ pF( p)

Do | f(t) | M e nn nu Rep = s > so th | f(t)e-pt | M e ( s o s ) t 0 khi t +.Vy:


so t

f ( t )e pt
+
0

+ o

= f ( 0)

Thay vo trn ta c:

pt

f ( t )dt = pF( p) f (0)

2. o hm cp cao: Nu f(t) c o hm ti cp n v cc o hm ny u l hm gc th bng cch p dng lin tip (22) ta c: (23) f(n)(t) = pnF(p) - pn-1f(0) - pn-2f(0) - L - f(n-1)(0) 3. H qu: Nu f(t) l hm gc v pF(p) gii tch ti th: (24) lim pF( p) = f (0)
p

8. TCH PHN GC

Nu f(t) F(p) th f ( t )dt l mt hm gc v


0

f ( t )dt
0

F( p) p
t 0

(25)

Chng minh: t ( t ) = f ( t )dt . R rng (0) = 0.. Hm (t) c o hm l hm f(t) lin tc tng khc. Bi v: M sot t e M 1e s o t 0 so 0 0 nn (t) l mt hm gc cng ch s tng vi f(t). Gi (p) l nh ca n. Ta phi tm (p). V (t) = f(t) nn theo cng thc o hm gc ta c: f(t) p(p) - (0) Vy F(p) = p(p) hay F( p) ( p) = p ( t ) f ( t ) dt Me s o t dt =
9. O HM NH
t t

Nu f(t) F(p) th:

108

F(p) -tf(t) Chng minh: Theo (6) ta c:

(26)

F( p) = tf ( t )e pt dt
0

Mt khc, theo nh ngha th:

tf ( t ) tf ( t )e pt dt
0

Vy: F(p) -tf(t) S dng cng thc ny lin tip ta c: tnf(t) (-1)nF(n)(p) Mt cch tng qut ta c: n! t n n +1 p 10. TCH PHN NH f (t) , ngha l: Nu tch phn F( p)dp hi t th n l nh ca hm t p
f (t) F( p)dp t p Chng minh: Ta c: p

(27) (28)

(29)

F( p)dp = dp f ( t )e
p 0

pt

dt

(30)

Ly s1 l mt s ln hn so. Gi s ng ly tch phn (p, ) nm hon ton trong na mt phng Rep 0. Khi ta c:
0

f ( t )e

pt

dt M e ( s1 s o ) t dt
0 0

D dng thy rng tch phn v phi hi t nn tch phn f ( t )e pt dt hi t u i vi p. Vy trong (3) ta c th i th t ly tch phn: f ( t ) pt pt F( p)dp = f ( t )dt e dp = t e dt p 0 p 0 Hay:
f (t) F( p)dp t p

e bt e at V d 1: Tm nh ca hm t 1 1 nn theo (29) ta c: V e bt e at pb pa
109

+ pa e bt e at 1 1 dp = ln pb t pa ppb t sin t V d 2: Tm nh ca hm t 0 1 nn theo (29) ta c: Ta bit sin t 2 p +1 sin t dp 2 = arctgp = arcotgp t 2 p p +1 Dng cng thc tch phn gc ta c: t sin t 1 arcotgp t p 0

11. NH CA TCH CHP 1. nh ngha tch chp ca hai hm s: Cho hai hm s f(t) v g(t). Tch phn
t

f ( )g( t )d
0 t

l mt hm s ca t v c gi l tch chp ca hai hm s f(t) v

g(t). N c k hiu l f g

f g = f ( )g( t )d
0

(31)

2. Tnh cht: a. Tnh cht 1: Tch chp c tnh cht giao hon f * g = g * f Tht vy dng php i bin 1 = t - , d1 = -d, ta c:

f g = f ( )g( t )d = f ( t 1 )g( t )d1 = g( 1 )f ( t 1 )d1 = g f


0 t 0

b. Tnh cht 2: Nu f(t) v g(t) l nhng hm gc th f * g cng l hm gc V d 1: Tnh tch chp e t t = e ( t )d


0 t

Tnh tch phn bn v phi bng phng php tch phn tng phn ta c:
e t = e ( t )d = t (e t 1) ( te t e t + 1) = e t t 1
t 0 t

t * e = e
at 0

a ( t )

t e at 1 d = e e d = + 2 2 a a a 0
at t a

V d 2:

110

sin t t = ( t ) sin d = sin t + t cos t t = ( t ) cos d = cos t + 1


0 0 t

3. nh ca tch chp: Nu f(t) F(p) v g(t) G(p) th nh ca tch chp bng tch cc nh: f * g F(p).G(p) (32) Chng minh: Theo nh ngha th:

f g = f ()g( t )d e pt dt f ()g( t )d
0 0 0

Xt tch phn bn v phi. V ng vi t c nh th tch phn theo ly t 0 n t, sau cho t bin thin t 0 n + nn v phi tch phn lp ly trong mn qut G: 0 < arg(t + j) < . V khi Rep > s + 1 4 th do tnh cht ca tch chp, tch phn lp ny hi t tuyt i nn ta c th i th t tch phn:
+

=t t

e
0

pt

dt f ()g( t )d = f ()d e pt g( t )d
0 0 t
p

i bin t1 = t - th:
+

e
0

pt

g( t )d = e
t

e
0

pt1

g ( t1 )dt1
0

Vy:

pt pt p e dt f ()g(t )d = e f ()d e 1g(t1 )dt1 = F(p).G(p) 0 0

ngha l:

f * g = F(p).G(p) 1 1 V d: t*sint = t - sint 2 . 2 p p +1


4. Cp cng thc Duhamel: Nu f(t) F(p) v g(t) G(p) th: p.F(p).G(p) f(0).g(t) + f * g (33) p.F(p).G(p) g(0).f(t) + f * g (34) Chng minh: Ta ch cn chng minh cng thc (33) v do tnh cht i xng ta suy ra cng thc (34). Ta c: pF(p).G(p) = f(0).G(p) + [ pF(p) - f(0) ].G(p) Theo cng thc o hm gc: pF(p) - f(0) f(t) Theo cng thc nhn nh: [ pF(p) - f(0) ].G(p) f(t) Vy: p.F(p).G(p) f(0).g(t) + f * g
111

12. NH CA TCH HAI GC Gi s f(t) v g(t) l hai hm gc c ch s tng s1 v s2. Khi tch f(t).g(t) cng l mt hm gc tnh theo cng thc: 1 a + j f ( t ).g ( t ) (35) F().G (p )d 2j a j 13. QUAN H GIA GC V NH nh l:Nu f(t) l mt hm gc vi ch s tng so v F(p) l nh ca n th ti mi im lin tc ca hm f(t) ta c: 1 a + j pt f (t) = (36) e F(p)dp 2j a j trong a l mt s thc bt k ln hn so. Tch phn bn v phi c hiu theo ngha gi tr chnh. Cng thc (36) c gi l cng thc ngc ca Mellin. Ta tha nhn m khng chng minh nh l ny. 14. IU KIN F(p) L MT HM NH nh l: Gi s F(p) l mt hm bin phc tho mn cc iu kin sau: 8F(p) gii tch trong na mt phng Rep > so 8F(p) 0 khi | p | + trong na mt phng Rep > a > so u i vi argp
a + j a j

8tch phn

pt

F(p)dp hi t tuyt i

Khi F(p) l nh ca hm gc cho bi cng thc: 1 a + j pt f (t) = e F(p)dp a > so t > 0 2j a j


15. TM HM GC CA MT PHN THC THC S

(37)

Mt phn thc hu t c gi l thc s nu bc ca mu s ln hn bc ca t s ca n. A ( p) , trong t s v mu s l cc a Cho mt phn thc thc s F(p) = B(p) thc khng c nghim chung. Nu gi ak (k = 1, 2,.., n) l cc im cc ca F(p) th F(p) l nh ca hm (t).f(t) trong : f ( t ) = Res F(p)e pt , a k
k =1 n

(40)

( Nu ak l cc im cp mk th theo cng thc tnh thng d:

112

( m k 1) 1 lim (p a k ) mk F(p)e pt (m k 1)! pa k nn cng thc (40) tr thnh: n ( m k 1) 1 f (t) = lim (p a k ) mk F(p)e pt (42) k =1 ( m k 1)! pa k ( c bit, nu cc cc im u n, tc mk = 1, th cch tnh thng d n gin hn: A (a k ) a k t e Res[ F(p)ept, ak ] = B(a k ) v ta c: n A (a k ) a k t f (t) = e (43) k =1 B(a k ) ( c bit hn na, nu s 0 cng l mt cc im n th khi mu s B(p) c tha s chung l p: B(p) = p.B1(p) vi B1(0) 0, B1(ak) = 0 khi k = 2, 3,..,n. Trong cng thc (43) chn a1 = 0 ta c: A(0) n A(a k ) a k t + f (t ) = e B(0) k =2 B(a k ) (p) nn B(0) = B1(0), B(ak) = a k B1 (a k ) nn: V B(p) = B1(p) + pB1 A(0) n A(a k ) a k t A ( p) + f (t ) = e (44) B(0) k =2 B(a k ) pB1 (p) ( Nu A(p) v B(p) l cc a thc c cc h s u l s thc v nu cc cc im u n gm: * nhng s thc b1, b2,..., br * nhng s phc lin hp a1, a2, ..., as, a1 , a 2 ,K, a s khi r + 2s = n l s cc im; ak = k + jk, a k = k j k v t A (a k ) = M k + jN k th (43) cn c th vit di dng sau: B(a k ) r A ( b k ) bk t s k t f (t ) = e + 2e [M k cos k t N k sin k t ] (46) k =1 k =1 B( b k ) 1 V d 1: Tm gc ca hm F(p) = p(p + a )(p + b) Trong v d ny A(p) = 1; B(p) = p.B1; B1 = (p + a)(p + b). Cc cc im ca F(p) l: a1 = 0; a2 = -a; a3 = -b p dng cng thc (44) ta c: 1 e at e bt f (t ) = + + ab a (a b) b(b a ) 3p 2 + 3p + 2 V d 2: Tm gc ca hm: F(p) = (p 2)(p 2 + 4p + 8)

Res[ F(p)ept, ak ] =

113

Trong v d ny A(p) = 3p2 + 3p + 2, B(p) = (p - 2)(p2 + 4p + 8), B(p) = 3p2 + 4p. Cc cc im ca F(p) l: b1 = 2, a1 = -2 + 2j a1 = -2 - 2j nn 1 = -2, 1 = 2 Theo (46) ta c: A (a 1 ) A(b1 ) b1t A (a 1 ) f (t) = e + 2e 1t Re cos 1t Im sin 1t B(b1 ) B(a1 ) B(a1 ) Nhng: A(2) 20 = =1 B' (2) 20 A(2 + 2 j) 18 j 4 j = =1+ B(2 + 2 j) 2(4 + 8 j) 4 Vy: 1 f ( t ) = e 2 t + 2e 2 t cos 2t sin 2t 4 p+2 V d 3: Tm gc ca hm F(p) = (p 1) 2 p 3 Ta c A(p) = p + 2, B(p) = p3(p - 1)2. Vy F(p) c hai cc im l: a1 = 1 (cp 2) v a2 = 0 (cp 3) tnh f(t) ta dng cng thc (42): p + 2 pt p 5 pt 3 p+2 1 1 1 p + 2 pt pt lim p 3 e = lim e e + te = = lim 2 2 p0 2 p0 (p 1) 2 2 p0 (p 1) 3 (p 1) 3 p (p 1) 2p + 16 pt p 5 pt 1 p + 2 2 pt p 5 pt lim e te t e + te = + + 2 p0 (p 1) 4 (p 1) 3 (p 1) 2 (p 1) 3 1 (16 + 5t + 2t 2 + 5t ) = t 2 + 5t + 8 2 v: 2p 6 pt p + 2 pt p+2 pt p + 2 = 3te t 8e t lim (p 1) 2 e pt = lim e lim e te = + 4 3 3 2 3 p 1 p1 p 1 (p 1) p p p p Thay vo (42) ta c: p + 2 p + 2 pt 1 f ( t ) = lim e + lim 3 e pt = t 2 + 5t + 8 + (3t 8)e t 2 p1 2 p0 (p 1) p p e V d 4: Tm gc ca hm F(p) = 3 (p + 1)p 1 . i vi hm ny A(p) = 1, Trc ht ta tm gc g(t) ca hm G (p) = 3 ( p + 1) p B(p) = p(p3 + 1). Vy G(p) c cc cc im thc l:
114

b1 = 0, b2 = -1 v cp cc im phc lin hp: 1+ j 3 1 j 3 a1 = v a1 = . 2 2 Ta c: B(p) = 4p3 + 1 nn: B(0) = 1 B(-1) = - 3


1+ j 3 1+ j 3 B 4 + 1 = 4 cos + i sin = + 1 = 4(cos 3 + j sin 3 ) + 1 = 3 2 2 3 3 1 1 M1 = Re = B(a1 ) 3 1 N1 = Im =0 B(a1 ) Thay vo (46) ta c: t 1 ot 1 3 3 g(t ) = e + e t + 2e 2 M1 cos t N1 sin t B(0) B(1) 2 2
3 3

1 2 3 = 1 e t e 2 cos t 3 3 2 tnh f(t) ta dng tnh cht tr theo (17): (t - 1)g(t - 1) e-pG(p) = F(p) tc l: 1 1 t +1 2 t 3 2 f ( t ) = ( t 1)g( t 1) = ( t 1)1 e e cos (t 1) 3 2 3
V d 5: Tm gc ca hm F(p) =
2

p +1 p + 2p 2 Phng trnh p + 2p c hai nghim n l a1 = 0 v a2 = -2. p dng cng thc thng d ti cc im n ta c: p + 1 pt 1 Res[(p)ept, 0] = lim e = p 0 2 p + 2 2 p + 1 pt 1 pt Res[(p)ept, -2] = lim e = e p2 2p + 2 2 1 Vy F(p) (1 + e pt ) 2

16. TM HM GC CA MT PHN THC HU T

Trong thc t, tm gc ca mt phn thc hu t ta phn tch chng thnh cc phn thc ti gin loi 1:
115

1 1 vi a thc, n nguyn dng hay pa (p a ) n v cc phn thc ti gin loi 2: Mp + N Mp + W vi M, N, b, c thc; b2 - c < 0; n nguyn hay 2 2 n p + 2bp + c (p + 2bp + c) dng. i vi phn thc ti gin loi 1 ta ch rng: 1 1 t n 1 1; n p p n! Do dng cng thc dch chuyn nh ta c: 1 1 t n 1 at at e ; e pa (p a ) n (n 1)! i vi phn thc ti gin loi 2 ta lm nh sau: Ta a tam thc mu s v dng chnh tc: Mp + N M(p + b) + N Mb M(p + b) + N Mb = = n 2 p + 2bp + c (p + b) 2 + (c b 2 ) n ( p + b) 2 + 2

vi = c - b

Tm gc ca

(p
(p
2

Mp
2

nh. Khi tm gc ca nh.

+ p

2 n

v ca hay ca

(p
(p

N Mb
2

2 n

+ 2 ) 1

ri dng cng thc chuyn dch ta thng ti cng thc o hm

2 n

p +1 p2 + p + 1 a mu s v dng chnh tc ta c: 2 1 3 2 p + p +1 = p + + 2 4

V d 1:Tm gc ca hm F(p) =

1 1 p +1 p +1 2 2 = = + Vy: F(p) = 2 2 2 2 p + p +1 1 3 1 3 1 3 p + + p + + p + + 2 4 2 4 2 4 p 3 Ta c: cos t 3 2 2 p + 4 1 2 3 sin t 3 2 2 3 p + 4 p+


116

p dng cng thc dch chuyn nh ta c: t 3 p 2 e cos t 2 2 1 3 p + + 2 4 t 3 2 1 2 e t sin 2 2 3 1 3 p + + 2 4 t t 3 2 3 2 2 t+e sin t Vy: f ( t ) e cos 2 2 3 3p 4 V d 2: Tm gc ca hm F(p) = 2 ( p 2 p + 2) 2 a mu s v dng chnh tc ta c: 3p 4 3(p 1) 1 3(p 1) 1 F(p) = 2 = = 2 2 2 2 (p 2p + 2) (p 1) 2 + 1 (p 1) 2 + 1 (p 1) 2 + 1 3p 1 th G(p - 1) = F(p). Vy nu tm c gc ca G(p) ta t G (p) = 2 2 2 (p + 1) (p + 1) 2 s dng cng thc dch chuyn nh tm gc ca F(p). 1 1 p 1 1 = 2 V: 2 + (p 2 + 1) 2 2 p + 1 2 p + 1 p 1 1 = 2 (p 2 + 1) 2 2 p + 1 3 1 1 p 1 1 (47) nn: G (p) = 2 2 p2 + 1 2 p2 + 1 + 2 p 1 1 p V: sin t; 2 cos t 2 p +1 p +1 nn p dng tnh cht o hm nh ta c: 1 p p2 + 1 t sin t; p2 + 1 t cos t T (47) ta suy ra: 3 3 1 g ( t ) = t sin t + t cos t sin t 2 2 2 3 1 3 f ( t ) = e t g( t ) = e t t sin t + t cos t sin t 2 2 2 2 3p + 2p + 2 V d 3: Tm gc ca hm F(p) = (p 2)(p 2 + 4p + 8)

] [

] [

117

Phn tch F(p) thnh phn thc ti gin ta c: 3p 2 + 2p + 2 1 2p + 3 1 2( p + 2) 1 F(p) = = + 2 = + 2 (p 2)(p + 4p + 8) p 2 p + 4p + 8 p 2 (p + 2) 2 + 4 p+2 1 1 = +2 2 (p + 2) + 4 (p + 2) 2 + 4 p2 1 V e2t p2 2p 2 cos 2t 2 p +4 1 1 sin t p2 + 4 2 Nn chuyn dch nh ta c: sin 2 t 1 p+2 2e 2 t cos 2 t; e 2 t 2 2 2 2 ( p + 2) + 4 ( p + 2) + 4 Cui cng: sin 2 t f ( t ) = e 2 t + 2e 2 t cos 2 t e 2 t 2
17. TM HM GC DI DNG CHUI nh l: Nu hm F(p) gii tch ti p = , ngha l ti ln cn p = , khai trin Laurent ca n c dng: C C 2 C3 Cn F(p) = 1 + 2 + 3 +L = n (48) p p p n =1 p th F(p) l nh ca hm (t)f(t) trong : C2 C2 2 t n 1 (49) f ( t ) = C1 + t+ t + L = Cn 1! 2! (n 1)! n =1

1 V d 1: Tm gc ca hm F(p) = n +1 e p p Khai trin 1 1 1 1 (1) k 1 p e = 1 + +L = p 2! p 2 3! p 3 k! p k k =0 (1) k 1 Vy: F(p) = k! p n +k +1 k =0 1 t n+k V: p n +k +1 (n + k )! (1) k t n +k nn: f ( t ) = k! (n + k )! k =0
118

p9 V d 2: Tm gc ca hm F(p) = 10 p 1 Khai trin F(p) ti ln cn p = ta c: p9 p9 1 1 1 1 1 F(p) = 10 = = = + 11 + 21 + L + 10 n +1 + L p 1 p p 1 1 p p p10 1 p10 p 1 p10 Theo nh l trn ta c: t10 t 20 t10 n f (t) = 1 + + +L+ +L (10n )! 10! 20! 1 V d 3: Tm gc ca hm F(p) = p2 + 1 p dng khai trin nh thc ta c: 1 1 1 1 1 3 1 1 1 3 5 1 1 = + + + F(p) = 1 . L 2 4 6 p 2 + 1 p 2 p 2 2 2! p 2 2 2 3! p

1 1 1.3 1.3.5 3+ 2 3 +L 5 p 2p 2 .2!p 2 .3!p 7

1 tn Do p n +1 n! Nn ta c:
t2 t4 t 2n n f (t) = 1 2 2 + 4 + L = (1) 2 n 2 (1!) 2 (2!) 2 2 (n!) 2 n =0

18. DNG CNG THC NHN NH V CNG THC DUHAMEL Ta nhc li cng thc nhn nh: F(p).G(p) = f*g pF(p)G(p) = f*g + f(0)g(t) 2p V d: Tm gc ca hm F(p) = 2 (p + 1)(p 2 + 4) Ta c th vit: 2p 2 p F(p) = 2 = 2 . 2 2 (p + 1)(p + 4) p + 1 p + 4 2 p V 2 sin t; 2 2 cos 2t 2 p +1 p +4 nn theo cng thc nhn nh ta c:

f ( t ) = 2 sin t * cos 2t = 2 sin( t ) cos 2d


0

Nhng 2sin(t - ).cos2 = sin(t + )sin(t - 3) nn:


119

120

f ( t ) = 2 sin( t + )d + sin( t 3)d


0 0

cos( t 3) 1 1 = cos( t + ) 0 + = cost cos2 t + cos2 t cost 3 3 3 0


t

2 2 = cost cos2 t 3 3
19. NG DNG CA PHP BIN I LAPLACE GII PHNG TRNH VI PHN TUYN TNH H S HNG 1. Phng php chung: Gi s ta cn tm nghim ca phng trnh vi phn tuyn tnh h s hng: dn x d n 1x a o n + a 1 n 1 + L + a n x = f ( t ) (1) dt dt tho mn cc iu kin ban u: x(0) = xo, x(0) = x1 ,.., x(n-1)(0) = xn-1 (2) vi gi thit ao 0, hm f(t), nghim x(t) cng cc o hm ti cp n ca n u l cc hm gc. tm nghim ca bi ton trn ta lm nh sau: bTrc ht ta lp phng trnh nh ca (1) bng cch gi X(p) l nh ca x(t), F(p) l nh ca f(t). Theo cng thc o hm gc ta c: x(t) = pX(p) - xo x(t) = p2X(p) - pxo - x1 x(n)(t) = pnX(p) - pn-1xo - - xn-1 Ly nh hai v ca (1) ta c phng trnh i vi nh X(p): (aopn + a1pn-1 + + an)X(p) = F(p) + xo(aopn-1 + a1pn-2 + + an-1) + x1(aopn-1 + a1pn-2 + + an-1) + + xn-1ao hay: A(p).X(p) = F(p) + B(p) (3) Trong A(p) v B(p) l cc a thc bit. Gii (3) ta c: F(p) + B(p) (4) X ( p) = A ( p) b Sau tm gc ca X(p) ta c nghim ca phng trnh V d 1: Tm nghim ca phng trnh x - 2x + 2x = 2etcost tho mn iu kin u x(0) = x(0) = 0 t x(t) X(p) th x(t) pX(p) v x(t) p2X(p). 2(p 1) 2(p 1) . Thay vo phng trnh ta c: Mt khc 2e t cos t = 2 2 (p 1) + 1 p 2p + 2
119

p 2 X 2pX + 2X = hay ( p 2 2 p + 2) X =

2(p 1) p 2p + 2
2

2(p 1) p 2p + 2
2

Gii ra ta c: 2(p 1) X= 2 (p 2p + 2) 2 Dng php bin i ngc ta c: x(t) = tetsint V d 2: Tm nghim ca phng trnh x - x = 4sint + 5cos3t tho mn cc iu kin ban u x(0) = -1, x(0) = -2 5p v t x(t) X(p) th x(t) p2X + p + 2. Mt khc 5cos2 t 2 p +4 4 . Thay vo phng trnh trn ta c: 4 sint 2 p +1 4 5p p2X + p + 2 X = 2 + 2 p +1 p + 4 nn: 4 5p p+2 X= 2 + 2 2 2 2 (p + 1)(p 1) (p + 4)(p 1) p 1 2 2 p p p+2 = 2 2 + 2 2 2 p 1 p +1 p 1 p + 4 p 1 2 p = 2 2 p +1 p + 4 Dng php bin i ngc ta c: x(t) = -2sint - cos2t V d 3: Tm nghim ca phng trnh x + 4x + 4x = t3e-2t tho mn cc iu kin ban u x(0) = 1, x(0) = 2. t x(t) X(p) th x(t) pX - 1, x(t) p2X - p - 2. Mt khc 3! 6 . Thay vo phng trnh trn ta c: t 3e 2 t = (p + 2) 4 (p + 2) 4 6 p 2 X p 2 + 4pX 4 + 4X = (p + 2) 4 Nh vy: 6 p+6 6 4 1 X= + = + + (p + 2) 6 (p + 2) 2 (p + 2) 6 (p + 2) 2 p + 2

120

1 5 2 t t5 2 t Vy x(t) = x ( t ) = e + 4te + t e = e 1 + 4t + 20 20 (4) V d 4: Tm nghim ca phng trnh x + 2x + x = sint tho mn cc iu kin ban u x(0) = x(0) = x(0) = x(3)(0) = 0. 1 . t x(t) X(p) th: x(t) p2X, x(4)(t) p4X. Mt khc sin t 2 p +1 Thay vo phng trnh trn ta c: 1 (p 4 + 2p 2 + 1)X = 2 p +1 1 1 1 X= 2 = 2 = 4 2 3 3 (p + 1)(p + 2p + 1) (p + 1) (p j) (p + j) 3 Hm X(p)ept c hai im cc cp 3 l j v -j. Ta tnh thng d ti cc cc im : e pt 12e pt 6 te pt t 2 e pt 1 1 pt = lim + Res[X(p)e , j] = lim 2 p j (p + j) 5 (p + j) 4 (p + j) 3 2 p j (p + j) 3 e jt = 3t + j( t 2 3) 16 e pt 12e pt 6 te pt t 2 e pt 1 1 pt = lim + Res[X(p)e , -j] = lim 2 p j (p j) 5 (p j) 4 (p j) 3 2 p j (p j) 3 jt e = 3t j( t 2 3) 16 Theo cng thc tm gc ca phn thc hu t ta c: x(t) = Res[X(p)ept, j] + Res[X(p)ept, -j] e jt e jt 2 = 3t + j( t 3) + 3t j( t 2 3) 16 16 e jt e jt = 3t + j( t 2 3) + 3t + j( t 2 3) 16 16 e jt 3 3 t2 = 2 Re 3t + j( t 2 3) = t cos t + sin t 16 8 8 V d 5: Tm nghim ca phng trnh x + x = et tho mn cc iu kin ban u x(1) = x(1) = 1. Cc iu kin ban u y khng phi cho ti t = 0 m ti t = 1. V vy ta phi bin i quy v trng hp trn. Ta t t = + 1, x(t) = x( + 1) = y(), Vy x(t) = y(), x(t) = y(). Bi ton c a v tm nghim ca phng trnh: y() + y() = e+1 tho mn y(0) = 1 v y(0) = 0
2 t 2 t

[ [

] ]

121

Gi Y(p) l nh ca y(). Vy y() p2Y(p) - p. Mt khc e +1 = e.e Vy phng trnh nh l: e p2Y p + Y = p 1 Gii phng trnh ny ta c: e p e e(p + 1) p + 2 = + 2 Y= 2 2 (p 1)(p + 1) p + 1 2(p 1) 2(p + 1) p + 1

e p 1

e e e p + 1 2 2(p 1) 2 p + 1 2(p 2 + 1) T ta c: e e e y() = e + 1 cos sin 2 2 2 Tr v bin t ta c: et e e x ( y) = + 1 cos( t 1) sin( t 1) 2 2 2 V d 6: Tm nghim ca phng trnh: 1 0 < t < 2 x + x = 0 t > 2 tho mn iu kin ban u x(0) = 0. t x(t) X(p) nn x(t) pX(p). V phi ca phng trnh c th vit c l f(t) = (t) - (t - 2). Vy: 1 f ( t ) (1 e 2 p ) p v phng trnh nh c dng: 1 pX + X = (1 e 2 p ) p Gii ra ta c: 1 e 2 p 1 e 2 p X= = p(p + 1) p(p + 1) p(p + 1) 1 1 1 Do = 1 e t p(p + 1) p p + 1 nn theo tnh cht tr ta c: 1 e 2 p ( t 2) 1 e ( t 2 ) p(p + 1) 1 e t 0<t<2 t ( t 2 ) Vy: x ( t ) = 1 e ( t 2) 1 e = t 2 e (e 1) t > 2 =

122

V d 7: Tm nghim ca phng trnh: sin t 0 < t < x + 2 x = t> 0 tho mn cc iu kin ban u x(0) = x(0) = 0. t x(t) X(p), nn x(t) p2X(p) Trc y ta tm c nh ca hm trong v phi l: 1 (1 + ep ) 2 p +1 Vy phng trnh nh tng ng l: 1 (1 + ep ) p 2 X + 2 X = 2 p +1 1 + e p hay: X = 2 (p + 1)(p 2 + 2 ) Ta xt hai trng hp: nu 2 1 th: 1 sin t sin t 2 2 2 (p + 1)(p + ) (1 2 ) Theo tnh cht tr e p sin ( t ) sin( t ) ( t ) 2 2 2 (p + 1)(p + ) (1 2 ) Vy: sin t sin t sin ( t ) sin( t ) + ( t ) x(t) = 2 (1 ) (1 2 ) hay: sin t sin t 0<t< (1 2 ) x(t) = sin ( t ) sin( t ) 2 cos 2 sin t 2 t> = 2 2 (1 ) (1 ) * nu 2 = 1 th: 1 + e p X= 2 (p + 1) 2 1 t cos t Ta bit 2 sin t 2 (p + 1) 2 Theo tnh cht tr ta c: e p ( t ) [sin( t ) ( t ) cos(t )] 2 2 (p + 1) 2
123

hay: Vy:

e p ( t ) [( t ) cos t sin t ] (p 2 + 1) 2 2 x(t) =


1 1 (sin t t cos t ) + ( t )[( t ) cos t sin t ] 2 2

hay:

1 (sin t t cos t ) 0 < t < 2 x(t) = cos t t> 2 V d 8: Gii h phng trnh: x + x y = e t t y + 3x y = 3e tho mn iu kin u x(0) = 1, y(0) = 1 t x(t) X(p), y(t) Y(p) nn x(t) = pX - 1, y(t) = pY - 1. Thay vo ta c h phng trnh nh: 1 + = pX 1 X Y p 1 pY 1 + 3X 2Y = 2 p 1 hay: 1 (p + 1)X Y = +1 p 1 3X + (p 2)Y = 2 + 1 p 1 Gii h ny ta c: 1 1 ; Y= X= p 1 p 1 t Vy: x(t) = e v y(t) = et V d 9: Gii h phng trnh: x x + y + z = 0 x + y y + z = 0 x + y + z z = 0 Tho mn cc iu kin u x(0) = 1, y(0) = z(0) = x(0) = y(0) = z(0) = 0. t x(t) X(p) x p2X - p y(t) Y(p) y p2Y z(t) Z(p) z p2Z
124

Do h phng trnh i vi cc nh l: (p 2 1)X + Y + Z = p 2 X + (p 1)Y + Z = 0 X + Y + (p 2 1) Z = 0 Gii h ny ta c: p3 (p 2 + 1)(p 2 2) p Y=Z= 2 (p + 1)(p 2 2) Nh vy: 2 1 x ( t ) = ch 2t + cos t 3 3 1 1 y( t ) = z( t ) = ch 2t + cos t 3 3 X=

( )

( )

2. Dng cng thc Duhamel: Nu bit nghim x1(t) ca phng trnh: + a 1x1 + a 2 x1 = 1 a o x1 (5) tho mn cc iu kin ban u thun nht x(0) = x(0) = 0 th cng thc m ta thit lp di y da vo cng thc Duhamel s cho ta nghim x(t) ca phng trnh: (6) aox + a1x + a2x = f(t) tho mn cc iu kin ban u thun nht x(0) = x(0) = 0. Ta c cng thc:

* f = f ()x1 ( t )d = f ( t )x1 ()d x ( t ) = x1


0 0

Chng minh: t x1(t) X1(p), x(t) X(p), f(t) F(p). Hm X1(p) tho mn phng trnh nh ca (5) l: 1 (7) ( a o p 2 + a 1p + a 2 ) X1 ( p ) = p Hm X(p) tho mn phng trnh nh ca (6) l: (a o p 2 + a 1p + a 2 )X (p) = F(p) (8) T (7) v (8) suy ra: X ( p) pX1 (p) = hay X(p) = pX1 (p).F(p) F(p) Theo cng thc tch phn Duhamel ta c: *f X(p) x1(t).f(0) + x1 *f V x1(0) = 0 nn X(p) x1 ngha l:
125

* f = f ()x1 ( t )d = f ( t )x1 ()d x ( t ) = x1


0 0

(9)

Ta cng c th dng cng thc Duhamel th 2:

x ( t ) = x1 ( t )f (0) = x1 ()f ( t )d
0

(10)

V d 1: Tm nghim ca phng trnh: 2 x + x = e t tho mn iu kin u x(0) = x(0) = 0. Ta thy nghim ca phng trnh x + x = 1 vi iu kin u x(0) = x(0) = 0 l x1(t) = 1 - cost. Vy theo (9) th nghim ca phng trnh ban u l:

x ( t ) = e ( t ) sin d
V d 2: Tm nghim ca phng trnh x + x = 5t2 vi iu kin u l x(0) = x(0) =0 Trong v d trn ta c x1(t) = 1 - cost. Vy:
0

x ( t ) = 5t 2 sin( t )d = 5( t 2 2 + 2 cos t )
0

20. BNG I CHIU NH - GC

Tt 1 2 3 4 5 6 7 8

f(t) 1 t tn eat eat - 1 teat tneat sinmt

F(p)
1 p

Tt 21 22 23 24 25 26 27 28

f(t) teatcosmt teatshmt teatchmt 1-cosmt f(t)sinmt f(t)cosmt f(t)shmt f(t)chmt

1 p2 n! p n +1
1 pa a p( p a ) 1 (p a ) 2 n! (p a ) n +1 m 2 p + m2

F(p) (p a ) 2 m 2 [(p a ) 2 + m 2 ]2 2m( p a ) [(p a ) 2 m 2 ]2 (p a ) 2 + m 2 [(p a ) 2 m 2 ]2 m2 p( p 2 + m 2 )


1 [ F( p jm) F( p + jm)] 2 1 [ F( p jm) + F(p + jm)] 2 1 [ F( p m ) F( p + m)] 2 1 [ F( p m ) + F( p + m)] 2
126

9 10 11

cosmt shmt chmt

p 2 p + m2

29 30 31 32 33 34 35 36 37 38 20

e at e bt ab

1 (p a )(p b)

m 2 p m2
2

e ab

t a

t b

1 (ap + 1)(bp + 1)
p (p a ) 2 1 (p a )p 2

p p m2 m 12 eatsinmt (p a ) 2 + m 2 pa 13 eatcosmt (p a ) 2 + m 2

(1+at)eat
e a at 1 a2

cos mt sin2mt ch2mt sh2t


e at e bt t e at t

14 15 16 17 18 19

eatshmt eatchmt tsinmt tcosmt tshmt tchmt

m (p a ) 2 m 2 pa (p a ) 2 m 2 2pm (p 2 + m 2 ) 2 p2 m2 (p 2 + m 2 ) 2 2pm 2 (p m 2 ) 2 p2 + m2 (p 2 m 2 ) 2

p 2 + 2m 2 p( p 2 + 4 m 2 ) 2m 2 p( p 2 + 4 m 2 ) p 2 2m 2 p( p 2 4 m 2 ) 2m 2 p( p 2 4 m 2 ) pb ln pa 1 p+a 2m( p a ) [(p a ) 2 + m 2 ]2

teatsinmt

127

CHNG 7: PHNG TRNH VT L - TON


1. PHN LOI CC PHNG TRNH O HM RING TUYN TNH CP 2 VI CC BIN C LP 1. Phn loi cc phng trnh: Khi kho st cc bi ton vt l, ta nhn c phng trnh o hm ring cp 2 dng: n n 2u u a ( x ) + (1) i, j x y bi ( x ) x + c(x )u = d(x ) i , j=1 i 1 = i j i Trong aij(x), bi(x), c(x) v d(x) l cc hm nhiu bin cho ca x = (x1, x2,...xn) cn u(x) l cc hm cn xc nh. Trong thc t ta thng gp cc phng trnh o hm ring tuyn tnh cp 2 vi hai bin c lp dng: 2u 2u 2u u u (2) a 2 + 2b + c 2 + d + e + gu = h x xy y x y Trong a, b, c, d, g, h l cc hm hai bin ca x v y. Trong gio trnh ny ta ch xt cc phng trnh dng (2). n gin ta vit li (2): u u 2u 2u 2u , x , y , u , a 2 + 2b + c 2 + (3) =0 y x y xy x Cc phng trnh ny c th phn thnh cc loi sau: Phng trnh hyperbolic: u u 2u = 1 x, y, u, , x y xy Phng trnh eliptic: u u 2u 2u + 2 = 2 x, y, u, , 2 x y y x Phng trnh parabolic: u u 2u = , x , y , u , 3 y x 2 x 2. Cc bi ton c bn ca phng trnh vt l - ton: a. Bi ton Cauchy v bi ton hn hp ca phng trnh truyn sng: Mt phng trnh truyn sng l mt phng trnh dng hyperbolic. Phng trnh truyn sng dng chnh tc l: 2 2u 2u 2 u ( x, y, z, t ) 2 u =a + f1 (x, y, z, t ) x 2 + y 2 + z 2 t 2

Gi s ta cn xc nh hm u(x, y, z, t) trong min V v t 0. V c gii hn bng mt bin kn v trn S vi cc iu kin u: u ( x, y, z, t ) t =0 = u o ( x, y, z)


u = u o ( x , y, z ) t t =0 v iu kin bin:

150

u ( x, y, z, t ) ( x ,y ,z )S = u1 ( x, y, z)
Bi ton gii phng trnh trn vi cc iu kin u v iu kin bin c gi l bi ton hn hp ca phng trnh truyn sng. Nu ta xt bi ton trong min cch xa cc bin m iu kin bin khng c tc dng th ta gp bi ton Cauchy vi iu kin u v xt trong ton b khng gian. b. Bi ton Cauchy v bi ton hn hp ca phng trnh truyn nhit: Cho phng trnh truyn nhit di dng chnh tc: 2 u ( x, y, z, t ) 2u 2u 2 u =a x 2 + y 2 + z 2 + f1 (x, y, z, t ) t Khi bi ton hn hp ca phng trnh truyn nhit l bi ton tm nghim ca phng trnh vi iu kin u v iu kin bin: u ( x, y, z, t ) t =0 = u o ( x, y, z)

u ( x, y, z, t ) ( x ,y ,z )S = u1 ( x, y, z)
Bi ton Cauchy ca phng trnh truyn nhit l bi ton tm nghim ca phng trnh truyn nhit trong ton b khng gian.
2. PHNG TRNH LOI HYPERBOLIC PHNG TRNH TRUYN NHIT 1. Bi ton Cauchy - Phng trnh sng ca dy v hn v na v hn: Bi ton Cauchy ca phng trnh hyperbolic trong trng hp mt bin c xc nh nh sau: 2 2 u ( x, t ) 2 u =a - x , t 0 (1) x 2 t 2 vi cc iu kin u u ( x , t ) t =0 = u o ( x ) ; = u1 ( x ) (2) t t =0 y l bi ton dao ng t do ca dy di v hn. gii phng trnh (1) ta bin i n bng cch dng cc bin: = x + at (3) = x at ngha l: + t= x= 2a 2 Ta c: ~ u ~ u ~ u = + x ~ ~ u u u ~ = a t 2~ 2~ u u 2~ u 2~ u = + + 2 2 2 2 x
151

2~ u 2~ u 2~ 2~ u 2 u =a 2 + 2 2 2 t Thay vo (2.1) ta c: 2~ u u ~ = 0 hay =0 ~ u Suy ra: = 1() vi 1() l hm tu Nh vy: ~ u (, ) = 1 ( )d + ()

vi () l hm tu . T ta c: ~ u (, ) = () + () hay: u(x, t) = (x + at) + (x - at) (3) Trong v l cc hm tu , lin tc v kh vi 2 ln. Nghim ca (3) c gi l nghim tng ca (1). T (3) nu tnh n iu kin (2) ta s c: (4) (x) + (x) = uo(x) a(x) - a(x) = u1(x) (5) Ly tch phn hai v ca (5) ta c:

a[( x ) (0)] a[( x ) (0)] = u1 ()d


0

Vy nn:

1x ( x ) ( x ) = u1 ()d + C a0 vi C = (0) - (0) T (4) v (6) rt ra: 1 1 x C = + + ( x ) u ( x ) u ( ) d o 1 2 2a 0 2 x ( x ) = 1 u ( x ) 1 u ()d C o 1 2 2a 0 2 t cc h thc trn vo (3) ta c nghim: 1 x +at 1 u ( x, t ) = [u o ( x + at ) + u o ( x at )] + u1 ()d 2a x at 2 y l cng thc DAlembert. V d 1: Gii phng trnh: 2 2u 2 u =a - < x < , t 0 t 2 x 2 vi cc iu kin: x u ( x , t ) t =0 = = u o (x) 1+ x2

(6)

152

u = sinx = u1 ( x ) t t =0 p dng cng thc DAlembert ta c: 1 x + at x + at 1 + sinatsinx u ( x, t ) = + 2 1 + ( x + at ) 2 1 + ( x at ) 2 a


V d 2: Gii phng trnh: 2 2u 2 u =a - < x < , t 0 t 2 x 2 vi cc iu kin: sin x u ( x , t ) t =0 = = u o (x) x u 1 = = u1 ( x ) t t =0 1 + x 2 p dng cng thc DAlembert ta c: 2at xsinxcosat atcosxsinat 1 + arctg u ( x, t ) = 2 2 2 2 x (at ) 2a 1 + x (at ) v nu t arctg(x + at) - arctg(x - at) = ta c: tg[arctg( x + at )] tg[arctg( x at )] ( x + at ) ( x at ) = tg = 1 + tg[arctg( x + at )] tg[arctg( x at )] 1 + ( x + at )( x at ) 2at = 2 1 + x (at ) 2 V d 3: Gii phng trnh: 2 2u 2 u = a - < x < , t 0 t 2 x 2 vi cc iu kin: u ( x , t ) t =0 = x 2 = u o ( x )

u = sin 2 x = u1 ( x ) t t =0 Trc ht tm nghim ca bi ton ta ko di l hm uo(x) = x2 v u1(x) = sin2x s c cc hm: x 2 x0 uo = 2 x x < 0 u ( x , t ) x =0 = 0

sin 2 x x0 u = 2 sin x x < 0 p dng cng thc DAlembert cho cc hm ny ta c:


1

153

u ( x, t ) =

1 1 x +at + u o ( x + at ) + u ( x at ) o u1 ()d 2 2a x at

1 1 x +at 2 2 2 2 ( x + at ) + ( x at ) + 2a sin ()d x at = x + at 0 1 ( x + at ) 2 ( x at ) 2 + 1 sin 2 ()d sin 2 ()d 2a x at 0 2 t 1 x 2 2 2 + + x a t cos 2 x sin 2 at t 2 4a a = 2axt + 1 [2 x sin2 xcos2at ] 4a

[ [

t t>

x a x >0 a

V d 4: Gii phng trnh in bo: 2 u RC + LG u RG 1 2u + + u =0 t 2 LC t LC LC x 2 2i RC + LG i RG 1 2i + + i =0 t 2 LC t LC LC x 2 Trong cc trng hp: - Dy khng tn hao R = G = 0 - Dy khng mo RC = LG ) Trng hp dy khng tn hao: Khi cc phng trnh trn c dng: 2 2u 2 u a = x 2 t 2 2 2i 2 i a = x 2 t 2 1 Trong a = LC Gi s cc iu kin ban u bit l: u ( x , t ) t = 0 = u o ( x ) - < x < , y > 0 i ( x , t ) i ( x ) = o t =0 Vi R = G = 0 ta c iu kin i vi phng trnh in bo: u 1 = io(x) t t =0 C i 1 = u o (x) t t =0 C T p dng cc cng thc DAlembert ta c: 1 x +at 1 u ( x, t ) = u o ( x + at ) + u o ( x at ) i o ()d 2aC x at 2

154

1 x +at 1 i o ( x + at ) + i o ( x at ) u o ()d 2aC x at 2 1 Nu tnh n a = ta suy ra nghim: LC L i o ( x at ) i o ( x + at ) 1 u ( x, t ) = u o ( x + at ) + u o ( x at ) + C 2 2 i( x , t ) =

i( x , t ) =

1 L u o ( x at ) u o ( x + at ) i o ( x + at ) + i o ( x at ) + 2 C 2

) Trng hp dy khng mo: khi RC = LC v ta tm nghim di dng: R u ( x ,t ) t~ u ( x , t ) = e L R ~ i( x , t ) = e L t i ( x ,t ) Ly o hm h thc trn hai ln theo x v theo t ri thay vo phng trnh ta c: ~ 2~ 2~ 2~ u 2 i 2 u 2 i =a ; =a 2 x 2 t 2 x 2 Cc iu kin u: ~ u ( x , t ) t = 0 = u ( x , t ) t = 0 = u o ( x ) ~ i ( x , t ) t =0 = i ( x , t ) = i o ( x ) t =0 u 1 ~ = i o(x) t C t =0 ~ 1 i = u o(x) t L t =0 T , theo cng thc DAlembert ta c nghim: R t u ( x + at ) + u ( x at ) L i o ( x at ) i o ( x + at ) o o + u ( x , t ) = e L 2 C 2 R t i ( x + at ) + i ( x at ) L u o ( x at ) u o ( x + at ) o o L = + i ( x , t ) e 2 C 2 2. Bi ton hn hp - Phng trnh sng ca dy hu hn: a. Khi nim chung: Bi ton hn hp ca phng trnh loi hyperbolic trong trng hp mt chiu l bi ton gii phng trnh: 2 2u 2 u =a + f (x, t ) 0 x l, 0 t T t 2 x 2 u vi cc iu kin : u ( x , t ) t =0 = u o ( x ); = u1 ( x ) t t =0
155

u ( x, t ) x =0 = 1 ( t ); u ( x , t ) x =l = 2 ( t ) Ta phn bi ton hn hp ny thnh cc bi ton nh sau: b. Bi ton 1: Gii phng trnh: 2 2u 2 u =a 0 x l, 0 t T x 2 t 2 vi cc iu kin: u = u1 ( x ) u ( x , t ) t =0 = u o ( x ); t t =0
u ( x , t ) x =0 = 0; u ( x , t ) x =l = 0 Bi ton ny m t qu trnh truyn sng ca dy hu hn vi hai u dy c nh. Bit dng ban u ca dy l uo(x) v vn tc ban u ca cc thnh phn dy l u1(x). Ta gii bi ton ny bng phng php tch bin, ngha l tm nghim ca phng trnh di dng tch hai hm s, mt hm ch ph thuc vo to x v hm kia ch ph thuc t. Nh vy nghim u(x,t) c dng: u(x,t) = X(x).T(t) Sau khi ly o hm v thay vo phng trnh ta c: 1 T( t ) X( x ) = a 2 T( t ) X(x ) Do v phi ch ph thuc t v v tri ch ph thuc x nn chng phi cng bng mt hng s m ta k hiu l -. Khi ta nhn c h phng trnh: X( x ) + X( x ) = 0 2 T( t ) + a T( t ) = 0 Nghim ca bi ton phi tho mn iu kin cho nn: X(0) = 0; X(l) = 0 Khi gii phng trnh vi phn tuyn tnh cp 2 h s hng ta c nhn xt v gi tr ca nh sau: * Nu = 0 th nghim tng qut ca phng trnh c dng: X(x) = C1(x) + C2 Vi iu kin u ta suy ra C1 = 0 v C2 = 0. Khi ny X(x) 0 khng c coi l nghim ca bi ton. * Nu < 0 th nghim tng qut c dng: X( x ) = C1e x + C 2 e x v vi cc iu bin ta c: X(0) = C1 + C 2 = 0 l + C 2 e l = 0 X(l) = C1e T h trn ta suy ra C1 = 0 v C2 = 0. Khi ny X(x) 0 khng c coi l nghim ca bi ton. Nu > 0 th nghim tng qut c dng: X( x ) = C1 cos x + C 2 sin x v vi cc iu bin ta c:
156

X(0) = C1 = 0 X(l) = C 2 sin l = 0


nghim khng tm thng th t phng trnh trn ta thy C2 0, suy ra sin l = 0 . Nh vy: k 22 l = hay = 2 k Z l 2 nn: k X ( x ) = C 2 sin x l Vi gi tr va tm c gii phng trnh ta c: k k Tk = A k cos at + Bk sin at l l Do : k k k u k ( x, t ) = a k cos at + b k sin at sin x l l l Nghim tng qut c dng: k k k u ( x, t ) = u k ( x, t ) = a k cos at + b k sin at sin x l l l k =1 k =1 Vn cn li l xc nh cc h s ak v bk tho mn cc iu kin u v iu kin bin, ngha l phi c: u u ( x , t ) t =0 = u o ( x ); = u1 ( x ) t t =0 Ta gi s cc hm uo(x) v u1(x) l cc hm c th khai trin thnh chui Fourier theo k sin x trn on [0, l]. Khi ta c: l k u ( x, t ) t =0 = a k sin x = u o ( x ) l k =1 Do : k 2l a k = u o ( x ) sin xdx l l0 u ka k = b k sin x = u1 ( x ) v: t t =0 k =1 l l k 2 l u1 ( x ) sin xdx nn: b k = l ka 0 V d : Tm dao ng gn cht ti hai mt x = 0 v x = l nu v tr ban u ca dy trng vi trc Ox v vn tc ban u ca cc thnh phn dy c cho bi hm s: l 2 v khi x o 3 h u = t t =0 l 0 khi x > 3 2h
157

(0 x < l, t > 0), h l hng s sao cho x tho mn x (0, l). Nh vy ta cn gii phng trnh: 2 2u 2 u = a x 2 t 2 vi iu kin u cho v iu kin bin: u ( x , t ) t =0 0 ; u ( x , t ) x =l = u ( x , t ) x =l = 0

l cha trong khong 3 2h

Nh vy, v uo(x) 0 nn: ak 0 4v o l 2 l k 2 l k k 2 h bk = u1 ( x ) sin l xdx = ka v o sin l xdx = k 2 2a sin 2 ka 0 0 2 k k h sin sin 4v l 3 2 sin kat sin kx v: u ( x, t ) = 2 o 2 a k =1 k l l
c. Bi ton 2: Gii phng trnh 2 2u 2 u + f (x, t ) =a 0 x l, 0 t T x 2 t 2 vi cc iu kin: u = u1 ( x ) u ( x , t ) t =0 = u o ( x ); t t =0
u ( x , t ) x =0 = 0; u ( x , t ) x =l = 0 Bi ton ny m t qu tnh truyn sng ca dy hu hn c tc ng ca lc cng bc bn ngoi vi hai u dy c nh. Dng ban u ca dy l uo(x) v vn tc ban u ca dy cho bi u1(x). Ta cng gii bi ton bng phng php phn ly bin s Fourier. Ta tm nghim di dng: kx u ( x , t ) = Tk ( t ) sin (1) l k =1 Ta gi s cc hm uo(x) v u1(x) khai trin c di dng chui Fourier theo sin trong khong [0, l], khi ta c: kx u o ( x ) = Tk (0) sin l k =1 l 2 k hay: Tk (0) = u o ( x ) sin xdx l0 l kx (0) sin u1 ( x ) = Tk l k =1 l 2 k (0) = u o ( x ) sin xdx hay: Tk l0 l Mt khc ly o hm hai ln u(x, t) trong (1) theo x v t ta c:
158

2u kx ( t ) sin t 2 = Tk l k =1 2 2 k kx u = T ( t ) sin k 2 l k =1 l x Khai trin hm f(x, t) theo sin: kx f ( x , t ) = C k sin l k =1 Trong : 2l k C k = f ( x, t ) sin xdx l0 l t cc iu kin trn vo phng trnh ca u(x, t) ta c: 2 ak kx + T ( t ) T ( t ) C ( t ) =0 sin k k k l l k =1 T suy ra Tk(t) trong (1) l nghim ca phng trnh tuyn tnh cp 2 h s hng: 2 ak ( t ) + Tk Tk ( t ) = C k ( t ) l tho mn cc iu kin u v iu kin bin V d 1: Gii phng trnh 2u 2u = +1 0 x 1, 0 t t 2 x 2 vi cc iu kin: u =1 u ( x , t ) t =0 = 0; t t =0
u ( x , t ) x =0 = u ( x , t ) x =l = 0

Ta tm nghim di dng (1). Trong v d ny f(x, t) 1. Nh vy: 1 2l k 2 C k = f ( x, t ) sin xdx = 2 sin kxdx = (1 cos k ) l0 l k 0 4 khi k = 2n 1 hay: C k = k khi k = 2n 0 Mt khc u1(x) 1, uo(x) 0 nn ta suy ra: 4 l l khi k = 2n 1 Tk = 2 u1 ( x ) sin kxdx = 2 sin kxdx = k 0 0 0 khi k = 2n Vy vi k chn ta phi gii phng trnh vi phn thng: n ( t ) + (2n) 2 T2 n ( t ) = 0 T2 n (0) = 0 vi iu kin: T2n(0) = 0; T2 Nh vy T2n(t) 0
159

Vi k l ta phi gii phng trnh vi phn tng ng l: 4 n 1 ( t ) + [(2n 1) ] 2 T2 n 1 ( t ) = T2 (2n 1) 4 n 1 (0) = vi iu kin: T2n-1(0) = 0; T2 (2n 1) Nghim tng qut ca phng trnh ny l:
T2 n 1 ( t ) = C1 cos(2n 1) t + C 2 sin( 2n 1) t + 4 (2n 1) 3 3

Khi t = 0 ta c: 4 (2n 1) 3 3 Mt khc ta c: n 1 ( t ) = C1 (2n 1) sin(2n 1) t + C 2 (2n 1) cos(2n 1) t T2 Theo iu kin u: C1 = n 1 ( t ) = C1 (2n 1) sin( 2n 1) t + C 2 (2n 1) cos(2n 1) t = T2 nn: C 2 = 4 (2n 1) 3 3

4 (2n 1) 2 2 Thay C1 v C2 vo biu thc ca T2n-1(t) ta c: 4 [(2n 1) sin(2n 1)t cos(2n 1)t + 1] T2 n 1 ( t ) = (2n 1) 3 3 v: 4 (2n 1) x [ ] + u ( x, t ) = ( 2 n 1 ) sin( 2 n 1 ) t cos( 2 n 1 ) t 1 sin (2n 1) 3 3 n =1 l

V d 2: Gii phng trnh 2u 2u = + x ( x 1) t 2 x 2 vi cc iu kin: u =0 u ( x , t ) t =0 = t t =0

0 x 1, 0 t

u ( x , t ) x =0 = u ( x , t ) x =1 = 0 Trong v d ny ta c f(x, t) = x(x - 1). Vy: 1 2l k C k = f ( x, t ) sin xdx = 2 x ( x 1) sin kxdx l0 l 0

1 1 2 cos kx = 2( x x ) + (2x 1) cos kxdx k x =0 k 0

x =1

160

8 khi k = 2n 1 4 k nn: C k = 2 (1) 1 = (2n 1) 3 3 k 0 khi k = 2n Ta tm nghim ca bi ton di dng (1) nn by gi phi tm Tk(t) Vi k = 2n (chn), ta tm T2n(t) t phng trnh : n ( t ) + (2n) 2 T2 n ( t ) = 0 T2 n (0) = 0 vi iu kin: T2n(0) = 0; T2 Nh vy T2n(t) 0 Vi k = 2n -1 (l) ta phi gii phng trnh vi phn tng ng l: 8 n 1 ( t ) + [(2n 1) ] 2 T2 n 1 ( t ) + T2 =0 (2n 1) 3 3 n 1 (0) = 0 vi iu kin: T2n-1(0) = 0; T2 Nghim tng qut ca phng trnh ny l: 8 T2 n 1 ( t ) = C1 cos( 2n 1) t + C 2 sin( 2n 1) t (2n 1) 5 5 Khi t = 0 th t cc iu kin u ta rt ra: 8 C1 = C2 = 0 (2n 1) 5 5 8 [cos(2n 1) t 1] T2 n 1 ( t ) = (2n 1) 5 5 8 u ( x, t ) = [cos(2n 1) t 1]sin( 2n 1) x (2n 1) 5 5 n =1

d. Bi ton hn hp: Sau khi gii 2 bi ton trn ta tr v gii phng trnh: 2 2u 2 u = a + f (x, t ) 0 x l, 0 t T (1) t 2 x 2 vi cc iu kin : u u ( x , t ) t =0 = u o ( x ); = u1 ( x ) t t =0
u ( x , t ) x =0 = 1 ( t ); u ( x , t ) x =l = 2 ( t ) Ta gii bi ton bng cch a vo hm ph: x ( x , t ) = 1 ( t ) + [ 2 ( t ) 1 ( t )] l Khi ta tm nghim ca bi ton hn hp di dng: u ( x, t ) = ~ u ( x , t ) + ( x , t ) ~ Trong hm u ( x, t ) ta phi xc nh. Trc ht ta c nhn xt: ( x, t ) x =0 = 1 ( t ) ( x, t ) x =l = 2 ( t )

(2)

Vy kt hp vi iu kin cho ta c: ~ u ( x , t ) x =0 = ~ u ( x , t ) x =l = 0

(3)
161

Khi t = 0 ta s c: x ~ [ ] [ 2 (0) 1 (0)] = ~ = = u ( x , t ) u ( x , t ) ( x , t ) u ( x ) ( 0 ) u o (x) o 1 t =0 t =0 l (4) ~ u u x (0) [2 (0) 1 (0)] = ~ = = u1 ( x ) 1 u o (x) l t t =0 t t =0 t t =0 o hm 2 ln (2) theo x v t ri thay vo (1) v rt gn ta c: 2~ 2~ u 2 u = a + f1 ( x , t ) (5) t 2 x 2 Trong : x ( t ) [2 ( t ) 1 ( t )] f1 ( x , t ) = f ( x , t ) 1 l Tm li, tm u(x, t) ta phi gii (5) vi cc iu kin (3) v (4). chnh l dng u ( x, t ) v (x, t) ta tm c bi ton 2 m ta bit cch gii. Sau kt hp ~ nghim.
3. BI TON CAUCHY CA PHNG TRNH TRUYN SNG TRONG KHNG GIAN Bi ton Cauchy ca phng trnh truyn sng trong khng gian l bi ton gii phng trnh: 2 2u 2u 2u 2 u =a (1) x 2 + y 2 + z 2 t 2 vi cc iu kin: u ( x , y, z, t ) t =0 = u o ( x , y, z) - < x < , - < y <, - < z < , t > 0 (2) u = u ( x , y , z ) 1 t t =0 Ngi ta chng minh c rng nghim ca phng trnh c dng: 1 1 + u ( x, y, z, t ) = u ( , , ) ds u ( , , ) ds 1 o 2 t 4a 2 t S 4 a t S Trong S l mt cu tm M(x,y,z) v bn knh at. Cng thc ny gi l cng thc Kirhoff. Trong trng hp mt phng, cng thc Kirhoff tr thnh cng thc Poisson: u o (, )dd 1 1 u1 (, )dd + u ( x , y, t ) = 2 2 2 2 a D (at ) 2 ( x ) 2 ( y) 2 t 2 a ( at ) ( x ) ( y ) D 4. BI TON DAO NG CNG BC 1. Nguyn l Duhamel: gii cc bi ton c tc ng ca ngoi lc ngi ta thng dng nguyn l Duhamel c pht biu nh sau: Nu H(, x, t) vi mi gi tr ca tham bin l nghim ca phng trnh: 2H = a 2 il 2 t
162

vi cc iu kin: H ( , x , t ) t = 0 = 0 t H(, x , t ) = h ( x, ) t =0 Khi hm:

u ( x, t ) = H(, x, t )d
0

s l nghim ca phng trnh: 2u = a 2 u + h ( x , t ) 2 t vi cc iu kin: u ( x , t ) t = 0 = 0 u =0 t t =0 hiu r hn v nguyn l Duhamel ta s dng n gii cc bi ton v dao ng cng bc sau: 2. Bi ton 1: Gii phng trnh: 2 2u 2u 2u 2 u =a (1) + f ( x, y, z, t ) x 2 + y 2 + z 2 t 2 vi cc iu kin: u ( x , y, z, t ) t =0 = u o ( x , y, z) - < x < , - < y <, - < z < , t > 0 (2) u = u ( x , y , z ) 1 t t =0 Ta dng phng php chng nghim, ngha l tm nghim ca phng trnh (1) di dng: u ( x, y, z, t ) = u ( x , y, z, t ) + u ( x , y, z, t ) Trong u ( x , y, z, t ) l nghim ca bi ton:
2 u = a 2u 2 t u u t =0 = u o ; = u1 t t =0

vi:

Cn u ( x , y, z, t ) l nghim ca bi ton:
2 u = a 2u + f 2 t

vi:

t =0

= 0;

u t

=0
t =0

Theo cng thc Kirhoff ta c:


163

1 1 u ( , , ) ds u ( , , ) ds + 1 2 o 4a 2 t S t 4a t S Mt khc theo nguyn l Duhamel ta c: 1 f (, , , ) ds H ( , x , y, z , t ) = 4 a 2 S t T suy ra: 1 l f (, , , ) u ( x , y, z , t ) = t ds d 4 a 2 0 S( t ) rt gn cng thc nghim trong tch phn trn ta i bin r = a(t - ). Do ta c: r r f , , , t f , , , t l 1 1 a a dV = ds dr u ( x, y, z, t ) = 2 2 r r 4a Vat 4a 0 S( r ) Trong Vat l hnh cu bao bi mt S v: r = ( x ) 2 + ( y ) 2 + (z ) 2 Vy nghim ca bi ton 1 l: r f , , , t 1 u1 (, , ) u (, , ) a u ( x, y, z, t ) = ds + o ds + dV 2 4a S t t r t S Vat Cng thc ny c gi l cng thc Kirhoff tng qut. u ( x, y, z, t ) =
3. Bi ton 2: Gii phng trnh: 2 2u 2u 2 u = + a (1) x 2 y 2 + f ( x, y, t ) t 2 vi cc iu kin: u ( x , y, t ) t = 0 = u o ( x , y ) - < x < , - < y <, t > 0 (2) u u ( x , y ) = 1 t t =0 Nghim ca bi ton rt ra nh cch gii tng t nh bi ton trc bng cch dng nguyn l Duhamel: 1 u1 (, ) dd u ( x, y, t ) = 4a 2 S a 2 t 2 ( x ) 2 ( y ) 2

l u o (, ) f (, , )dd d d + 2 2 + d 2 2 2 2 2 2 t a t ( x ) ( y ) a ( t ) ( x ) ( y ) D 0 D a a ( t )
Trong Dat v Da(t-) l min trn c cng tm (x, y) v bn knh l at v a(t-) . Cng thc ny c gi l cng thc Poisson tng qut.
4. Bi ton 3: Gii phng trnh:
164

2 2u 2 u + f (x, t ) =a (1) x 2 t 2 vi cc iu kin: u ( x , t ) t = 0 = u o ( x ) - < x < , t > 0 (2) u u ( x ) = 1 t t =0 Da trn nguyn l Duhamel v cng thc DAlembert ta a n nghim bi ton: 1 l x + a ( t ) 1 1 at [ ] + + + + f ( , ) d u ( x at ) u ( x at ) u ( ) d u ( x, t ) = d o o 1 2a 0 x a ( t ) 2 2a x at

y l cng thc DAlembert tng qut.


5. BI TON HN HP CA PHNG TRNH LOI HYPERBOLIC Cho D l mt min phng vi bin l ng cong trn. Ta cn tm nghim ca phng trnh: 2 2u 2u 2 u =a (1) x 2 + y 2 t 2 vi cc iu kin u : u ( x , y, t ) t = 0 = u o ( x , y ) - < x < , - < y <, t > 0 (2) u u ( x , y ) = 1 t t =0 v iu kin bin: u ( x, y, t ) ( x ,y ) = 0 (3)

Bi ton ny ta gii bng phng php phn ly bin s v s tm nghim ca n di dng: u(x, y, t) = u*(x, y).T(t) (4) o hm 2 v ca (4) theo x, y v t hai ln ri thay vo (10) ta nhn c phng trnh: 1 2 u 2 u 1 T 2 + 2 = 2 u y x a T T suy ra: 2u 2u (5) + 2 + u = 0 2 x y (6) T( t ) + a 2 T ( t ) = 0 Trong l mt hng s. tm nghim ca bi ton (1) vi cc iu kin (2), (3) ta thy rng T(t) 0 v i vi cc im trn bin ta phi c: u ( x, y) ( x ,y ) = 0 (7)
Nhng gi tr tn ti nghim u*(x, y) 0 c gi l cc gi tr ring v cc
165

nghim u*(x, y) tng ng c gi l cc hm ring ca bi ton. Tnh cht ca gi tr ring v hm ring l: * Mi gi tr ring u dng * Tp cc gi tr ring l mt tp v hn m c * Nu i j th cc hm ring tng ng vi chng tho mn h thc: u i ( x, y)u j ( x, y)dxdy = 0
D

ngha l cc hm ring trc giao vi nhau * Mt gi tr ring c th ng vi nhiu hm ring c lp tuyn tnh khc nhau. Gi tr ring nh vy c gi l gi tr ring bi * i vi cc hm ring nu cha l h trc chun th bng phng php trc giao ho Schmidt c th xy dng h hm ring trc giao chun, ngha l i vi h ta c: i j 0 u ( x , y ) u ( x , y ) dx dy = j i i= j D 1 * Mi hm (x, y) kh vi, lin tc 2 ln v tho mn iu kin bin: ( x , y) ( x ,y ) = 0 u c th khai trin theo h thng cc hm trc giao chun thnh chui hi t tuyt i v u trn min D, ngha l n c th biu din di dng:
( x , y) = a k u k ( x , y)
k =1

trong ak c tnh theo cng thc: a k = ( x , y)u k ( x , y)dxdy


D

T nhng tnh cht nu ca hm ring v gi tr ring ta thy bi ton (5) & (6) c cc gi tr ring dng nn (6) c nghim tng qut l: Tk ( t ) = b k cos k at + c k sin k at T suy ra: u ( x , y, t ) = u k ( x , y)Tk ( t ) = u k ( x, y) b k cos k at + c k sin k at
k =1 k =1

Nu xt n cc iu kin ban u ta c: b k = u o ( x , y) ~ u k ( x , y)dxdy


D

1 u1 ( x, y)u k ( x, y)dxdy a k D V d 1: Gii phng trnh: 2 2u 2u 2 u =a x 2 + y 2 t 2 0 x l t0 trn min D = 0 y m vi cc iu kin u: ck =


166

u ( x , y, t ) t =0 = xy(l x )(m y) = u o ( x , y) u = 0 = u1 ( x , y) t t =0 v cc iu kin bin: u ( x, y, t ) ( x ,y ) = 0

trong l bin ca min D. Ta tm nghim ca bi ton di dng (4): u(x, y, t) = u*(x, y).T(t) trong u*(x, y) li c tm di dng u*(x, y) = X(x).Y(y) bng phng php phn li bin s. Khi phng trnh (5) c vit thnh: Y( y) X( x ) += Y ( y) X( x ) T ta suy ra: X( x ) + X( x ) = 0 (8) Y( y) + Y( y) = 0 T iu kin bin ca bi ton ta rt ra: X(0) = X(l) = 0 (9) + = Y ( 0 ) Y ( l ) 0 Khi gii phng trnh (8) vi iu kin (9) c nghim khng tm thng ta cn c: 2n 2 k 22 k = 2 ; n = 2 m l T ta c: 2 n2 2 k kn = l2 + m2 Nghim ring ng vi cc gi tr ring l h hm trc chun: kx ny 2 sin sin u kn = l m lm Khi nghim ca phng trnh (6) c dng: k2 n2 k2 n2 Tk ( t ) = b kn cos at 2 + 2 + c kn sin at 2 + 2 l m l m Tm li nghim ca bi ton s l: k2 n2 k2 n2 kx ny u ( x , y, t ) = b kn cos at 2 + 2 + c kn sin at 2 + 2 sin sin l m l m l m k =1 n =1 Trong bkn v ckn c tnh nh sau: ckn = 0 k, n v u1 0

167

b kn =

4 lm kx ny xy(l x )(m y) sin sin dxdy lm 0 0 l m

4 l kx m ny = x ( l x ) sin dx y ( m y ) sin dy lm 0 l m 0 64m 2 l 2 = 6 (2k + 1) 3 (2n + 1) 3 0 Nh vy: k = 2k + 1, n = 2n + 1 k = 2k1

(2n + 1) y (2k + 1) x 64m 2l 2 cos at kn sin sin u ( x, y, t ) = 6 3 3 m l k =1 n =1 ( 2k + 1) ( 2n + 1)


V d 2:Gii phng trnh: 2 2u 2u 2 u =a x 2 + y 2 t 2

0 x l trn min D = t0 0 y m vi cc iu kin u: y x = = sin u ( x , y , t ) u ( x , y ) sin o t =0 l l u = u1 ( x , y) = a l t t =0 v cc iu kin bin: u ( x, y, t ) ( x ,y ) = 0


trong l bin ca min D. Tng t nh v d 1, sau khi dng phng php phn li bin s ta tm c gi tr ring l: 2 n2 2 k kn = l2 + l2 2 k x n y h hm trc chun tng ng l sin v cc h s trong nghim tng sin l l l qut c tnh nh sau: 0 k, n 1 4 ll x y kx ny b kn = 2 sin sin sin sin dxdy = k = n =1 l 00 l l l m 1 ny a x y 4 ll dxdy k 2 + n 2 c kn = 2 sin sin sin m l l l 00 l

168

k , n chan 0 4a l ny x l y dx sin sin dy = = 3 sin 16a l 0 l l m 0 l 2 (2k + 1)(2n + 1) k , n le Nghim ca bi ton l: at 2 x y at 2 16 + sin sin sin u ( x , y, t ) = cos l l l l 3 2

at (2k + 1) 2 + (2n + 1) 2 sin l 16 sin 2k + 1 x sin 2n + 1 y + 3 k =1 n =1 (2k + 1)(2n + 1) (2k + 1) 2 + (2n + 1) 2 l l

6. BI TON CAUCHY- PHNG TRNH TRUYN NHIT TRONG THANH V HN V NA V HN 1. Bi ton ban u: Bi ton Cauchy ca phng trnh truyn nhit - phng trnh loi parabolic l bi ton gii phng trnh: 2 2u 2 u (1) =a x 2 t 2 vi cc iu kin u : u ( x, t ) t =0 = u o ( x ) - < x < , - < y <, t 0 (2) Dng phng php phn li bin s ta tm nghim di dng: u(x, t) = X(x).T(t) Ly o hm theo x v t ri a vo phng trnh (1) ta c: T(t) + a2T(t) = 0 (3) X(x) + X(x) = 0 (4) vi l hmg s. Phng trnh (3) cho nghim l:

u h ( x, t ) = e h a t [C1 (h ) cosh x + C 2 (h ) sinh x ] u(x,t) l nghim ring ca (1) vi C1 v C2 l cc h s c th ph thuc h. H cc nghim y l mt tp hp v hn khng m c. Do ta s tm nghim ca bi ton di dng tch phn theo tham bin h.
2 2

T( t ) = Ce a t vi C l mt hng s Mt khc nhit ca thanh khng th t n khi t tin n . Do vy phi l s dng. Kt hp vi nghim ca phng trnh (4) ta c:

u ( x , t ) = u h ( x , t ) = e h

2a 2t

[C1 (h ) cosh x + C2 (h ) sinh x ]dh

Khi t = 0 ta c:

u ( x , t ) t =0 = u o ( x ) =

[C1 (h ) cosh x + C2 (h ) sinh x ]dh


169

Gi s hm uo(x) khai trin c di dng tch phn Fourier ta s c: 1 C1 (h ) = u o () cosh d 2 1 C 2 (h ) = u o () sinh d 2 T ta c: 1 h 2a 2t u ( ) e cosh( x ) d u ( x, t ) = o dh 2

1 h 2a 2t u ( x, t ) = e cosh( x )dh u o ( )d 0
ta tnh tch phn n bn trong bng phng php i bin: = ha t ; =
0

(5)

x a t

h 2a 2 t

cosh( x )dh =

1 a

e t
0

cos d =

1 a t

I()

Trong

I() = e cos d
0

(6)
2

Sau khi ly o hm ca (6) theo ri tch phn tng phn ta c:


I() I() = I() = I() = Ce 4 2 I() 2 Khi = 0 ta c: 2 C = I ( 0) = e d = 2 0

4 e nn: I() = 2
h 2a 2 t

( x ) 2 cosh( x )dh = exp e 2 a t 0 4a t Thay vo ta c: ( x ) 2 1 u ( x, t ) = (7) u o () exp 4a 2 t d 2a t Cng thc (7) c gi l cng thc Poisson ca bi ton Cauchy. i vi bi ton Cauchy trong khng gian n chiu ca qu trnh truyn nhit: 2 2u 2u 2u 2 u =a x 2 + x 2 + L + x 2 t 2 1 21 n vi cc iu kin u : u ( x1 , x 2 ,..., x n , t ) t =0 = u o ( x1 , x 2 ,..., x n ) Ngi ta chng minh c nghim ca phng trnh l:
170

1 n 2 L u ( , ,..., ) exp 2 ( i x i ) d1...d n1 n 0 1 2 n 4a t i=1 2a t i vi bi ton truyn nhit trong thanh na v hn ta xt cc bi ton nh ng vi tng trng hp ring ri mi xt n bi ton tng qut. 2. Bi ton 1: Truyn nhit trong thanh na v hn, cch nhit u mt thanh bit nhit ban u ca thanh bng khng: Khi ny ta phi gii phng trnh: 2 2u 2 u = a 0 x , t 0 (8) x 2 t 2 vi cc iu kin : u ( x , t ) t =0 = u o ( x ) (9)

u ( x1 , x 2 ,..., x n , t ) =

u =0 (10) t t =0 gii bi ton ta ko di chn hm uo(x) sang phn m ca trc x v p dng cng thc (7). Khi n s tho mn (8). Mt khc theo (7) ta cng s c: ( x ) 2 1 u = 3 u ( )( x ) exp o 4a 2 t d x x =0 4a t t x =0 T suy ra: ( x ) 2 ( x ) 2 1 0 1 u ( x, t ) = u o () exp 4a 2 t d + 2a t u o () exp 4a 2 t d 2a t 0 Trong tch phn u ta i bin = - v do tnh cht chn ca u() ta s c: ( x ) 2 ( x ) 2 1 0 1 u ( x, t ) = u o () exp 4a 2 t d + 2a t u o () exp 4a 2 t d 2a t 0

( + x ) 2 ( x ) 2 u ( x, t ) = u o ()exp 4a 2 t + exp 4a 2 t d 2a t 0 3. Bi ton 2: Truyn nhit trong thanh na v hn bit nhit ban u ca thanh mt u mt lun lun gi 0 . Khi ny ta phi gii phng trnh: 2 2u 2 u =a 0 x , t 0 t 2 x 2 vi cc iu kin : u ( x,0) t =0 = u o ( x ) 1

u ( x, l) t =0 = 0 gii bi ton ny ta ko di l hm uo(x) sang phn m ca trc x v p dng cng thc (7) ta s c: 2 1 u ( x , t ) x =0 = u o () exp 4a 2 t d = 0 2at t Tch phn ny bng 0 v uo() l hm l. T suy ra:
171

( x ) 2 ( x ) 2 1 u ( x, t ) = u o () exp 4a 2 t d + 2a t u o () exp 4a 2 t d 2a t 0 1
0

Trong tch phn u ta i bin = - v do tnh cht l ca u() ta s c: ( + x ) 2 ( x ) 2 1 u ( x, t ) = u o ( )exp 4a 2 t exp 4a 2 t d 2a t 0


4. Bi ton 3: Truyn nhit trong thanh na v hn bit nhit u mt ca thanh v nhit ban u ca thanh bng 0. Khi ny ta phi gii phng trnh: 2 2u 2 u =a 0 x , t 0 x 2 t 2 vi cc iu kin : u ( x , t ) x =0 = q ( t )

u ( x , t ) t =0 = 0 Ta gii bi ton ny bng phng php bin i Laplace ca hm phc. Gi s u(x, t) v q(t) l cc hm gc trong php bin i Laplace. p dng php bin i Laplace cho u(x, t) v cc o hm ca n ta c: 2 u ( x, t ) u ( x , t ) p 2 U ( x , p) u ( x , t ) U( x , p); pU ( x , p); 2 t t Coi p l thng s v t cc biu thc trn vo phng trnh truyn nhit ta c: 2 U ( x , p) pU( x, p) = a 2 (10) x 2 Vy ta phi gii phng trnh (10) vi iu kin: U ( x , p ) x = 0 = Q( p ) q ( t ) (11) Nghim tng qut ca (10) c dng: p p + C 2 exp U( x , p) = C1 exp x a a x
Hm U(x, p) l hm b chn khi x nn C1 = 0. T iu kin (11) ta suy ra: C2 = U(0, p) = Q(p) Vy nghim ca phng trnh (10) tho mn (11) s l: p U( x , p) = Q(p) exp a x Ta vit li n di dng: p 1 U( x , p) = pQ(p) exp x p a p dng cng thc Duhamel: pF(p)G(p) g(0)f(t)+f*g v tnh cht:
172

p 1 x x 1 erf exp x = Erf a p 2 a t 2 a t Trong : 2 x erf ( x ) = exp( 2 )d 0 Ta s rt ra: t x u ( x, t ) = q( t )Erf () + Erf q( t )d 2a 0 Mt khc ta bit rng: 2 Erf () = 1 exp( 2 )d = 0 0

2 x Erf = 1 2a = 0 T suy ra:

x 2a 0

exp(

)d
=0

x2 exp = 3 4a 2 2 2a x

x2 x t q ( t ) u ( x, t ) = 3 exp d 4a 2 2a 0 2

5. Bi ton 4: Truyn nhit trong thanh na v hn, bit nhit ban u ca thanh v bit nhit ti u mt. Khi ny ta phi gii phng trnh: 2 2u 2 u =a 0 x , t 0 t 2 x 2 vi cc iu kin : u ( x , t ) x =0 = u 1 ( t )

u ( x , t ) t =0 = u o ( x ) gii bi ton ny ta dng kt qu ca bi ton 2 v bi ton 3 v nguyn l chng nghim. C th ta tm nghim u(x, t) di dng: ) u ( x, t ) = ~ u ( x, t ) + u ( x, t ) u ( x, t ) l nghim ca bi ton: Trong ~ 2~ u 2~ 2 u =a x 2 t 2 vi cc iu kin: ~ u ( x , t ) t =0 = u o ( x ) ; ~ u ( x , t ) x =0 = 0 ) cn u ( x, t ) l nghim ca bi ton: ) 2) 2u 2 u =a x 2 t 2 vi cc iu kin:
173

) ) u ( x , t ) t =0 = 0 ; u ( x , t ) x =0 = u 1 ( t ) Theo kt qu bi ton 2 v bi ton 3 ta suy ra: ( + x ) 2 ( x ) 2 1 ~ u (x, t ) = u o ( )exp 4a 2 t exp 4a 2 t d 2a t 0


) u ( x, t ) = x2 x t u1 ( t ) 3 exp 4a 2 d 2a 0 2

V d: Gi s trong mt ng na v hn ti mt u ng x = 0 b chn bi mt tm khng thm thu v ti x = l cng c mt tm nh vy. Trong on [0, l] c mt mi trng vi h s khuych tn D1 = a-2 c cha y mt loi cht vi h s m c uo. Trong phn cn li ca ng [l, ] c mt mi trng vi h s khuych tn D2 = b-2 v trong khng cha cht trong on [0, l]. Ti thi im t = 0 ta b tm chn ti x = l v khi bt u qu trnh khuych tn. Gi nng cht khuych tn ti x thi im t l u(x, t). Trc ht ta thit lp phng trnh khuych tn trong ng. Gi u1(x, t) l nng ca cht lng trong khong 0 x l. Khi ta c: u1 1 2 u1 0xl (13) = t a 2 x 2 vi iu kin: u1 u 1 ( x , t ) t =0 = u o ; =0 x x =0 Gi u2(x, t) l hm nng cht khuych tn trong on [l, ] ta c: u 2 1 2 u 2 = 2 lx (14) t b x 2 vi iu kin: u 2 ( x, t ) t =0 = 0; u 2 ( x, t ) x = = 0 Mt khc ti im x = l th u1(x, t) v u2(x, t) phi tho mn iu kin lin tc (kt dnh), ngha l: u 1 ( x , t ) x =l = u 2 ( x , t ) x =l

1 2 u1 (l, t ) 1 2 u 2 (l, t ) = 2 a 2 x 2 b x 2 Gi s u1(x, t) v u2(x, t) v o hm ca chng l cc hm gc trong bin i Laplace, ta s c cc hm nh l: u 1 ( x , t ) U1 ( x , p ) u 2 ( x , t ) U 2 ( x , p) u1( x , t ) u 2 ( x , t ) pU1 ( x , p) u o pU 2 ( x , p) t t 2 u1( x , t ) 2 u 2 ( x, t ) 2 U1 ( x , t ) 2 U 2 (x, t ) x 2 x 2 x 2 x 2 T cc phng trnh (13) v (14) v cc h thc trn ta suy ra:
174

1 2 U1 ( x , t ) pU1 ( x , p) = 2 + uo x 2 a 1 2 U 2 ( x, t ) pU 2 ( x , p) = 2 x 2 b Nghim tng qut tng ng s l: U1 ( x , p) = C1chax p + C 2shax p +

U 2 ( x , p) = C3 exp b p ( x l) + C 4 exp b p ( x l)

uo p

(15)

(16)

Mt khc t iu kin

u1 x

= 0 ta suy ra
x =0
x =0

dU1 ( x, p) = 0 . T ta c: dx x =0

C1a pshax p + C 2 a pchax p

= C 2a p = 0

Vy C2 = 0 Mt khc u 2 ( x, t ) x = = 0 nn nh ca u2(x, t) l mt hm b chn khi x v do C3 = 0. Khi (15) v (16) tr thnh: u U1 ( x , p) = C1chax p + o p

U 2 ( x, p) = C4 exp b p ( x l) Xt n iu kin kt dnh ta c: U1 (l, p) = U 2 (l, p) 1 dU1 ( x , p) 1 dU 2 ( x , p) = 2 2 a dx b dx x =l x =l T ta c h phng trnh: u C1chal p + o = C 4 p 1 C a pshal p = 1 C b p 1 4 b2 a 2 Gii h phng trnh trn ta c: u oa = C 1 p achal p + bshal p u o bshal p C = 4 p achal p + bshal p Vy: u o achax p u U1 ( x, p) = o p p achal p + bshal p

u o bshal p exp b p ( x l) p achal p + bshal p Sau khi dng bin i Laplace ngc ta c cc hm u1(x, t) v u2(x, t). U 2 ( x, p) =

) [

175

7. BI TON TRUYN NHIT TRONG THANH V HN V NA V HN C NGUN NHIT 1. Nguyn l Duhamel: Trc ht ta xt nguyn l Duhamel ca phng trnh truyn nhit khng thun nht trong khng gian n chiu Rn. Nu H(, x, t) vi mi gi tr ca tham bin l nghim ca phng trnh: H = a 2 H (1) t vi iu kin: H ( , x , t ) t = 0 = h ( x , ) (2) khi hm:

u ( x, t ) = H(, x, t )d
0

(3)

l nghim ca phng trnh: u = a 2 u + h ( x , ) (4) t v tho mn iu kin: u ( x , t ) t =0 = 0 Ta p dng vo cc bi ton sau y 2. Bi ton 1: Truyn nhit trong thanh v hn c ngun nhit vi nhit ban u bng 0. Khi ny ta gii phng trnh: 2 u 2 u =a + f (x, t ) -< x < ; t 0 t x 2 vi iu kin ban u: u ( x , t ) t =0 = 0 S dng nguyn l Duhamel v kt qu cho bi cng thc Poisson ca bi ton Cauchy ta c nghim l: t f ( , ) ( x ) 2 exp 2 u ( x, t ) = d d 4 a ( t ) 0 2a ( t ) y hm H(, x, t ) l tch phn:

( x ) 2 f (, ) exp 4a 2 t d

3. Bi ton 2: Truyn nhit trong thanh v hn c ngun nhit vi nhit ban u bng uo(x). Khi ny ta gii phng trnh: 2 u 2 u + f (x, t ) =a -< x < ; t 0 x 2 t vi iu kin ban u: u ( x , t ) t =0 = u o ( x )
176

Dng phng php xp chng nghim v kt qu ca bi ton 1 ta c nghim: t f ( , ) ( x ) 2 ( x ) 2 1 u ( x, t ) = u o ( ) exp 4a 2 t d + 2a ( t ) exp 4a 2 ( t ) d d 2a t 0


4. Bi ton 3: Gi bi ton truyn nhit: 2 u 2 u + f (x, t ) =a -< x < ; t 0 x 2 t vi iu kin ban u: u ( x , t ) t =0 = u o ( x )

u ( x , t ) x =0 = u1 ( t ) Dng phng php xp chng nghim v kt qu ca cc bi ton trn ta c nghim: ( x ) 2 ( + x ) 2 1 u ( x, t ) = u o ( )exp 4a 2 t exp 4a 2 t d 2a t 0
+ x2 1 t u1 ( t ) exp 4a 2 d 2a 0 3 2

( + x ) 2 x t f ( , ) ( x ) 2 + exp 2 d d exp 2 2a 0 4a ( t ) 0 t 4a ( t )
8. BI TON HN HP CA PHNG TRNH LOI HYPERBOLIC TRUYN NHIT TRONG THANH HU HN V MIN HU HN 1. t bi ton: Ta xt bi ton truyn nhit trong khi lp phng ng cht, khng c ngun nhit vi iu kin bit nhit ban u ca khi v nhit trn bin lun lun bng khng. Bi ton ny a n phng trnh: 2 2u 2u 2u u 2 u = a 2 u + 2 + 2 =a (1) 2 2 z x y t x

Trong min T t 0 v 0 < x < l1; 0 < y < l2; 0 < z < l3 vi iu kin : u ( x, y, z, t ) t =0 = u o ( x, y, z)

(2) (3)

u x =0 = u x =l = u y =0 = u y =l = u z =0 = u z =l = 0
1 2 3

Dng phng php tch bin ta tm nghim di dng: u(x, y, z, t) = u*(x, y, z).T(t) v ta c: T' ( t ) + a 2 T( t ) = 0 (4) u ( x , y, z) + u ( x , y, z) = 0 (5) T (4) c nghim ring khng suy bin v km iu kin b chn ca u khi t ta suy ra nghim ca n c dng: T(t) = Cexp(-a2t) Mt khc ta li dng phng php tch bin tm u*(x, y, z), ngha l:
177

u*(x, y, z) = X(x).Y(y).Z(z) Sau khi o hm v thay vo phng trnh ta c: X( x ) Y( y) Z(z) + + +=0 X ( x ) Y ( y ) Z( z ) T ta rt ra h phng trnh vi phn tng ng l: X(x) + X(x) = 0 Y(y) + Y(y) = 0 ++=0 Z(z) + Z(z) = 0 i vi hm X(x), Y(y), Z(z) ta thy do iu kin bin (3) chng phi tho mn cc iu kin sau: X(0) = X(l1) = 0; Y(0) = Y(l2) = 0; Z(0) = Z(l3) = 0 Gii h phng trnh vi phn trn vi cc iu kin bit ta suy ra cc gi tr ring (-): k 22 m22 n22 k = 2 ; k = 2 ; k = 2 l1 l2 l3

k 2 m2 n 2 k ,m ,n (l1 , l 2 , l3 ) = l2 + l2 + l2 2 3 1 Cc hm ring tng ng khi s l: 8 k m n u k ,m ,n ( x , y, z) = sin x sin y sin z l1 l2 l3 l1l 2 l3 Vy nghim tng qut ca bi ton l: 2 2 k 2 m2 n 2 n m k z y sin x sin t sin u ( x , y, z, t ) = C k ,m ,n exp a + + 2 l2 l3 l2 l1 l2 l3 k =1 m =1 n =1 2 1 Trong Ck,m,n c suy ra t iu kin: 8 l1 l2 l3 k m n C k ,m ,n = u o (, , ) sin sin sin ddd l1l 2 l3 0 0 0 l1 l2 l3 2. Bi ton 1: Gii phng trnh truyn nhit: 2 u 2 u =a t x 2 Trong min T t 0 v 0 < x < l vi iu kin : u ( x , t ) t =0 = u o ( x )
2

u x = 0 = u x =l = 0 Nghim l:
k2 k u ( x, t ) = C k exp a 2 2 2 t sin x l l k =1 Trong : 2l k C k = u o ( ) sin d l0 l

178

3. Bi ton 2: Gii phng trnh truyn nhit: 2 u 2 u =a + f (x, t ) t x 2 Trong min T t 0 v 0 < x < l vi iu kin : u ( x , t ) t =0 = u o ( x )

(6)

u ( x , t ) x = 0 = u ( x , t ) x =l = 0 Ta tm nghim di dng: k u ( x , t ) = Tk ( t ) sin x (7) l k =1 Gi s uo(x) c cc o hm lin tc trong khong (0, l) v uo(0) = uo(l) = 0 cn hm f(x, t) c cc o hm ring cp 1 theo x lin tc trong khong (0, l) v: f(0, t) = f(l, t) = 0 Khi ta c th thu c cc khai trin Fourier ca chng l: k f ( x , t ) = f k ( t ) sin x (8) l k =1 k u o ( x ) = k sin x (9) l k =1 vi: 2l k f k ( t ) = f ( x, t ) sin xdx l0 l 2l k k = u o ( x ) sin xdx l0 l Sau khi ly o hm hai v ca (7) v ch n (8) v (9) ta c: 2 k k + T ( t ) a T ( t ) f ( t ) sin x = 0 k k k l k =1 l Nn: 2 k (10) Tk ( t ) + a Tk ( t ) = f k ( t ) l Khi t = 0 ta c: k k u t =0 = u o ( x ) = Tk (0) sin x = k sin x l l k =1 k =1 nn: (11) Tk(0) = k Sau khi gii (10) vi iu kin (11) ta c Tk(t), ngha l c nghim ca bi ton.
4. Bi ton 3: Gii phng trnh truyn nhit: 2 u 2 u =a + f (x, t ) t x 2 Trong min T t 0 v 0 < x < l

(12)

179

vi iu kin : u ( x , t ) t =0 = u o ( x )

u ( x, t ) x =0 = 1 ( x ); u ( x, t ) x =l = 2 ( x ) Ta gii bi ton ny bng phng php chng nghim, ngha l tm nghim di dng: x u ( x, t ) = u ( x, t ) + ~ u ( x , t ) + 1 ( t ) + [ 2 ( t ) 1 ( t )] l Trong u ( x , t ) l nghim ca phng trnh: 2 u 2 u =a + f (x, t ) t x 2 vi cc iu kin: u ( x , t ) t =0 = u o ( x ) u ( x , t ) x = 0 = u ( x , t ) x =l = 0 u ( x, t ) l nghim ca phng trnh: Cn ~ 2~ ~ u 2 u =a + f (x, t ) t x 2 vi cc iu kin: ~ u ( x , t ) t =0 = 0 ~ u ( x, t ) = ~ u ( x, t ) = 0

u ( x, t ) c rt ra t bi ton 2. R rng u ( x , t ) cc rt ra t bi ton 1 v ~

x =0

x =l

180

You might also like